Você está na página 1de 123

Universidad La Salle. Facultad Mexicana de Medicina.

Curso de Extensin Universitaria para la Preparacin del Examen Nacional para Aspirantes a Residencias Mdicas. Examen Diagnstico.

Nombre: Examen diagnstico. Nmero de intentos: 3. Vigencia: 5 de Febrero del 2014. Horario: 9:00 a.m. a 9:00 p.m. Programar aleatorio. (5 bloques con 20 preguntas cada uno).

1.- Femenino de 27 aos de edad con deseo de un embarazo, antecedentes de G3, A2, P1, se le realiza una histerosalpingografa, se constata que existe un sndrome de Asherman. Ello significa que se trata de:

a) b) c) d)

tero bicorne Sinequias uterinas Endometriosis en la trompa Insuficiencia istmico cervical

El sndrome de Asherman es una enfermedad ginecolgica rara que se caracteriza por la presencia de sinequias (adherencias) intrauterinas que pueden ocasionar amenorrea (ausencia de perodos menstruales regulares) e infertilidad.

En 1894 Heinrich Fritsch describe por primera vez la presencia de sinequias intrauterinas de tipo postraumtico, en una paciente que desarroll una amenorrea secundaria a un curetaje. Posteriormente en 1927 Bass inform de veinte casos de atresia (oclusin de una abertura natural) cervical tras abortos inducidos, pero no fue hasta 1948, cuando Joseph G. Asherman recopil la informacin hasta entonces existente y acu el nombre con el que se conoce actualmente a la enfermedad.

Asherman describi originalmente dos tipos diferentes de amenorrea secundaria, en funcin de su etiologa (estudio de las causas de las enfermedades): la amenorrea traumtica atrtica, debida a estenosis del orificio cervical interno y la amenorrea debida a adherencias intrauterinas. Posteriormente ambas entidades se agruparon en una nica entidad bajo el nombre de sndrome de Asherman.

Suele presentarse en mayor proporcin tras dilataciones y curetajes uterinos de repeticin y sobre todo si se realizan durante el embarazo o si existe infeccin uterina en el momento en el que se realizan estas intervenciones.

Las adherencias intrauterinas pueden producirse debido a cualquier factor que lleve a una destruccin de las paredes del miometrio (capa muscular de la pared del tero). Sin embargo, hay que distinguir entre factores predisponentes, siendo el principal de ellos el embarazo y factores causales, entre los que se encuentran: traumatismos uterinos, intervenciones quirrgicas que afecten al tero, agentes fsicos o qumicos e infecciones uterinas por tuberculosis o esquistosomiasis. En cualquier caso, el factor ms importante es el trauma uterino en el momento del parto o el puerperio.

El cuadro clnico es muy variable y las manifestaciones clnicas varan con el grado de oclusin de la cavidad uterina y la severidad de las adherencias, pudiendo presentarse: esterilidad cuando la oclusin de la cavidad uterina incluye porciones proximales (ms cerca de un centro, tronco o lnea media) de las trompas de Falopio o cuando las adherencias impiden la nidacin del huevo; las pacientes presentan con frecuencia amenorrea, oligomenorrea (disminucin de la frecuencia de las menstruaciones), dismenorrea (menstruacin dolorosa) y abortos repetidos. Hysteroscopic treatment of severe Asherman's syndrome and subsequent fertility. Capella-Allouc S; Hum Reprod.

2.- Se trata de femenino de 31 aos de edad, segunda gesta a trmino sin anormalidades en el transcurso de ste. Inicia trabajo de parto de forma espontnea, con evolucin normal hasta que se rompe la bolsa, con una dilatacin de 4 cm. A partir de entonces, comienza con hemorragia de sangre roja, en moderada cantidad y aparecen signos de sufrimiento fetal agudo. El estado general de la mujer es bueno y la dinmica uterina es normal. Este cuadro corresponde a: a) b) c) d) Abruptio placentae. Rotura uterina. Rotura de vasa previa. Placenta previa central

La vasa previa es una condicin de alto riesgo obsttrico en la cual vasos fetales o placentarios cruzan el segmento uterino por debajo de presentacin. Estos vasos estn desprotegidos de gelatina de Wharton o tejido placentario, lo que los hace altamente vulnerables y susceptibles de ruptura o laceraciones en cualquier perodo del embarazo, principalmente en el momento del parto. Tambin es frecuente la compresin

de estos vasos, especialmente durante el tercer trimestre de la gestacin, lo que puede condicionar asfixia y muerte fetal.

Esta condicin ocurre como resultado de que vasos velamentosos cruzan por el segmento uterino debido a una insercin velamentosa del cordn, situacin en la cual el cordn umbilical se inserta en las membranas ovulares en vez del tejido placentario (vasa previa tipo I), o por el cruce de vasos fetalesentre uno o ms lbulos accesorios de la placenta (vasa previa tipo II) (Figura 1).

Figura 1 . A, Vasa previa tipo I, debida a insercin velamentosa. B, Vasa previa tipo II, debida a cotiledones aberrrantes. Reproducido de Daly-Jones y cols. Ultrasound 2008.

Vasa previa se puede presentar si existe alguna (o ninguna) de las siguientes condiciones: placenta baja (que puede ser causa de abortos previos seguidos por legrado o por operaciones uterinas, que provocan cicatrices en el tero), placenta bilobada o de lbulo succensuriado, embarazos resultado de fertilizacin in vitro, o embarazos mltiples (5-6). El sangrado por vasa previa no es doloroso. Otros sangrados por complicaciones o por nacimiento no necesariamente son sin dolor.

Referencias: 1. Oyalese Y, Smulian JC. Placenta previa, placenta acreta, and vasa previa. Obstet Gynecol 2006; 107: 927-941. 2. Oyalesse KO, Turner M, Less C, Campbell S. Vasa previa: an avoidable obstetric tragedy. Obstet Gynecol Surv 1999; 54: 138-145. 3. Seplveda W, Sebire NJ, Harris R, Nyberg DA. The placenta, umbilical cord, and membranas. In Diagnostic Imaging of Fetal Anomalies, Nyberg DA, MaGahan JP, Pretorius DH, Pilu G (eds). Philadelphia, PA: Lippicont Williams & Wilkins 2003, 85-132. 4. Daly-Jones E, John A, Leahy A, McKenna C, Sepulveda W. Vasa praevia; a preventable tragedy. Ultrasound 2006; 16: 8-14.

5. Derbala Y, Grochal F, Jeanty P. Vasa previa. J Prenat Med 2007; 1: 2-13. 6. Fung TY, Lau TK. Poor perinatal outcome associated with vasa previa. It is preventable? A report of three cases and review of the literature. Ultrasound Obstet Gynecol 1998; 12: 430-433. 7. Robert JA, Sepulveda W. Fetal exsanguination from ruptured vasa previa: still a catastrophic event in modern obstetrics. J Obstet Gynaecol 2003; 23: 574. 8. Cordero DR, Helfgott AW, Landy HJ, et al. A non-hemorrhagic manifestation of vasa previa: a clinicopathologic case report. Obstet Gynecol 1993; 82: 698-700. 9. Schachter M, Tovbin Y, Arieli S, et al. In vitro fertilization as a risk factor for vasa previa. Fertil Steril 2002; 78: 642-643.

3.- Masculino en edad preescolar. Acude al servicio de consulta externa. Antecedentes: historia de geofagia. Hace 2 semanas presenta tos hmeda y "silbido del pecho". E.f.: temp. :37.2 c., mala higiene personal, trax con sibilancias espiratorias bilaterales. Se reportan labs. BH con anemia y eosinofilia. El diagnstico ms probable en ste paciente es:

a) b) c) d)

Ascaris. Sx de loeffer Tenias. Tricocefalos. Enterobius.

En 1932 Loeffler describe un sndrome caracterizado por sintomatologa respiratoria compatible con "pulmn sibilante", radiografa de trax con infiltrado intersticial bilateral, difuso, abigarrado y transitorio compatible con neumonitis asociada de modo obligatorio a eosinofilia perifrica. Este sndrome clsicamente se presenta cuando existe migracin parasitaria alveolo capilar pulmonar y es causado por larvas de parsitos con ciclo hstico tisular como Ascaris l, Strongyloides s, Ancylostoma duodenale, Necator americanus y Filarias (1-2-3) excepcionalmente est asociado al sndrome de larva migrans cutnea (1). La invasin por larvas de helmintos puede originar infiltracin pulmonar, con marcada dificultad respiratoria por el espasmo bronco-alveolar como respuesta a la invasin parasitaria (pulmn sibilante), cuadro pulmonar inflamatorio transitorio, con respuesta alrgica local que explica y exige para su diagnstico, una intensa eosinofila perifrica(5-67) Este sndrome es ms frecuente en personas que se infectan por primera vez o que viven en zonas no endmicas; por hipersensibilidad a las larvas presentndose como una neumona eosinoflica, aguda, benigna y con infiltrados pulmonares migratorios y transitorios(7), no existiendo restriccin para ninguna edad, describindose aun en neonatos ya desde el ao 2001 con cuadro clnico similar al de nios de mayor edad (4).

Cuadro I. Principales causas de eosinofilia Parasitosis Helmintiasis con fase tisular

Infecciones micticas Coccidioidomicosis Enfermedades alrgicas Asma bronquial Rinitis alrgica Alergia a medicamentos Edema angioneurtico Fiebre del heno Neoplasias Enfermedad de Hodking Carcinomas Sarcomas Tumores seos Tumores de ovario Leucemia mieloide crnica Leucemia eosinoflica Enfermedades gastrointestinales Gastritis eosinoflica Enfermedades inmunolgicas Artritis reumatoide Vasculitis Sndrome de Wiskott-Aldrich Sndrome de Hiper-IgE Dficit selectivo de IgA Injerto contra husped Enfermedades respiratorias Aspergilosis broncopulmonar alrgica Sinusitis aspergilar alrgica Neumona crnica eosinfila Intoxicaciones Intoxicacin con fsforo Inhalacin de humos Enfermedades dermatolgicas Urticaria aguda Pnfigo Penfigoide ampolloso Sndrome de Wells Dermatitis atpica Hiperplasia angiolinfoide con eosinofilia Herpes gestationis Convalecencia de infecciones Escarlatina Fiebre reumtica Otros Eosinofilia post-irradiacin Sndrome hipereosinoflico REFERENCIAS

1.- Del Giudice P, Desalvador F, Bernard E, Caumes E, Vandenbos F, et al. 2002. Lffler's syndrome and cutaneous larva migrans: a rare association. British J Dermatol 2002; 147: 385- 7. 2. Botero D, Restrepo M. Parasitosis Humanas. 3 Ed., Medelln Colombia: Corporacin para Investigaciones Biolgicas 1998. 3.- Noem I, Atias A. Eosinofilia y parasitosis. En Atias A. Parasitologia Mdica. Mediterraneo, Santiago-Chile. 2000. 4.- Fujimura J, Murakami Y, Tsuda A, Chiba T, Migita M, Fukunaga Y. 2001.A neonate with Loeffler syndrome.Journal of Perinatology 2001; 21: 207-8. 5.- Hunninghake GW, RichersonHB. Neumonitis por hipersensibilidad y neumonias eosinfilas. En Harrison TR. Principios de Medicina Interna. 15Ed. McGraw-Hill -Interamericana de Espaa, Madrid- Espaa. 2001. 6.- Chusid MJ. Eosinophilia in childhood. Immunol and Allergy Clinics North America 1999; 19: 327-46. 7.- Rothenberg ME. Eosinophilia N Engl J Med 1998; 338: 1592. 8.- Noemi IH. Eosinofilia y parasitosis. Rev. Chil. Pediatr. 1999; 70: 1-7. 9.- Hotez PJ, Broker S, Bethony JM, Bottazzi ME, Loukas A. Hookworm Infection. N Engl. J Med 2004; 351(8): 799-808. 10.- Yilderan A, Ikinciogullari A. In the light of recent advances: eosinophil, eosinophilia and idiopathic hypereosinophilic syndrome. Turk Haematol 2005; 22(3): 107-16.

4.- Femenino de 3 meses de edad que hace tres das inicia con rinorrea hialina, tos y estornudos. El da de ayer presenta taquipnea con 56 rpm, tiraje intercostal con aleteo nasal. A su ingreso se observan distrs respiratorio con sibilancias inspiratorias y espiratorias y algunos crepitantes bilaterales. Una Rx de trax muestra hiperinsuflaccin bilateral con una atelectasia laminar y corazn pequeo. Presenta: pH 7,24; pCO2: 58 mmg; CO3H: 21 mEq/L. El diagnstico ms probable es? a) b) c) d) Bronquiolitis Bronconeumona bilateral. Crisis asmtica de origen infeccioso. Neumonitis intersticial.

La bronquiolitis es la enfermedad del tracto respiratorio inferior ms frecuente en los dos primeros aos de vida. Aunque su mortalidad es baja, su elevada morbilidad origina una gran demanda asistencial y causa un importante nmero de ingresos hospitalarios. Se presenta de forma epidmica durante el invierno y principio de la primavera. Su etiologa es viral, y el virus respiratorio sincitial (VRS) es el agente que con ms frecuencia se asla. El diagnstico es eminentemente clnico. En la mayora de los casos, cursa de forma benigna y autolimitada; las formas ms graves se presentan en lactantes ms pequeos y en los que tienen algn factor de riesgo asociado, en estos casos la tasa de hospitalizacin es alta.

El cuadro comienza como un catarro de vas altas, con estornudo, tos, rinorrea y en ocasiones fiebre; y en el transcurso de 2 3 das se intensifica la tos, aparecen los sntomas de obstruccin de la va respiratoria respiratoria inferior con aumento del trabajo respiratorio, taquipnea e irritabilidad.

En los casos ms graves, la dificultad respiratoria es marcada y presenta rechazo de las tomas de alimento y postracin. La apnea puede ser la primera manifestacin de bronquiolitis en lactantes pequeos. Dada la evolucin del curso clnico de la bronquiolitis, se recomienda reevaluar a las 24-48 horas a todos los lactantes menores de 12 meses que, en ambiente epidmico consulten por un catarro de vas altas. Adems, es conveniente advertir a los padres de la necesidad de consultar ante la presencia de dificultad respiratoria, agitacin, mal color, vmitos o rechazo de la alimentacin. A la exploracin fsica, podemos encontrar retracciones costales (tiraje subcostal e intercostal) y la auscultacin pulmonar puede ser normal, aunque en la mayora de los casos presenta espiracin alargada, roncus, sibilantes y crepitantes bilaterales. Los sibilantes pueden escucharse con el odo desnudo, sin ayuda del fonendoscopio. No existe un patrn radiolgico tpico en la bronquiolitis. Los hallazgos radiolgicos ms comunes son: el atrapamiento areo, engrosamientos peribronquiales, infiltrados intersticiales y atelectasias laminares o segmentarias. El hemograma es inespecfico y slo estara indicado realizarlo en aquellos casos en los que sospechemos alguna complicacin. Para el diagnstico de la hipoxemia se recomienda la pulsioximetra transcutnea, ya que tiene muy buena correlacin con la PaO2 sangunea. Se considera hipoxia leve del 96-98%; moderada, del 93-95%, y grave, menos de 93%. Gonzlez Caballero D, Gonzlez Prez- Yarza E. Bronquiolitis aguda: bases para un protocolo racional. An Esp Pediatra 2001; 55 (4): 355-64.

5.- Se trata de paciente femenino en quien se sospecha la presencia de insuficiencia suprarrenal primaria, en ste caso las alteraciones de laboratorio que podemos encontrar son: a) Hipercalcemia e hipernatremia b) Hipernatremia e hiperglucemia c) Hipocalcemia y leucopenia d) Hiperkalemia e hipoglucemia

En la insuficiencia suprarrenal primaria se observa disminucin en los niveles sricos de sodio, cloruro y bicarbonato, mientras que el potasio srico se eleva. Esta se debe a los efectos combinados de dficit de aldosterona, reduccin del filtrado glomerular y acidosis. Y sobre todo durante el estrs puede haber disminucin en los niveles de glucosa.

1.

Datos de laboratorio Hiponatremia: los niveles sricos bajos de sodio se debe a su prdida por la orina por dficit de aldosterona y al desplazamiento del sodio hacia el compartimento intracelular. Esta prdida de sodio extravascular reduce el volumen plasmtico y acenta la hipotensin.

2. Hiperkalemia: aumento de los niveles sricos de potasio. Se debe a los efectos combinados del dficit de aldosterona, la reduccin del filtrado glomerular y la acidosis. 3. Hipocortisolemia: los niveles de cortisol y aldosterona son bajos y no aumentan con la administracin de ACTH. 4. Hipercalcemia: aumento de los niveles sricos de calcio. Ocurre en un 10-20% de los pacientes de causa desconocida. 5. Cambios electrocardiogrficos: suelen ser inespecficos, aunque con lentificacin generalizada del trazado. 6. Hemograma: puede haber anemia normoctica, linfocitosis relativa y eosinofilia moderada. 7. Prueba de estimulacin de ACTH: prueba principal que confirma el diagnstico de insuficiencia suprarrenal, al evaluar la capacidad de las suprarrenales para producir esteroides, que suelen estar ausentes o disminuidos tanto en sangre como en orina tras la estimulacin de ACTH. 8. Determinacin de la ACTH: en la insuficiencia suprarrenal primaria o Enfermedad de Addison, la ACTH y sus pptidos afines, estn elevados en plasma ante la prdida del mecanismo de retroalimentacin del eje hipotlamo-hipfisariosuprarrenal. 9. hipertermia: la hormona del hipotlamo no controla la homeostasis

Williams GH, Dluhy RG. Enfermedades de la corteza suprarrenal. En Jameson JL (ed): Harrison. Endocrinologa. 1a ed. Madrid. MacGraw-Hill Espaa, 2006: 137-138.

6.- Femenino de 39 aos inicia recientemente con fatiga, somnolencia, piel seca, estreimiento y aumento de peso de 5 kg. Su tiroides est firme y tiene el doble del tamao normal. Cul de las siguientes pruebas de laboratorio confirma el presunto diagnstico de hipotiroidismo?

a) b) c) d)

Tiroxina srica (T4) Hormona estimulante de tiroides (TSH) en suero Triyodotironina srica (T3) Captacin de resina T3

Hipotiroidismo La instauracin es habitualmente lenta y progresiva. Los sntomas se relacionan con una disminucin en la actividad funcional de todos los sistemas del organismo. Los ms clsicos son cansancio, intolerancia al fro (carcter muy friolero), apata e indiferencia, depresin, disminucin de memoria y de la capacidad de concentracin mental, piel seca, cabello seco y quebradizo, fragilidad de uas, palidez de piel, aumento de peso, estreimiento pertinaz y somnolencia excesiva. En situaciones extremas puede evolucionar hacia la insuficiencia cardiaca, la hinchazn generalizada (mixedema), insuficiencia respiratoria y abocar al coma mixedematoso con prdida de conocimiento que conlleva un alto grado de mortalidad.

Al igual que el resto de enfermedades del tiroides, el hipotiroidismo es ms frecuente en el sexo femenino. Es a partir de los 40-50 aos cuando las mujeres tienden a desarrollar con ms frecuencia hipotiroidismo de causa autoinmune (tiroiditis de Hashimoto). El periodo postparto es igualmente propenso a la aparicin de este problema. La ciruga de tiroides y la aplicacin de yodo radioactivo representan situaciones de riesgo para el desarrollo de hipotiroidismo, lo que obliga a controlar evolutivamente la funcin tiroidea en estos casos.Los recin nacidos de madres hipertiroideas, hayan recibido o no tratamiento antitiroideo durante la gestacin, deben ser evaluados en este sentido. Las personas en las que se detectan anticuerpos antitiroideos (antimicrosomales, antitiroglobulina) tienden a desarrollar con el tiempo alteraciones de la funcin tiroidea, por lo que deben ser evaluados crnicamente de forma peridica. La determinacin de TSH es el parmetro ms sensible para el diagnstico del hipotiroidismo. Su elevacin es indicativa de que la funcin del tiroides es insuficiente. Este fenmeno se produce antes de que comiencen a descender en la sangre las concentraciones de hormonas tiroideas. Generalmente, en el hipotiroidismo establecido, adems de la elevacin de TSH, se produce un descenso de T4. El nivel de T3 con frecuencia se encuentra dentro de la normalidad. As pues, cuando aparecen sntomas sugestivos, el mdico solicitar una determinacin de TSH que es el mejor mtodo para descartar que exista hipotiroidismo. Puede acompaarse de una determinacin de T4 y de anticuerpos antitiroideos si se desea conocer si la causa se debe a fenmenos de autoinmunidad. En los casos de hipotiroidismo secundario debido a disminucin de la secrecin de TSH por parte de la hipfisis, el diagnstico se basa en confirmar concentraciones disminuidas de T4 y TSH en la sangre. Cuando la elevacin de TSH se acompaa de niveles normales de T4 la condicin es conocida con el nombre de hipotiroidismo subclnico. Si existe bocio puede ser conveniente realizar una ecografa tiroidea. Cuando existe sospecha de alteraciones en el desarrollo de la glndula o de deficiencia enzimtica, puede ser til obtener una gammagrafa tiroidea. Si se confirma un diagnstico de hipotiroidismo de causa autoinmune, es habitual evaluar la asociacin de alteraciones en otras glndulas como las suprarrenales, paratiroides o gnadas. REFERENCIAS (1.) Anderson R, Harnes J. 1975. Thyroid hormones secretion rates in growing and mature goats. J Anim Sci 40: 11301135. (2.) Anke M, Henning A, Grun M, Partschefeld M, Groppel B. 1977. Der einluss des mangan, zink, kupfer, jod, selen, molybdan und nickelmangels aauf die fortpflanzuggsleistung des wiederkauers. Mathem Natur Reihe (Leipzig) 26: 283-292. (3.) Balbuena O. 2003. Nutricin Mineral del Ganado. Sitio Argentino de Produccin Animal: 1-5, www.produccionanimal. com.ar. (4.) Beckett GJ, Beddows SE, Morrice PC, Nicol F, Arthur JR. 1987. Inhibition of hepatic deiodination of thyroxine is caused by selenium deficiency in rats. Biochem J 248: 443447.

(5.) Brem JJ, Pochon DO, Roux JP, Trulls H. 1998. Exploracin diagnstica de la funcin

tiroidea en ovinos. Rev Vet 8/9: 23-26. (6.) Castillo V. 2001. Cambios de la funcin tiroidea en cachorros alimentados con dietas comerciales con alto contenido de yodo. On line: http://www.idealibrary.com

7.- Masculino de 54 aos que acude con resultados de manometra esofgica los que muestran un aumento de la presin basal del esfnter esofgico inferior (EEI) junto a una disminucin franca de su relajacin con la deglucin, el diagnstico ms probable es: a) Esclerodermia con afectacin esofgica. b) Enfermedad por reflujo gastroesofgico c) Espasmo esofgico difuso. d) Acalasia.

Diagnstico de acalasia
Aspecto radiogrfico Un esofagograma puede mostrar la disminucin de peristalsis, la dilatacin del esfago proximal y el estrechamiento del esfago en su parte inferior. El paciente se traga una solucin de bario, con fluoroscopia continua que son grabaciones de rayos X para observar el flujo del fluido a lo largo del esfago, sin que se observe el movimiento peristltico normal del esfago. Hay un agudo estrechamiento en el esfnter esofgico inferior y reduccin del dimetro en la unin gastro-esofgica. La imagen que proyecta se denomina clsicamente en pico de loro o en cola de ratn. Por encima de la reduccin, el esfago a menudo se observa con una dilatacin de diversos grados a medida que poco a poco se va estirando en el tiempo. Por la falta de movimientos peristlticos, se suele observar en la radiografa un margen entre aire y lquido. Manometra esofgica Debido a su sensibilidad, el diagnstico es confirmado por medio de una manometra esofgica, que mide las presiones del esfago mediante una sonda nasoesofgica y permite comparar las presiones en situacin basal y durante la deglucin.4 Se inserta un tubo delgado a travs de la nariz, y se le instruye al paciente a deglutir varias veces. La sonda mide las contracciones musculares en diferentes partes del esfago durante el acto de la deglucin. La manometra revela la falla del EEI para relajarse con cada deglucin y la falta de peristaltismo funcional del msculo liso en el esfago. Para descartar complicaciones se suele acudir a una endoscopia digestiva alta.

Esquema manomtrico de acalasia demostrando contracciones aperistlticas, un aumento de la presin intraesofgica y el fallo de la relajacin del esfnter esofgico inferior. Debido a la similitud en los sntomas, la acalasia se puede confundir con trastornos ms comunes, tales como la enfermedad de reflujo gastroesofgico, la hernia de hiato, e incluso trastornos psicosomticos. REFERENCIAS BIBLIOGRFICAS 1. Garca Gutirrez A. Acalasia de esfago. http://www.sld.cu/galerias/pdf/uvs/cirured/acalasia..pdf Consultado Mayo 27, 2006. Disponible en:

Fareras Rozman, et al. Acalasia esofgica.Tratado de Medicina Interna. 15 Edicin. Espaa: Ediciones Harcourt; 2003. Seccin 2. Cap 20.p.354-9.

8.- Ingresa al servicio de urgencias masculino de 47 aos con hemorragia digestiva alta. No hay antecedentes de consumo de AINE. La endoscopia revela lcera gstrica en incisura angularis con un punto de hematina y mnimos restos de sangre oscura en el estmago. Se realizan biopsias del margen de la lcera y una biopsia antral para prueba rpida de ureasa positiva. La actitud ms correcta ante este paciente es:

a) Iniciar con sucralfato y Ranitidina 150 mg/da, tratamiento anti-Helicobacter pylori si la histologa confirma la presencia del germen. b) Esclerosis endoscpica de la lcera seguida de tratamiento con omeprazol, 20 mg/da durante 28 das. c) Iniciar tratamiento con Omeprazol 40mg/da durante 1 mes. d) Se debe indicar anti-Helicobacter pylori durante 14 das, seguido de un antisecretor hasta que se confirme la erradicacin del germen.

El Helicobacter pylori (HP) es una bacteria microaerfila, gramnegativa, de crecimiento lento y forma helicoidal con abundantes flagelos. Fue descubierta por dos mdicos autralianos. Robin Warren y Barry Marshall; trabajando en colaboracin, detectaron que este microorganismo se encontraba en casi todos los pacientes con inflamacin gstrica, lcera duodenal o gstrica. Basndose en estos resultados propusieron que HP estaba implicado en la etiologa de estas enfermedades. Antes de 1982, se pensaba que la mayor causa de la lcera pptica era el

estrs y el estilo de vida. Ahora se sabe que HP est implicado en ms del 90% de las lceras duodenales y hasta el 80% de las lceras gstricas. Gracias a los descubrimientos de Marshall y Warren, la lcera pptica no es una enfermedad crnica sino que puede ser curada con una pauta de tratamiento con antibiticos y con inhibidores de la secrecin cida. Afecta al 50 % de la poblacin mundial, ha sido identificado como el agente causal de la lcera pptica y se ha clasificado adems como carcingeno tipo I. Como resultado de su interferencia con la secrecin de cido por el estmago, esta bacteria es capaz de generar deficiencias en la absorcin de nutrientes y vincularse con la aparicin de manifestaciones carenciales o con el agente causal de enfermedades crnicas

El objetivo del tratamiento mdico ser promover la cicatrizacin de la lcera para prevenir la recurrencia de la hemorragia. As, la ranitidina puede prevenir la lcera duodenal en pacientes tomadores de AINES, pero es ineficaz en la prevencin de la ulcera gstrica. Por el contrario, la famotidina protege la mucosa gstrica contra tratamientos cortos de aspirina o naproxeno. Si se demuestra que existe infeccin por H.Pylori estar indicada su erradicacin para evitar recurrencias. Amoxicilina + Klaritromicina OD+ esomeprazol o lanzoprazol Erradicacin en 14 dias 100% De existir una complicacin, como la hemorragia digestiva, los pacientes debern ser internados, suprimir el aporte oral, efectuar el diagnstico de hemorragia, determinar la cuanta de la prdida hemtica y realizar un adecuado control de la hemodinamia.

Lahaie RG, Gaudreau C. Helicobacter pylori antibiotic resistance: trends over time. Canadian Journal of Gastroenterology. 2000;14(10):895899. Manes G, Balzano A, Iaquinto G, et al. Accuracy of the stool antigen test in the diagnosis of Helicobacter pylori infection before treatment and in patients on omeprazole therapy. Alimentary Pharmacology and Therapeutics. 2001;15(1):7379. McManus TJ. Helicobacter pylori: an emerging infectious disease. Nurse Practitioner. 2000;25(8):4246.

9.- Hombre de 30 aos de edad. Refiere frecuente irritacin conjuntival, comezn en el borde palpebral, aparicin habitual de orzuelos y enrojecimineto crnico de la conjuntiva. A la exploracin se observa, entre las pestaas y al tallarlas, descamacin en forma de caspa, escasa secrecin en los ngulos e hiperemia tarsal y conjuntival bulbar leves. El diagnstico ms probable es:

a) b) c) d)

Conjuntivitis bacteriana Conjuntivitis viral Blefaroconjuntivitis Conjuntivitis primaveral

Las blefaroconjuntivitis (blefaritis) son una de las patologas ms frecuentemente encontradas en la poblacin general. Consisten en infecciones crnicas de las glndulas sebceas del borde libre del prpado, pudiendo ser: Ant er ior es, que pr esent an escamas y anillos quer t icos alr ededor de las pest aas y generalmente son estafiloccicas. Post er ior es, o meibomit is, son secundar ias a la inf eccin de las glndulas de Meibomio; pueden ser tratadas con algn antibitico tpico y/o sistmico y en ocasiones algn esteroide tpico, pero lo ms importante en todos los casos son las medidas generales de aseo. PAC de Oftalmologa Parte B libro 4

10.- Femenino de 78 aos de edad con antecedente de insuficiencia cardiaca congestiva presenta angina de pecho. Sus medicamentos se ajustan con: furosemida, digoxina, nitroglicerina y potasio complementario. Poco despus presenta cefaleas pulstiles intermitentes que incapacitan a la paciente. Cul de las siguientes medidas debe iniciarse para su control: a) Suspender la nitroglicerina b) Comenzar la administracin de propranolol c) Comenzar la administracin de ergotamina sublingual d) Realizar biopsia de arteria temporal

Allen R. M. MMS Medicina Interna. 5. Edicin. National Medical Series. Mc. Graw Hill. 2006. (captulo 11IV B 1 a (2), d (1) (a), 4 a-b). La nitroglicerina puede causar cefaleas "vasculares" terebrantes; por tanto, la ms sencilla opcin de tratamiento es suspender el preparado de nitroglicerina y utilizar, si es posible, un medicamento cardaco alternativo. Siempre debe considerarse la arteritis temporal como posible causa de cefalea en pacientes mayores de 50 aos. Favorecen este diagnstico el aumento de la velocidad de eritrosedimentacin; la claudicacin mandibular; las artralgias y las mialgias; y una arteria temporal hipersensible e indurada. El propranolol es un antimigraoso eficaz. Sin embargo, antes de prescribir estos frmacos deben descartarse los posibles factores que la precipitan. La ergotamina es un tratamiento abortivo eficaz para migraa, pero es un vasoconstrictor y no debe usarse en pacientes con angina de pecho. En la valoracin de ancianos con cefalea de inicio reciente debe considerarse realizar una tomografa computadorizada del cerebro. En este caso, si es posible suprimir la cefalea al suspender la nitroglicerina, no es necesaria una CT.

11.- Hombre de 36 aos con presencia de desarrollo de manchas lenticulares de color caf, rosado y blanco, levemente descamativas, se diagnostica micosis crnica asintomtica por Malassezia furfur. La localizacin anatmica ms frecuente de esta patologa es:

a) b) c) d)

Dorso de manos y pies. Codos y rodillas Cara y cuello. Pecho y espalda.

La pitiriasis versicolor (PV) es una micosis superficial crnica, caracterizada por mculas hipo o hiperpigmentadas levemente descamativas. El trmino versicolor se refiere al color variable de las lesiones que pueden ser caf, rosadas o blancas. Esta enfermedad tiene distribucin mundial y es uno de los desrdenes ms comunes en la pigmentacin de la piel. La mayor prevalencia y el mayor porcentaje de recurrencias se observan en reas clidas y hmedas. Eichstedt, en 1846, fue el primero en reconocer la naturaleza fngica de la PV, al describir un hongo asociado a esta afeccin. Sin embargo, el gnero Malassezia, con Malassezia fufur (M. furfur) como especie tipo, fue creado por Baillon medio siglo despus, en 1889, en honor a Luis Malassez. Pitiriasis versicolor, a veces denominada Tinea versicolor, es uno de los trastornos de pigmentacin ms comunes en el mundo. Es una micosis crnica, leve y usualmente asintomtica. Se produce por el crecimiento de la levadura lipoflica Malassezia furfur en la capa ms superficial de la piel o estrato crneo.

La infeccin se caracteriza por el desarrollo de manchas o mculas lenticulares de color caf, rosado o blanco, levemente descamativas, en tronco y brazos.

BIBLIOGRAFA 1. Ashbee HR, Evans E.G. Inmunology of Diseases Associated with Malassezia species. Clin Microbiol Rew. 2002;21-57. 2. Crespo Erchiga V, Delgado Florencio V. Malassezia species in skin diseases. Curr Opinin Infect Dis, 2002, 15:133-142.

3. Guho E, Boekhout T, Ashbee HR, Guillot J, Van Belckum A, Faergeman J. The role of Malassezia species in the ecology of human skin and as pathogen. Med Mycol 1998;36 (supp1):220-229. 4. Crespo Erchiga V, Ojeda Martos A, Vera Casao A, Crespo Erchiga A, Snchez Fajardo F, Guho E. Mycology of pityriasis versicolor. J. Mycol. Med. 1999; 9:143-148. 5. Katoh T, Irimajiri J. Pityriasis versicolor and Malassezia folliculitis. Nippon Ishinkin Gakkai Zasshi. 1999;40:69-71. 6. Gupta AK, Batra R, Bluhm R, Boekhout T, Dawson T. Skin diseases associated with Malassezia species. J Am Acad Dermatol 2004; 51:785-798.

12.- A 7 year old girl who presented skin lesions such macules and papules that evolve into blisters within hours. The mother states that introduced upper respiratory infection 2 weeks ago. The diagnosis you do it? a) b) c) d) Herpes Zoster. Rubella Scarlet fever Varicella

Varicela Manifestaciones clnicas: la infeccin primaria produce fiebre moderada, y un exantema vesicular genereralizado y pruriginoso. Las lesiones se observan en distintos estados (mcula, ppula, vescula y costras) y duran alrededor de una semana. Es posible tambin apreciar vesculas o lceras en la mucosa oral. La reactivacin del virus es responsable del herpes zoster. Agente etiolgico: virus varicella zoster (ADN) perteneciente a la familia Herpesviridae Epidemiologa: los humanos son la nica fuente de contagio para este agente. La transmisin es a travs del contacto persona a persona y por la ruta respiratoria. Los brotes aparecen hacia fines del invierno y en primavera. El perodo de incubacin vara entre 10 a 21 das, y usualmente es de dos semanas. El momento de mayor contagiosidad ocurre desde dos das antes de que el exantema aparezca, hasta que las lesiones se encuentren en etapa de costra. En nios inmunodeprimidos, los perodos de incubacin, de contagiosidad y de erupcin pueden ser ms largos.

Diagnstico: el diagnstico es fundamentalmente clnico; sin embargo, existen situaciones de diagnstico diferencial de lesiones vesiculares en el husped inmunodeprimido, en que la

inmunofluorescencia directa permite hacer el diagnstico rpido de infeccin por virus varicela y as, tomar decisiones teraputicas precoces. Aislamiento del paciente hospitalizado: Aislamiento respiratorio y de contacto por al menos cinco das desde que comienza el exantema o hasta que todas las vesculas estn en estado de costra. Tratamiento: en el husped inmunocompetente el manejo del paciente con infeccin aguda es sintomtico. Si se requiere uso de antipirticos se recomienda usar slo paracetamol, por la posible riesgo de desarrollo del Sndrome de Reye con el uso de cido acetilsaliclico. Debe prevenirse la sobreinfeccin bacteriana de las lesiones, evitando el prurito y el grataje, manteniendo las uas cortas, bao diario y usando antihistamnicos. El uso de aciclovir est indicado en sujetos con riesgo de desarrollar complicaciones (inmunodeprimidos, adolescentes, adultos, pacientes con terapia crnica con saliclicos y esteroides) y debe iniciarse en las primeras 24 hrs del exantema. El uso de aciclovir puede tambin considerarse en el segundo caso intrafamiliar ya que en estos pacientes se ha observado una evolucin ms severa de la enfermedad.

Bibliografa
Report of the Committee on Infectious Diseases, 25h edition, Red Book 2000. American Academy of Pediatrics. Fifth (human parvovirus) and sixth (herpesvirus 6) diseases. Koch WC. Curr Opin Infect Dis 2001, Jun; 14 (3): 343-356. Primary human herpesvirus 8 in immunocompetent children. Andreoni M, Sarmati L, Nicastri E, El Sawaf G, El Zalabani M, Uccella I, et al. JAMA 2002 Mar 13; 287 (10): 1295-300.

Varicella vaccine update. AAP. Pediatrics 2000, Jan 105: 136-141.

13.- Se trata de preescolar de 5 aos de edad, con antecedente de neumona por Pneumocystis c., la profilaxis de sta patologa en ste paciente, se encuentra indicada cuando las cifras de linfocitos ajustados son de: a) b) c) d) < 1500 Clulas/l < 500 Clulas/l < 750 Clulas/l < 200 Clulas/l

La NPC en nios y en el embarazo La NPC es infeccin diagnstica de SIDA en un alto porcentaje de nios, sobre todo en el primer ao de vida. Los nios menores de 1 ao con CD4+ por debajo de 1500//l t ienen un 90% de riesgo de padecerla. Las manifestaciones clnicas, el diagnstico y el tratamiento no difieren de los del adulto. Para la prevencin deben seguirse las siguientes recomendaciones: Se aconseja la profilaxis primaria en todos los nios menores de 1 ao hijos de madre VIH+ La profilaxis debera comenzar a las 4-6 semanas de vida y suspenderse en caso de que posteriormente no resulten estar infectados. Los nios infectados y aquellos cuyo estado de infeccin por VIH se desconozca debern seguir recibiendo tratamiento profilctico durante el primer ao de vida. En los mayores de esta edad se considerar continuar con la profilaxis atendiendo al recuento de linfocitos CD4+ segn la edad: de 1 a 5 aos, cuando el recuento sea < 500 clulas/l o el por cent aj e < 15%; en nios de 6 a 12 aos, cuando el r ecuent o sea inferior a 200 clulas/ l o el por cent aj e < 15%. No se ha estudiado la seguridad de interrumpir la profilaxis en nios infectados con el VIH y sometidos a tratamiento antirretroviral. Los nios con un historial que incluya episodios de NPC debern ser tratados de por vida con quimioprofilaxis para evitar casos de recurrencia.

NCP en el embarazo Como ya se ha dicho, la quimioprofilaxis anti-NPC debe administrarse a las mujeres embarazadas al igual que al resto de adultos y adolescentes. En este caso, el agente profilctico recomendado sigue siendo el cotrimoxazol, con dapsona como alternativa. Debido a la posibilidad terica de una posible teratogenicidad asociada a la exposicin a los frmacos durante el primer trimestre de gestacin, durante dicho perodo puede considerarse la alternativa de pentamidina en aerosol debido a que dicho agente no se absorbe sistmicamente por lo que el feto en desarrollo no sufre exposicin al frmaco.

Lectura recomendada: Neumona por Pneumocystis carinii en nios infectados por el virus de inmunodeficiencia humana (VIH). Gac Med Mex 2004; 140 (1): 59-70

14.- Femenino de 20 aos la cual refiere que desde hace varios meses presenta astenia, cansancio, prdida de apetito y dificultades para concentrarse en los estudios. Al interrogatorio refiere que a perdido inters en los estudios, frecuenta menos a sus amigos, con pesimismo en la mayora de sus actividades. Cul es el diagnstico ms probable?

a) Anorexia nerviosa b) Trastorno de ansiedad. c) Distimia. d) Depresin mayor

LA DISTIMIA La distimia es un estado de nimo crnicamente deprimido, menos grave que la depresin y que no cumple los criterios para una depresin mayor, o lo hace slo en perodos muy cortos. Su evolucin suele ser de ms de dos aos. Se caracteriza por un abatimiento prolongado del estado de nimo en que el sujeto distmico se describe a s mismo como triste o desanimado, perdiendo el inters por las cosas y vindose a menudo como intil y poco interesante. Posee sntomas persistentes o intermitentes, de intensidad ms leve comparacin a la depresin mayor. Aiskal (1983) la define como mal humor y se caracteriza porque el individuo est habitualmente triste, introvertido, melanclico, excesivamente consciente, incapaz de alegra y preocupado por su insuficiencia personal. Los criterios de diagnstico de Distimia son los que a continuacin se detallan: A. Estado de nimo crnicamente depresivo la mayor parte del da de la mayora de los das, manifestado por el sujeto u observado por los dems, durante al menos 2 aos. Nota:

En los nios y adolescentes el estado de nimo puede ser irritable y la duracin debe ser de al menos 1 ao.

B. Presencia, mientras est deprimido, de dos (o ms) de los siguientes sntomas: 1. 2. 3. 4. 5. 6. Prdida o aumento de apetito Insomnio o hipersomnia Falta de energa o fatiga Baja autoestima Dificultades para concentrarse o para tomar decisiones Sentimientos de desesperanza

C. Durante el perodo de 2 aos (1 ao en nios y adolescentes) de la alteracin, el sujeto no ha estado sin sntomas de los Criterios A y B durante ms de 2 meses seguidos. D. No ha habido ningn episodio depresivo mayor durante los primeros 2 aos de la alteracin (1 ao para nios y adolescentes); por ejemplo, la alteracin no se explica mejor por la presencia de un trastorno depresivo mayor crnico o un trastorno depresivo mayor, en remisin parcial. Nota: Antes de la aparicin del episodio distmico pudo haber un episodio depresivo mayor previo que ha remitido totalmente (ningn signo o sntoma significativos durante 2 meses). Adems, tras los primeros 2 aos (1 ao en nios y adolescentes) de trastorno distmico, puede haber episodios de trastorno depresivo mayor superpuestos, en cuyo caso cabe realizar ambos diagnsticos si se cumplen los criterios para un episodio depresivo mayor. E. Nunca ha habido un episodio manaco, un episodio mixto o un episodio hipomanaco y nunca se han cumplido los criterios para el trastorno ciclotmco. F. La alteracin no aparece exclusivamente en el transcurso de un trastorno psictico crnico, como son la esquizofrenia o el trastorno delirante. G. Los sntomas no son debidos a los efectos fisiolgicos directos de una sustancia (p. ej., una droga, un medicamento) o a enfermedad mdica (p. ej., hipotiroidismo). H. Los sntomas causan un malestar clnicamente significativo o deterioro social, laboral o de otras reas importantes de la actividad del individuo. Como vemos, la Distimia presenta sntomas ms o menos similares a la Depresin mayor, pero se diferencian entre s respecto a: Tipo de evolucin: el estado de nimo depresivo es crnico (no presenta intervalos libres de sntomas o mejoras significativas) y dura por lo menos 2 aos. Severidad de los sntomas: los mismos suelen ser leves o moderados, sin una alteracin significativa de las relaciones familiares, sociales y laborales del individuo.

Referencias Bibliogrficas
- Diagnostic and Statistical Manual of Mental Disorders, 4 edition (DSM-IV TR). American Psychiatric Association. American Psychiatric Press, 2000. - Kaplan and Sadock's Synopsis of Psychiatry, 9 edition. Lippincott Williams & Wilkins Press, 2003.

15.- Se trata de femenino de 54 aos con datos clnicos de trombosis venosa profunda Cul de los siguientes procedimientos es la primera prueba que debe solicitar el mdico para establecer el diagnstico? a) b) c) d) Pletismografa de impedancia Cateterismo cardaco Venografa con contraste Estudios de ventilacin y perfusin pulmonares

La pletismografa de las extremidades inferiores es una prueba incruenta til para establecer el diagnstico de trombosis venosa profunda. La venografa con medio de contraste proporciona el diagnstico definitivo casi en cualquier caso; sin embargo, esta prueba cruenta puede, de hecho, causar tromboflebitis en una minora de los casos. El cateterismo cardaco, el gammagrama pulmonar y la tomografa computadorizada (CT) no son tiles para diagnosticar trombosis venosa profunda.

Allen R. M. MMS Medicina Interna. 5. Edicin. National Medical Series. Mc. Graw Hill. 2006. (captulo 1 VIIIA 4 a-b),

16.- Masculino de 30 aos de edad adicto a las drogas intravenosas presenta debilidad del hemicuerpo derecho y cefalea en un periodo de dos das. La exploracin revela un individuo mal nutrido y afebril con hemiparesia derecha leve. Cul de los siguientes es el diagnstico ms probable?

a) b) c) d)

Endocarditis bacteriana Absceso cerebral Meningitis por virus de la inmunodeficiencia humana (VIH) Meningitis criptoccica

Los adictos a drogas intravenosas estn propensos a sufrir bacteriemia, que a su vez puede producir absceso cerebral y disfuncin neurolgica progresiva. Los pacientes por lo comn se encuentran afebriles a no ser que haya endocarditis acompaante u otro origen endovascular de infeccin. Los adictos a drogas intravenosas tienden a presentar endocarditis bacteriana y pueden cursar con dficit neurolgicos en forma apopltica debido a embolia sptica del cerebro. Sin embargo, por lo regular tienen fiebre. La meningitis por VIH produce cefalea y datos de irritacin menngea, pero no se presenta dficit neurolgico focal. La meningitis criptoccica se manifiesta con conducta alterada y cefalea y los pacientes estn afebriles. Sin embargo, es raro que haya datos de seudoapopleja. Por ltimo, el uso de drogas intravenosas puede provocar embolia de cuerpo extrao pero con problemas neurolgicos apoplticos. Un mbolo puede llegar al cerebro por un cortocircuito cardaco de derecha a izquierda o bien por una malformacin arteriovenosa pulmonar si la inyeccin es venosa. El mbolo puede entrar a la circulacin cerebral de manera directa en caso de inyeccin intracarotdea.

Referencias Nath A. Brain abscess and parameningeal infections. In: Goldman L, Ausiello D, eds. Cecil Medicine. 23rd ed. Philadelphia, Pa: Saunders Elsevier; 2007: chap 438 Allen R. M. MMS Medicina Interna. 5. Edicin. National Medical Series. Mc. Graw Hill. 2006. (Captulo 11 XVI B 1-2).

17.- Mujer de 60 aos con antecedentes de cuadro respiratorio alto hace 3 semanas. Desde hace dos presenta vrtigo de inicio sbito intenso y persistente, con varias horas de duracin e inestabilidad a la marcha, no refiere tinitus ni hipoacusia. El diagnostico clnico ms probable es: a) b) c) d) Mastoiditis. Otoesclerosis. Neuronitis vestibular. Otitis media.

Neuronitis Vestibular (NV) Es una causa relativamente frecuente de SV. Hay compromiso de etiologa viral del nervio vestibular, a nivel del ganglio de Scarpa (ganglio vestibular). Se ve preferentemente en adultos (14). Cuadro clnico: inicio sbito de vrtigo intenso y persistente, acompaado de nuseas y vmitos e inestabilidad en la marcha (15). Sin tinnitus ni hipoacusia. Son crisis de das o semanas de duracin, que se presentan espordicamente en un lapso de hasta 3 aos desde el inicio de los sntomas (16). Fisiopatologa: se postula que la NV se produce en respuesta a una infeccin viral o a cambios inflamatorios posteriores en el ganglio de Scarpa. La NV no tiene correlacin o base histopatolgica conocida.

Diagnstico: se basa principalmente en la clnica, ya que no existe un examen especfico. En el examen de VIII par lo nico positivo suele ser la presencia de desequilibrio en la marcha, paresia vestibular unilateral y excepcionalmente VPPB asociado (17,18). Hay que tener presente que algunos tumores del ngulo ponto-cerebeloso puede manifestarse como cuadros de neuronitis vestibular de difcil manejo (19,20). Tratamiento: se debe indicar reposo con licencia mdica, ya que las cadas y traumatismos que pueden sufrir los pacientes constituyen complicaciones generalmente peores que la enfermedad causal. Se utilizan sedantes vestibulares, tanto en el episodio inicial como en las reagudizaciones posteriores (21). Algunas alternativas de tratamiento farmacolgico: difenidol (Vontrol), 25 mg vo cada 8 hrs. cinarizina, 25 mg vo cada 12 hrs. Tambin puede ser necesaria la rehabilitacin vestibular a largo plazo. Sndromes Vestibulares Perifricos: Segunda parte Diagnstico Diferencial y Etiologas J. Ignacio Mndez 1, Hctor Riveros 2, M. Jos Concha

18.- Se trata de femenino de 50 aos con antecedente: Dx. De sndrome anmico sin causa aparente. Exmenes de Laboratorio Reportan: Hb de 8,5 g/dl, VCM de 75 fl, y HCM de 25 pg. El examen de laboratorio que conforma el origen del diagnstico de esta paciente es la determinacin de: a) Sideremia. b) Ferritina. c) Saturacin de la transferrina. d) Hemoglobina A2.

La anemia por dficit de hierro es un proceso que aparece de forma rpida o de forma ms bien lenta? La anemia por falta de hierro se va estableciendo de forma gradual, cuando el hierro que se pierde supera al hierro aportado por la dieta; lo primero que ocurre es que el organismo empieza a utilizar las reservas de hierro. En esta etapa la ferritina empieza a descender, pero el hierro srico y la TIBC generalmente permanecen inalterados no establecindose todava anemia. En el momento en que el dficit de hierro empeora, el hierro srico disminuye mientras que la transferrina y la TIBC aumentan, empezando las clulas de la serie roja (hemates) a palidecer (hipocromia) y a disminuir de volumen (microcitosis), existiendo todava un nmero suficiente de clulas de la serie roja. Si la deficiencia de hierro contina o empeora aparece entonces la anemia. Mourey L. Manual de procedimientos del Laboratorio Clnico. Mxico:

IMSS, 158. ltima edicin.

19.- Se trata de masculino de 58 aos de edad con diagnostico de HPB, se niega a tomar tratamiento farmacolgico y elige una RTUP, la complicacin ms frecuente de este procedimiento es: a) b) c) d) Contractura de cuello vesical Impotencia Incontinencia Eyaculacin retrgrada

Reseccin Transuretral (RTU): es el gold standard de las intervenciones. __ En general es una ciruja corta. __ 90% mejora importante de los sntomas. __ Morbimortalidad baja del 0,2%. __ El 20 % debe reoperarse a los 10 aos aproximadamente. __ Complicaciones: impotencia 10%, eyaculacin retrograda 50%, Incontinencia 4%.

BIBLIOGRAFIA: 1. Medina JJ, Parra RO, Moore RG. Benign prostatic hyperplasia (the aging prostate). Med Clin North Am 1999 Sep;83(5):1213-29 2. Oesterling JE. Benign prostatic hyperplasia. Medical and minimally invasive treatment options. N Engl J Med 1995 Jan 12;332(2):99-109

20.- Se reportaron 45 casos de varicela entre los alumnos de primer grado de una escuela secundaria durante la segunda quincena del mes de agosto de 2012. La tasa de ataque es de 26.4%. Esta situacin nos indica que estamos ante un(a): a) b) c) d) Epidemia Endemia Brote Pandemia

De acuerdo a la NOM- 017 un brote se define como la ocurrencia de dos o ms casos asociados epidemiolgicamente (tiempo, lugar y persona) entre s. La medida cuantitativa de la extensin de un brote es la Tasa de Ataque (TA) que se calcula dividiendo el nmero de casos nuevos entre el total de personas expuestas por 100.

Norma Oficial Mexicana NOM-017-SSA2 -1994, Para la vigilancia epidemiolgica. Apartados 3.1.6 y 3.1.78.1

21.- En consulta externa de su hospital, recibe usted los resultados histopatolgicos de una paciente de 22 aos de edad que acudi a revisin rutinaria, los resultados reportan imagen histolgica de coilocitos lo cual sugiere infeccin por:

a) b) c) d)

Herpes virus tipo 2 Citomegalovirus Vaginosis bacteriana. Virus del papiloma humano

Clulas del epitelio cervical con un halo perinuclear tpico secundario a la infeccin por el virus del papiloma humano ( VPH ). Su presencia determina el diagnstico de lesin intraepitelial de bajo grado ( LSIL ) en la patologa cervical e indica la infeccin de las clulas por el VPH. Para que una clula sea considerada como coiloctica, debe combinar la atipa nuclear con la presencia de vacuola perinuclear, ya que esta ltima caracterstica puede observarse en procesos no relacionados al HPV como ser infecciones por Trichomonas o Cndidas, sin asociacin a atipas nucleares. El coilocito es la manifestacin clsica de la infeccin por VPH en la clula. Fue descrito por primera vez por Koss y Durfee en 1956. Esta clula tambin ha sido llamada clula en baln. El coilocito es una clula epitelial escamosa, ms comnmente superficial e intermedia, aunque tambin puede verse en clulas parabasales y metaplsicas. Esta clula presenta cambios tpicos tanto en su ncleo como en su citoplasma, pierde los bordes angulados usuales de la clula escamosa superficial y su forma tiende a ser redondeada y ovoide. El citoplasma muestra una condensacin perifrica que le da un aspecto en asa de alambre, es opaco, denso y de aspecto creo, anfoflico, acidoflico o de color rojo/naranja brillante. Adems se observa una gran cavidad o halo con un margen muy bien definido, de forma oval o ligeramente festoneado. El ncleo de la clula se localiza de manera excntrica, lo que lo convierte en un halo paranuclear, no perinuclear. Ocasionalmente puede encontrarse material fagocitado dentro del espacio coiloctico.

Lesin Intraepitelial de Bajo Grado. Alteraciones Celulares compatibles con Infeccin (Coilocitos). Schlecht, N.F., Kulaga, S., Robitaille, J., Ferreira, S., Santos, M., Miyamura, R.A., Duarte-Franco, E., Rohan, T.E., Ferenczy, A., Villa, L.L., & Franco, E.L. (2002) Persistent Human Papillomavirus Infection as a Predictor of Cervical Intraepithelial Neoplasia. JAMA, 286, 3106-3114

22.- Mujer de 32 aos, con un cuadro clnico de sangrado menstrual irregular, dismenorrea, dispareunia y una esterilidad de 3 aos de evolucin, con resultados de laboratorio hormonal normal y un estudio ecogrfico transvaginal que informa de un tero normal y sendas formaciones qustica ovricas bilaterales de 4 cms. La orientacin diagnstica es: a) Hemorragia uterina disfuncional. b) Sndrome del ovario poliqustico. c) Endometriosis. d) Quistes dermoides bilaterales. Causas y sntomas de endometriosis: Las causas de la endometriosis an no se conocen. Las clulas del revestimiento interno del tero de alguna manera se desplazan hasta zonas externas al mismo y siguen creciendo. Este desplazamiento podra quizs deberse a que pequeos fragmentos del revestimiento uterino, desprendidos durante la menstruacin, retrocedan hacia las trompas de Falopio en direccin a los ovarios hasta entrar en la cavidad abdominal, en lugar de salir con el flujo menstrual a travs de la vagina. La endometriosis causa dolor en la parte inferior del abdomen y la zona plvica, irregularidades menstruales (como manchar antes de la menstruacin) e infertilidad.

Algunas mujeres con endometriosis grave no presentan sntomas, mientras que otras con la enfermedad en grado mnimo sufren un dolor invalidante. Con frecuencia, el dolor menstrual debido a la endometriosis no aparece hasta aos despus de desarrollar la enfermedad. En algunos casos, se constata dolor durante el coito (dispareunia), antes o durante la menstruacin. El tejido endometrial adherido al intestino grueso o a la vejiga urinaria puede provocar hinchazn abdominal, dolor durante las deposiciones, hemorragia rectal durante la menstruacin o dolor en la parte inferior del abdomen durante la miccin. As mismo, cuando el tejido se localiza en un ovario o una estructura cercana puede dar lugar a la formacin de una masa llena de sangre (endometrioma). En ocasiones, el endometrioma se rompe bruscamente o se escapa algo de su contenido, lo que causa un agudo y repentino dolor abdominal.

23.- Se trata de masculino de 25 das de nacido al cual se desprendi el cordn umbilical a los 9 das y cuyo ombligo no ha cicatrizado. Presenta en este nivel una tumoracin roja, prominente, circular y que segrega un contenido alcalino. Refiere la madre que a veces nota en esta tumoracin ruido de gases. El diagnstico ms probable es?

a) b) c) d)

Persistencia del conducto onfalomesentrico. Fstula vsico-umbilical. Granuloma umbilical. Onfalocele.

Durante la vida fetal el conducto onfalomesentrico une al saco vitelino con el intestino medio y se cierra normalmente para desaparecer por completo. Se encuentra conectado con el intestino primitivo en el saco amnitico. En el desarrollo embriolgico normal, el conducto onfalomesentrico involuciona entre las 5a y 7a semanas de vida intrauterina. Un fracaso en la regresin produce varias anormalidades, en dependencia del lugar donde se localice este fallo: en el lado umbilical o en el intestinal. Los vestigios del conducto onfalomesentrico (vitelino) pueden presentarse como anomalas relacionadas con la pared abdominal. Sin embargo, puede ocurrir que todo o parte del conducto fetal se mantenga y entonces se produzca sintomatologa clnica. Tambin puede persistir como una estructura permeable en toda su longitud o mantenerse como un divertculo o quiste cuando persiste en su periferia, parte central o media; o quizs quede representado simplemente por un resto de epitelio intestinal ectpico a nivel umbilical o como cordn fibroso. La explicacin embriolgica de la persistencia del conducto onfalomesentrico puede explicarse diciendo que el conducto onfaloentrico o vitelino que representa una comunicacin entre el vrtice del asa umbilical fetal y el saco vitelino se oblitera y desaparece junto con sus vasos acompaantes arteria y vena onfalomesentricas hacanla

sptima semana-fetal. Sin embargo, cuando este proceso no ocurre, da lugar a la patologa conocida de las estructuras remanentes del conducto onfalomesentrico. Despus de los cuidados iniciales en la edad del recin nacido que se han dado al cordn umbilical y una vez que ste se ha desprendido, esta zona recibe poca atencin en los das subsiguientes; sin embargo, puede ser asiento de muchas lesiones, tanto congnitas como adquiridas, con lascuales el mdico debe estar familiarizado para poder diagnosticarlas. El diagnstico se hace mediante la observacin, cuando el recin nacido realiza esfuerzos como el provocado por el llanto, pues se produce la salida de material fecal o gases por el ombligo.

REFERENCIAS: 1. Pomeranz A. Anomalies, abnormalities, and care of the umbilicus. Pediatr Clin N Am 51 (2004) 819. 2. Rowe M, Chapter 48: Disorders of the umbilicus. Essentials of Pediatric Surgery, 1995. 3. Care of the umbilical cord, a review of the evidence. OMS. 1998. 4. Cushing A. Omphalitis: a review. Pediatr Infect Dis J 1985, 4(3); 282-285 5. Janssen P. To dye or not to Dye: A randomized, Clinical Trial of a Triple Dye/alcohol Regime Versus Dry cord care. Pediatrics 2003; 111, 15-20. 6. Zupan J. Topical umbilical cord care at birth The Cochrane Database of Systematic Reviews, 2004. Issue 3. 7. Sawardekar K. Changing spectrum of neonatal omphalitis. Pediatr Infect Dis J, 2004;23:22-6 8. Soto G. Anomalas uracales: un diagnstico rara vez planteado. Pediatra al da 2001;17(5) 331) 9. Cilento BG Jr. Urachal anomalies: defining the best diagnostic modality. Urology 1998, 52 (1) 120-2. 10. Ossandn F. Malformaciones congnitas: patologa umbilical. Pediatra al da 1995;11(2) 11. Lotan G. Double ligature: A treatment for pedunculated umbilical granulomas in children. Am Fam Physician 2002;65:2067-8 12. Novoa A. El pediatra ante un lactante con cada tarda del cordn umbilical. Arch arg Pediatr 2004; 102(3).

24.- Se trata de preescolar de 4 aos de edad presenta un peso y talla en el percentil 3, se refiere por presentar anorexia, irritabilidad, apata al juego, se le observa con palidez, el laboratorio reporta una hemoglobina de 7.3g/dl, hierro srico de 36mcg/dl y saturacin de transferan de 12%, El tipo de anemia ms probable es?

a) b) c) d)

Hemoltica Perniciosa Ferropnica Drepanoctica

Es el tipo de anemia ms frecuente en la poblacin general. Se trata de una anemia caracterizada por la disminucin o ausencia de los depsitos de hierro, baja concentracin de hierro srico, baja saturacin de transferrina y baja concentracin de la Hb o del hematocrito. La ferropenia se manifiesta con un sndrome caracterstico: SNC: irritabilidad, labilidad emocional, disminucin de la concentracin y la memoria, cefalea, ataxia, parestesias, trastornos del sueo y la alimentacin (PICA: apetencia desmedida por sustancias poco habituales. Es muy caracterstica la pica de hielo o pagofagia), aumento de la sensibilidad al fro, sndrome de piernas inquietas

Piel y mucosas: coloracin azulada de la esclertica, fragilidad de uas y cabello, coiloniquia o aspecto cncavo de las uas, glositis, queilitis angular o rgades, atrofia gstrica, en casos graves produccin de membranas esofgicas (sndrome de Plummer-Vinso). Etiologa: La anemia ferropnica puede deberse a: Aumento de la utilizacin: gestacin, lactancia, crecimiento corporal rpido en la infancia y adolescencia. Prdidas fisiolgicas: menstruacin Prdidas patolgicas: Hemorragia digestiva. El sangrado digestivo crnico es la causa ms frecuente en este grupo: hemorragias gstricas por medicamentos (AAS, AINES, corticoides asociados a AINES), hernia hiatal, diverticulosis, hemorroides. En pacientes mayores de 60 aos debe considerarse la posibilidad de neoplasia, sobre todo de colon y la angiodisplasia intestinal. Genito-urinarias Aparato respiratorio Hemlisis intravascular Alteraciones en la absorcin: Dietas insuficientes: Debe interrogarse siempre acerca de los hbitos dietticos. El hierro presente en alimentos de origen vegetal como cereales y legumbres tiene baja biodisponibilidad (algunos alimentos como las lentejas tienen excesiva fama de riqueza en hierro). Incluso en carnes y pescados el

contenido en Fe es relativamente bajo. En gestantes, nios, mujeres en edad frtil, la dieta puede ser un factor ostmenopusi o causal de deficiencia de hierro. En mujeres ostmenopusicas y en el varn la dieta sera causa excepcional de ferropenia. Absorcin defectuosa: gastrectomas parciales o totales, enfermedad inflamatoria intestinal, enfermedad celaca (causa relativamente frecuente de anemia ferropnica); en algunos casos podra estar indicada la deteccin de anticuerpos antiendomisio y antigliadina para descartar una enfermedad celaca no diagnosticada. Diagnstico: El nivel de ferritina srico bajo es indicativo de una situacin de deplecin de hierro y es la prueba definitiva de la existencia de anemia por dficit de hierro (AF). Los dems parmetros no permiten diferenciar con seguridad la AF de la anemia por enfermedad crnica. Generalmente aparece microcitosis, aunque hasta en el 30% de las anemias ferropnicas, en algunas poblaciones, el VCM puede ser >80; asimismo en el 10% de las microcitosis los parmetros relacionados con el hierro son normales. Otros datos a tener en cuenta: El frotis perifrico habitualmente muestra anisocitosis y poiquilocitosis. Cuando la microcitosis se asocia a aumento de glbulos rojos, sobre todo si es mantenida, pensar en talasemia. Puede haber trombocitosis reactiva.

Bibliografa 1. Abramson S, Abramson N. Common' Uncommon Anemias. Am Fam Physc 1999; 59 (4). 851-8 2. Adamson JW. Anemia and Polycytemia. En: Kaasper DL, Braunwald E (Edit.). Harrisons principles of internal medicine. 16th. ed. Nwe Cork: McGraw-Hill; 2005. p.329-336 3. Hernndez Nieto L, Hernndez Garca MT, Junc Piera J, Vives-Corrons JL, MartnVega C. Enfermedades del sistema eritrocitario: anemias. En: Farreras Valent P, Rozman C. (Dir.). Medicina Interna. Barcelona: Elsevier; 2004. p. 1644-1669 4. Glader B. Anemia: General Considerations. En: Wintrobes Clinical Hematology. 11th. Philadelphia: Lippincott Williams & Wilkins; 2004. p. 947-978.

25.- Femenino de 28 aos. Acude a consulta por hirsutismo e irregularidades menstruales. Antecedente familiar de DM tipo 2 en madre y una hermana. Su padecimiento empez desde la adolescencia con ritmo menstrual de 40 60 x 5 -7 das. EF: peso 87 kg, estatura 1.54 m, TA 130/80, FC 72x, acantosis nigricans en cuello y axilas, no galactorrea, permetro abdominal de 112 cm. Vello supralabial, en patilla, en lnea media infraumbilical y en cara interna de muslos. Laboratorio: glucosa 104 mg/dL, resto de qumica sangunea normal, Bh normal. Prolactina 16 ng/dL, testosterona: 78 ng/dL (25 a 50 ng/dl).

El diagnstico probable es:

a) b) c) d)

Sndrome de ovarios poliqusticos por resistencia a la insulina Hiperprolactinemia por microprolactinoma Virilizacin por tumor productor de testosterona Diabetes mellitus secundaria a la obesidad

El sndrome de ovarios poliqusticos (SOPQ) afecta aproximadamente a un 4% de mujeres en edad reproductiva y se caracteriza por anovulacin crnica e hiperandrogenismo. Es la causa ms comn de infertilidad en mujeres. Se caracteriza clnicamente por acn, alopecia, hirsutismo, irregularidades menstruales e infertilidad. Los hallazgos de laboratorio ms frecuentes son: aumento de la hormona luteinizante (LH), aumento de la relacin LH/FSH (hormona folculoestimulante), aumento de andrgenos (tanto ovricos como adrenales) y de estrgenos circulantes. Otros hallazgos de laboratorio habituales son una prueba tolerancia oral a la glucosa anormal y alteraciones en el perfil lipdico. Todo esto junto con las imgenes ecocardiogrficas caractersticas definen al sndrome. La teraputica permite dos grandes enfoques que pueden superponerse: la correccin de las manifestaciones de hiperandrogenismo y el tratamiento de las alteraciones del eje reproductivo (anovulacin, esterilidad). Los antiandrgenos estn fundamentalmente indicados para tratar los sntomas virilizantes. Las alternativas para inducir la ovulacin son numerosas: al citrato de clomifeno y a la antigua reseccin en cua se agregan las gonadotrofinas humanas, pulsos de GnRH (hormona liberadora de gonadotrofinas), medidas o frmacos para modificar los niveles de insulina, y finalmente tcnicas quirrgicas endoscpicas para reducir la masa ovrica.

Revista de Posgrado de la VIa Ctedra de Medicina - N 125 Marzo 2003 Pg. 37-40 SINDROME DE OVARIOS POLIQUISTICOS Dra. Sandra Beneyto, Dra. Mara Andrea Ferreyra, Dr. Andrs Galfrascoli, Dr. Andrs Gonzlez, Dra. Susana Sosa

26.- Se trata de femenino de 22 aos que acude a consulta por prdida de peso, diarrea, palpitaciones, temblor y labilidad emocional. No tiene antecedentes importantes. Refiere evacuaciones diarreicas de 5 a 7 por da, sin moco o sangre. EF: peso 52 kg, estatura 1.59, FC 108x, TA: 120/65, retraccin palpebral, hiperemia conjuntival y de carncula, cuello con tiroides aumentada de tamao 3 veces de forma difusa, extremidades superiores con hiperhidrosis palmar, temblor fino distal, reflejos exaltados

El diagnstico probable es: a) b) c) d) Parasitosis del tipo de la giardiasis Bocio multinodular txico Taquicardia paroxstica Enfermedad de Graves

El bocio txico difuso (BTD) constituye la forma ms frecuente de hiperfuncin de la glndula tiroidea (70 % de los casos), que puede aparecer a cualquier edad, aunque por lo general aparece entre la tercera y cuarta dcada de la vida. Esta enfermedad es ms frecuente en la mujer, donde se observa un predominio de 7:1 en relacin con los hombres en regiones no bocigenas. Esta relacin se reduce en las zonas de bocio endmico. Los factores genticos desempean un papel esencial en la etiologa y existe una predisposicin familiar a esta enfermedad de Graves-Basedow. El BTD se caracteriza por la presencia de hipertiroidismo, bocio difuso y elstico, oftalmopata, dermopata, acropaquia tiroidea y onicolisis. Es importante el diagnstico y tratamiento precoz del hipertiroidismo para evitar complicaciones, principalmente las cardiovasculares. Criterios diagnsticos y teraputicos Sospecha clnica Sntomas de mayor valor diagnstico Intolerancia al calor Palpitaciones Nerviosismo Insomnio Irritabilidad Hiperquinesia Fatigabilidad fcil Polifagia Perdida de peso Hiperdefecacin y/o diarreas Signos de mayor valor diagnstico Bocio Taquicardia Temblor de las manos Debilidad muscular Piel hmeda y caliente Hiperquinesia Pelo fino y frgil Bibliografa: 1. Bryer-Ash M. Evaluation of the patient with a suspected thyroid disorder. Obstet Gynecol Clin North Am 2001 Jun;28(2):421-38

2. Weber AL, Randolph G, Aksoy FG. The thyroid and parathyroid glands. CT and MR imaging and correlation with pathology and clinical findings. Radiol Clin North Am 2000 Sep;38(5):1105-29 3. Jarlov AE, Nygaard B, Hegedus L, Hartling SG, Hansen JM. Observer variation in the clinical and laboratory evaluation of patients with thyroid dysfunction and goiter. Thyroid 1998 May;8(5):393-8 4. Costa AJ. Interpreting thyroid tests. Am Fam Physician 1995 Dec;52(8):2325-30

27.- Hombre de 55 aos, el cual presenta distensin abdominal en incremento e ictericia de 6 semanas de evolucin. La EF revela araas vasculares, venas ingurgitadas alrededor del ombligo y ascitis. Lla causa ms probable de esta ictericia es:

a) Hepatitis b) Coledocolitiasis c) Cirrosis d) Cncer de pncreas

1.- CIRROSIS HEPTICA DEFINICIN: Se trata de una alteracin crnica e irreversible que representa la etapa final de diversos procesos agudos o crnicos que afectan al hgado, si bien el concepto es fundamentalmente morfolgico donde se define como una alteracin difusa de la arquitectura del hgado por fibrosis y ndulos de regeneracin, clnicamente se puede sospechar por los antecedentes y los diversos datos clnicos que se pueden documentar. Se sabe que los cambios histolgicos condicionan una alteracin vascular intraheptica y una reduccin de la masa funcional heptica y como consecuencia se desarrolla hipertensin portal e insuficiencia heptica, dando como resultado final sus complicaciones mayores: ascitis, hemorragia digestiva, ictericia y encefalopata heptica.

Cirrosis: Proceso heptico difuso con fibrosis y formacin de ndulos Muchas causas; resultado final similar Necrosis Fibrosis I III Regeneracin Disminucin sntesis Alteraciones vasculares: Hipertensin portal

ALCOHOL Es una de las principales causas de cirrosis El consumo per capita est correlacionado a la mortalidad global por cirrosis Dependencia de alcohol en 5% de la poblacin Umbral de riesgo: 60 g/d en hombres, 20 g/d en mujeres

ETIOLOGA: Consumo excesivo de alcohol. Infeccin crnica por los virus de la hepatitis B y C. Hemocromatosis hereditaria Enfermedad de Wilson Hepatitis autoinmune Esteatohepatitis no alcohlica. Colestasis obstructivas crnicas Obstruccin crnica del drenaje venoso Toxicidad por frmacos (Metotrexato, metildopa, vitamina A, etc.). En los nios la causa ms frecuente es la anormalidad congnita de la va biliar (atresia de va biliar, enfermedad de Alagille, enfermedad de Byler). Cuando no es posible determinar el factor etiolgico (5%) se denomina cirrosis criptogentica. CLNICA: La cirrosis compensada puede ser clnicamente latente, asintomtica y ser descubierta fortuitamente en un examen clnico efectuado por otro motivo. Otros pacientes se diagnostican cuando son explorados por sntomas inespecficos, o en el seguimiento de una hepatitis crnica viral, en algunos otros casos los datos son muy floridos aunados a los antecedentes de relevancia (ingesta intensa de alcohol). Los signos y sntomas que pueden acompaar a la cirrosis son mltiples y muy variados, si bien no todos se presentan en todos los casos ni tiene un carcter patognomnico se pueden observar en la gran mayora de estos enfermos; los podemos clasificar por aparatos y sistemas: SIGNOS Y SNTOMAS DE LA CIRROSIS HEPTICA Sntomas generales: Astenia Adinamia Signos cutneos-ungueales Araas vasculares Telangiectasias Eritema palmar Hipertrofia parotdea Contractura de Dupuytren Acropaquia Uas en vidrio de reloj Desaparicin de la lnula (leuconiquia) Exploracin abdominal Hepatomegalia Esplenomegalia

28.- Masculino de 47 aos de edad con cirrosis heptica tuvo dolor abdominal generalizado durante 24 h sin nuseas ni vmitos. Su temperatura es de 38.3C y ha tenido distensin abdominal con onda de lquido claro. Hay hipersensibilidad difusa en la palpacin abdominal. En la paracentesis se obtuvo lquido transparente con 816 leucocitos/mm3 (85%

polimorfonucleares, 15% linfocitos). La tincin de Gram no muestra bacterias. Cul de los siguientes diagnsticos es ms probable? a) b) c) d) Enfermedad ulcerosa pptica Pancreatitis Colecistitis Peritonitis primaria

El diagnstico ms probable es peritonitis primaria. Aunque es difcil diferenciar la peritonitis primaria (espontnea) por rotura de vscera hueca y contaminacin peritoneal, la presencia de fiebre y la leucocitosis la ascitis sugiere alguna clase de infeccin peritoneal. La pancreatitis se caracteriza por dolor localizado intenso (mesoepigstrico), que se irradia a la espalda. En general, las nuseas y vmitos no se relacionan con pancreatitis aguda. El dolor abdominal en caso de colecistitis se sita en el cuadrante superior derecho y suele haber nuseas y vmitos. El absceso heptico tiende a ser un trastorno subagudo sin datos peritoneales prominentes. En caso de ascitis crnica infectada, las enfermedades ulcerosas ppticas son causa poco probable. Allen R. M. MMS Medicina Interna. 5. Edicin. National Medical Series. Mc. Graw Hill. 2006. (captulo 8 V E 1 a).

29.- Femenino de 56 aos que actualmente se diagnstica con DM tipo 2, se desconoce el tiempo de evolucin de la enfermedad, se enva a valoracin por oftalmologa, la primera lesin que se observa en la retinopata diabtica temprana es:

a) b) c) d)

Exudados blandos Exudados duros Microaneurismas Hemorragias intrarretinianas

La retinopata diabtica tiene cuatro etapas: 1. Retinopata no proliferativa ligera. Esta es la etapa ms temprana de la enfermedad en la que aparecen los microaneurismas. Estas son pequeas reas de inflamacin, que parecen ampollas, en los pequeos vasos sanguneos de la retina. 2. Retinopata no proliferativa moderada. Segn avanza la enfermedad, algunos vasos sanguneos que alimentan la retina se obstruyen. 3. Retinopata no proliferativa severa. En esta etapa muchos ms vasos sanguneos se bloquean, haciendo que varias partes de la retina dejen de recibir sangre. Entonces estas reas de la retina envan seales al cuerpo para que haga crecer nuevos vasos sanguneos. 4. Retinopata proliferativa. En esta etapa avanzada, las seales enviadas por la retina para alimentarse causan el crecimiento de nuevos vasos sanguneos. Esto se llama la retinopata proliferativa. Estos nuevos vasos sanguneos son anormales y frgiles. Crecen a

lo largo de la retina y de la superficie del gel vtreo, el gel incoloro que llena el interior del ojo.

Retinopata diabtica. Forma leve de retinopata diabtica, en la que destacan microaneurismas /microhemorragias de predominio en arcada temporal superior, junto con exudados duros que se acercan a la mcula.

1. The Diabetes Control and Compications Trial Research Grop. The Effect of Intensive treatment of Diabetes on the development and progression of long term complications in insulin dependent Diabetes Mellitus N Engl J. Med; 1993 ; 329 : 977-986. 2. Early treatment Diabetic Retinopathy Study Research Group Early photocoagulation for diabetic Retinopathy ETDRS Report 9. Ophthalmology, 1991 ; 98 : 1316-26. 3. Kahn HA Hiller R. Blindness caused by diabetic retinopathy Am. J. Ophthalmol, 1974; 78 ; 58-67. 4. Ladas ID. Theossiadis GA Long term efectiveness of modified grid photocoagulation for diffuse Macular edema Acta Ophthalmol, 1993 ; 71(3) ; 393-7. 5. Klein R, Klein BEK , Moss SE , et al The Eisconsin Epidemiology Group Ophthalmology, 1984 ; 91 : 1464-74.

30.- Masculino de 50 aos con dolor precordial relacionado al esfuerzo, de corta duracin, de 4 meses de evolucin y con ECG en reposo normal, el siguiente estudio de eleccin es: a) b) c) d) Ecocardiograma de reposo Prueba de Talio- Dipiridamol Ecocardiograma con estrs farmacolgico Prueba de esfuerzo con protocolo de Bruce

El propsito fundamental de la prueba es el de demostrar la existencia de isquemia miocrdica en los subgrupos de poblacin con mayor prevalencia de cardiopata isqumica o bien en los subgrupos en donde la prueba se efecta a manera de evaluacin del tratamiento en pacientes ya conocidos con cardiopata isqumica. Existe otro subgrupo de poblacin en quienes este estudio ayuda a determinar la clase funcional en la que se encuentran e incluye a pacientes con valvulopatas o con insuficiencia cardiaca de cualquier etiologa.

Protocolos de esfuerzo. El protocolo ms empleado es el de Bruce sobre treadmill, aunque existen otros protocolos y su eleccin depender de las condiciones del individuo. Los protocolos discontinuos son los que alternan periodos de esfuerzo que se intercalan con periodos de reposo de duracin similar, se emplean en escasas circunstancias. Los protocolos continuos son los que no interrumpen el esfuerzo una vez iniciado hasta finalizada la prueba, permiten mejor adaptacin fsica y psicolgica y es posible adaptar la intensidad de forma individualizada para que la prueba tenga una duracin de 6 a 12 minutos. Los protocolos mximos son los que se suspenden debido a la sintomatologa del paciente, a los signos registrados durante la prueba o se alcanzan valores mximos de FC y VO2. Los protocolos submximos son los que se suspenden cuando el sujeto alcanza un nivel determinado de carga, habitualmente el 85% de la FC mxima terica (que se encuentra entorno a los 170 lpm). En la prctica diaria, el nivel de carga (VO2) se expresa en forma de trabajo externo (MET equivalentes metablicos) que corresponden a 3,5ml/kg/min de VO2, lo que permite comparar protocolos entre s (cada protocolo dispone de frmulas para realizar el clculo de los METS), el error que cometen en el clculo de los METS es mayor en protocolos discontinuos. Emplear la FC como nico criterio para determinar el esfuerzo mximo es errneo, por lo que deberan tenerse en cuenta otros criterios, como es la percepcin subjetiva por parte del paciente mediante la escala de Borg (tabla I). Esta dificultad en la prediccin del esfuerzo mximo es lo que limita la realizacin de pruebas submximas a la determinacin de la condicin fsica de sujetos aparentemente sanos. Tabla I. Escala de Percepcin del esfuerzo de Borg. Escala de 15 grados Valor Percepcin 6 7 No se siente nada Muy muy leve Escala de 10 grados Valor Percepcin 0 0,5 Nada Muy muy leve

8 9 10 11 12 13 14 15 16 17 18 19 20 Muy leve

1 2

Muy leve Leve Moderada Algo fuerte Fuerte o intensa Muy fuerte

Considerablemente leve 3 Moderadamente dura Dura Muy dura Muy muy dura Esfuerzo mximo 4 5 6 7 8 9 10

Muy muy fuerte (submxima)

* A la izquierda la escala original de esfuerzo percibido en 15 grados (de 6 a 20) y a la derecha la ms nueva de 10 categoras. Tabla II. Indicaciones clsicas de la ergometra. I. Fines diagnsticos A. Pacientes sintomticos. 1. Dolor torcico: a) Tpico b) Atpico 2. Clnica de equivalentes isqumicos. A. Pacientes asintomticos. 1. Con alteraciones en el ECG sugestivas de isquemia. 2. Con alta probabilidad de padecer Cardiopata Isqumica (paciente con mltiples factores de riesgo) 3. Cuando convenga descartar con cierta seguridad CI. 4. Con sospecha de CI silente. 5. Sedentarios que inician programa de actividad fsica. 6. Para estudio funcional de ciertas arritmias. II. Con fines valorativos y pronsticos. 1. Seguimiento de paciente con CI conocida. 2. Tras IAM. 3. En exmenes prelaborales o laborales. 4. De la eficacia del tratamiento: Mdico. Cateterismo y angioplastia. Quirrgico.

5. Respuesta de la Tensin Arterial. 6. En valvulopatas o miocardiopatas. 7. Estudio de arritmias y trastornos de la conduccin aurculo-ventricular. 8. En cardiopatas congnitas. Bibliografa 1. Gibbons RJ (Edit.). ACC/AHA 2002 Guideline Update of Exercise Testing. 2002 American College of Cardiology Foundation and American Heart Association ACC/AHA; 2002 [Acceso 1-4-06]. Disponible en: Guidelines for cardiac exercise testing. ESC Working Group on Exercise Physiology, Physiopathology and Electrocardiography Eur Heart J 1993; 14: 969-988. Fernando Ars Aros F, Boraita A, Alegria E, Alonso AM, Bardaji A, Lamiel R el al. Guas de prctica clnica de la Sociedad Espaola de Cardiologa en pruebas de esfuerzo. Rev Esp Cardiol 2000; 53 (8): 1063-94 Chaitman B. Las pruebas de esfuerzo. En: Braunwald E, editor. Tratado de Cardiologa. Medicina Cardiovascular. 4 ed. Madrid Mc-Graw-Hill-Interamericana de Espaa; 1993. p. 177-197. Schlant RC, Friesinger GC 2nd, Leonard JJ. Clinical competence in exercise testing: a statement for physicians from the ACP/ACC/AHA Task Force on Clinical Privileges in Cardiology. J Am Coll Cardiol 1990; 16: 1061-5 Reyes Lopez de los M, Iiguez Romo A, Goicolea de Oro A, Funes Lopez B, Castro Beiras A. El consentimiento informado en cardiologa. Rev Esp Cardiol 1998; 51: 782796. Fletcher GF, Flipse T, Malouf J, Kligfield P. Current status of ECG stress testing. Curr Probl Cardiol. 1998 Jul; 23(7): 353-423. Alegra Ezquerra E, Alijarde Guimer M, Cordo Mollar JC, Chorro Gasc FJ, Pajarn Lpez A. Utilidad de la prueba de esfuerzo y de otros mtodos basados en el electrocardiograma en la cardiopata isqumica crnica. Rev Esp Cardiol 1997; 50: 6-14 Wasserman K, Hansen JE, Sue DY, Whipp BJ, Casaburi R. Principles of exercise testing and interpretation . 2 ed. Philadelphia: Lea & Febiger; 1994. p. 95-111. American college of Sports Medicine. Guideliness for exercise testing and prescription. 5 ed. Baltimore: Williams & Wilkins; 1995. Borg GA. Psychophysical bases of perceived exertion. Med Sci Sports Exerc 1982; 14: 377-381 Froelicher VF, Umann TM. Exercise testing: clinical applications. En: Pollock ML, Schmidt DH, editors. Heart disease and rehabilitation. 3 ed. Champaign, IL: Human Kinetics, 1995; p.57-79. Myers J, Froelicher VF. Exercise testing. Procedures and implementation. Cardiol Clin. 1993; 11(2): 199-213. Weiner DA, McCabe C, Hueter DC, Ryan TJ, Hood WB Jr. The predictive value of anginal chest pain as an indicator of coronary disease during exercise testing. Am Heart J 1978; 96: 458-462.

2. 3.

4.

5.

6.

7. 8.

9. 10. 11. 12.

13. 14.

31.- Preescolar de 4 aos de edad que acude al servicio de consulta externa diagnosticado con imptigo, el tratamiento fundamental primario de este padecimiento es:

a) b) c) d)

Es suficiente descostrar y usar antibitico tpico Penicilina por 10 dias Dicloxacilina por 7 das Aciclovir por 5 das a dosis habituales

IMPETIGO Frecuente en la infancia. Ms raro en el adulto, donde hay que buscar una dermatosis subyacente (a menudo parasitaria). Favorecida por falta de higiene. Muy contagiosa. No deja inmunidad por lo que frecuentemente recidiva. Definicin Dermo-epidermitis superficial microbiana producida por Streptococcus pyogenes y/o Staphylococcus aureus. Etiologa Actualmente es ms frecuente la etiologa estafiloccica que la estreptoccica, aunque muchas veces ambos grmenes se asocian. Pilares diagnsticos a) clnico b) paraclnica. De poca utilidad prctica. Tratamiento primario: Mejorar las condiciones de higiene de la piel (generales y locales). Evitar maceracin, humedad y rascado de las lesiones. Ducha diaria. Lavado repetido de manos, cepillado de uas que deben mantenerse cortas. Cambio frecuente de ropa interior. a) local, nico tratamiento en las formas muy localizadas. Realizar varias veces al da en forma sucesiva: Lavado y descostrado de las lesiones. Reblandecer las costras con compresas hmedas o aplicacin de vaselina. Antisptico local: solucin de chlorhexidine o permanganato de potasio o sulfato de cobre. Crema con cido fucdico o bacitracina o mupirocina. En lo posible tapar la lesin para evitar auto y heteroinoculacin Ropa interior de algodn y amplia para evitar el roce.

BIBLIOGRAFIA

- Gantz NM, Brown RB, Berk SL, Espsito AL, Gleckman RA. Human infections following animal bites. In Manual of Clinical Problems in Infectious Disease. Nelson M Gantz et al. - Finegold SM, Baron EJ. Microorganisms encountred in wounds, abscesses, skin and soft tissue lesions. In Bailey & Scotts Diagnostic Microbiology - Gradon J & Adamson C. Infections of Pressure Ulcers: Manegement and Controversies. Inf Dis Clin Practice 1:11-16 - Hirschman JV, Finegold DS. Cutaneaus Abcsesses and Ulcers in Infectious Diseases Ch 154. Gorbach SL, Bartlett JG and Blacklownr editors. WB Saunders co. 199 - Swartz MN. Enfermedades Infecciosas. Mandell GL, Bennett JE y Dolin R. 4ta Edicin. T 1Cap 72:1010-1032.

32.- Thrombocytopenia that is caused by increased platelet destruction is most closely associated with which of the following conditions?

a) b) c) d)

Combination chemotherapy Systemic lupus erythematosus Acute leucemia Excessive ethanol intake

El diagnstico de LEG se hace mediante la identificacin de manifestaciones clnicas de la enfermedad acompaadas de uno o ms autoanticuerpos tpicos (Tablas 2 y 3).

Tabla 3 Criterios para la Clasificacin del Lupus Eritematoso Generalizado Eritema malar Eritema discoide Fotosensibilidad Ulceras orales Artritis Serositis a. pleuritis b. pericarditis Renal:

c. proteinuria persistente > 0,5 g/24 h o +++ d. cilin

Neurolgicos: e. convulsiones

f.

psicosis

Hematolgicos: g. Anemia hemoltica con reticulocitosis h. Leucocitopenia < 4.000 por dos veces i. Linfocitopenia < 1.500 por dos veces j. Trombocitopenia < 100.000 10. Inmunolgicos: a) Anti-DNA elevado b) Anti-Sm c) Antifosfolpidos: Anticardiolipinas (IgG o IgM) o Anticoagulante lpico o VDRL falso positivo por 6 meses. 11. Anticuerpos antinucleares (en ausencia de lupus por drogas)

33.- Hombre de 44 aos, refiere malos hbitos alimenticios comenta que con mucha frecuencia consume alimentos en la va pblica, actualmente se presenta con fiebre, edema facial, fotofobia y mioartralgias, estos datos son sugestivos de: a) b) c) d) Larva migrans visceral Fasciolosis Criptosporidiosis Trichinellosis

DEFINICIN Se denomina triquinosis a la Infeccin parasitaria producida por nemtodos del gnero Trichinella, transmitida por carnivorismo, y caracterizada por un sndrome febril, signos oculopalpebrales, mialgias y eosinofilia elevada. Sinonimia: Trichinellosis. AGENTE Y CLASIFICACIN La Trichinella es un pequeo nemtodo blanquecino y filiforme, con su extremidad anterior ms adelgazado que la posterior. La hembra mide entre tres y cuatro milmetros, en tanto que el macho es de menor tamao. Originalmente, se reconoca como nica especie a la Trichinella spiralis (Owens, 1835), pero en diversas reas geogrficas se han descrito recientemente triquinas que, aunque morfolgicamente similares, presentan sutiles diferencias en sus caractersticas biolgicas. As, en la actualidad se distinguen:

1. 2.

Trichinella spiralis, propia de las zonas geogrficas templadas. Trichinella pseudospiralis, la cual, aunque no es frecuentemente observada afecta ms a las aves que a los mamferos y se caracteriza por ser de menor tamao y por no formar quistes en la musculatura del hospedador. Trichinella nelsoni del Africa tropical, la cual se encuentra en los grandes carnvoros de la regin, presenta un bajo grado de infectividad para los cerdos domsticos y ratas de laboratorio, y en el hombre provoca intensas infecciones con un gran nmero de larvas por gramo de msculo; aunque ha sido fatal en ocasiones, es muy bien tolerada a pesar de la masividad de la infeccin. Trichinella nativa de las zonas rticas que, distintivamente, presenta una considerable resistencia a la congelacin, tiene bajo grado de infectividad para el cerdo domstico, y en el hombre provoca importantes sntomas digestivos, principalmente diarreas prolongadas. Si bien T. spiralis, T. nelsoni y T. nativa son morfolgicamente similares, se ha establecido la diferenciacin entre ellas por sus caractersticas isoenzimticas y mediante el uso de anticuerpos monoclonales. En cambio, existen diferencias estructurales entre esas tres "especies" y la T. pseudospiralis.

3.

4.

Ciclo domstico: El cerdo adquiere la infeccin, principalmente, por la ingestin de ratas infectadas, lo que es posible cuando es criado en malas condiciones higinicas o, simplemente, cuando debe buscar su propia fuente de alimentacin en sitios eriazos o basurales; adems, el cerdo se infecta con carnes de otros animales que encuentra en los criaderos o en los basurales. Las ratas, debido principalmente a sus hbitos de canibalismo, mantienen y propagan la infeccin en la naturaleza.

Los principales huspedes domsticos de la T. spiralis son la rata, el cerdo y el hombre. El hombre adquiere la infeccin a travs de la ingestin de carne de cerdo cruda o insuficientemente cocida, con larvas de triquina. Los jugos digestivos digieren la carne y las larvas quedan en libertad en el intestino, donde rpidamente, ya a las cuarenta y ocho horas, se diferencian en hembras y en machos adultos. Copulan en el lumen intestinal y, mientras los machos son eliminados con las deposiciones del husped luego de cumplida su funcin gensica, las hembras grvidas - que son vivparas- se localizan en el interior de la mucosa del duodeno y del yeyuno. Entre el tercero y el quinto da, comienza la postura de larvas. Cada hembra coloca alrededor de 1.500. Estas larvas miden entre 80 y 120 micrones, se profundizan en la mucosa intestinal, penetran a travs de los capilares linfticos y venosos y llegan a la circulacin general, diseminndose por todo el organismo, pero enquistndose slo en la musculatura, esqueltica. Las larvas se localizan en el interior de las fibras musculares, destruyndolas parcialmente; al cabo de unos quince das, quedan rodeadas por una envoltura constituida por el sarcolema. As se origina el quiste larval, que mide entre 250 a 400 micrones y que, en consecuencia, no es visible a simple vista, aunque puede observarse con una lupa o con un microscopio de poco aumento. Tiene

un aspecto fusiforme o alargado, que recuerda la forma de un limn, y contiene, enrollado en su interior, una o varias larvas de triquina. La invasin de la musculatura esqueltica por las larvas, comienza alrededor del sptimo da de ocurrida la infeccin y contina mientras existan hembras grvidas en el intestino. Al cabo de un mes, las larvas completan su encapsulamiento y a los seis meses, se inicia el depsito de calcio en las paredes del quiste. La calcificacin total se alcanza en un plazo aproximado de un ao. En consecuencia, un mismo individuo es, sucesivamente, husped definitivo e intermediario del parsito. Es hospedero definitivo cuando alberga en su intestino las formas adultas, y es intermediario cuando las larvas se localizan en su musculatura. Sin embargo, para completar todo su desarrollo, la Trichinella requiere siempre de dos huspedes Referencias: 1. Leo X. Liu Peter F. Weller. Helminthic Infections: Trichinella and other tissue Nematodes. In Baunwald E, Faucy AS, Kasper DL, Hauser SL, Longo DL, Jameson JL, editors. Harrisons Principles of Internal Medicine. 15 Th ed. New York . McGraww-Hill; 2001; 1231-3. 2. Pozio E. New patterns of Trichinella infections. Vet Parasitol 2001; 98: 133-48. 3. Murrel KD. Trichinellosis: now and forevermore?. Parasite 2001; 8 (2 supl.): 11-3. 4. Snchez Rodrguez A, Martnez Lpez de Letona J, Arias paciencia M, Snchez Garca AM, Paz Bouza J, Jarrin J, et al. Triquinosis. Estudio de 21 casos en un mismo brote. Rev Clin Esp 1982; 165: 79-84. 5. Perucha Gonzlez M, Lezaun Larrumbe ME, Torres Baile JL, Campo Hernndez JM, Bernal Martnez A. Brote de Triquinosis en varias localidades de la Rioja Baja. Rev Sanid Hig Pblica (Madrid) 1987; 61: 1035-47. 6. De la Torre Cecilia C, Espino Aguilar R, Crdenas Talavern C, Canuelo Ruiz O, Garrido Palomo R, Baena Sez J, et al. Triquinosis: presentacin de 2 casos. An Esp Pediatr 1989; 30: 227-8.

34.- Ante un paciente con diagnstico de trastorno obsesivo-compulsivo, usted decide indicar el siguiente frmaco por ser el principal tratamiento de eleccin: a) b) c) d) Antidepresivos inhibidores de la recaptacin de serotonina. Antipsicticos. Antidepresivos inhibidores de la recaptacin de noradrenalina. Benzodiacepinas.

El abordaje farmacolgico del TOC se basa en los inhibidores selectivos de la recaptura de serotonina (ISRS), medicamentos que han resultado ser efectivos y seguros. Como grupo, los ISRS son igual de eficaces que la clorimipramina, pero producen menos efectos secundarios y, por lo tanto, mejor tolerancia y mejor apego al tratamiento. La efectividad antiobsesiva parece ser independiente de su actividad antidepresiva. Entre los factores predictores de respuesta al tratameinto, se ha propuesto que los altos puntajes en las compulsiones predicen una mala respuesta al tratamiento. Para los casos

refractarios y resistentes o cuando hay sntomas de comorbilidad, se han utilizado, con relativo xito, combinaciones con diferentes IRS, con benzodiacepinas, o bien, con potenciadores como el litio, o antipsicticos, como la risperidona y el haloperidol. El uso del carbonato de litio es controvertido, aunque parece ser til como potenciador a largo plazo entre 15 y 30% de los pacientes. La combinacin de ISRS con antipsicticos comenz a utilizarse en los pacientes con sntomas psicticos, aunque ahora se combinan tambin en los pacientes resistentes.

Tratamiento farmacolgico del TOC Cristina Lyzaga* Humberto Nicolini* *Divisin de Investigaciones Clnicas. Instituto Nacional de Psiquiatra. Ramn de la Fuente. Calzada Mxico-Xochimilco, 101, San Lorenzo Huipulco, 14370, Mxico D.F. Primera versin: 14 de septiembre de 2000. Aceptado: 26 de septiembre de 2000.

35.- Femenino de 66 aos que acude al servicio de neurologa para valoracin, al realizar exploracin fsica, usted determina que ste, comprende pero no puede articular palabra, El tipo de afasia que presenta ste paciente es:

a) Wernike b) Broca c) Global d) Anmica

Afasia De broca, motriz o de expresin (no fluente) De Wernicke (no comprende) De conduccin (no denomina no repite) Anmica (no denomina) Transcortical motora (no fluente) Transcortical sensitiva (no denomina no comprende) Global (no fluente-no denomina-no repite-no comprende) Talmicas y putaminales (lenguaje espontaneo incomprensible)

Referencias: 1.Aronson A y cols. Examen clnico neurolgico, 3 Edicin. La Prensa Mdica Mexicana, Mxico, 1995. 2. Uribe CS, Arana A, Pombo PL. Neurologa, 5 Edicin. Corporacin para investigaciones biolgicas. Colombia, 1996. 3.-Adams R, Vctor M. Principles of Neurology. Mc Graw Hill. 7th ed. USA 2001.

4.-Bradley W.G. Neurology in clinical practice. Butterworth Heinemann. 4th ed. Philadelphia USA, 2004. 5.-Silberstein SD, Lipton RB, Goadsby PJ. Headache in clinical practice. Isis Medical Media. Oxford University Press, UK, 1998. 6.-Barinagarrementera F. Cant C. Enfermedad vascular cerebral. McGrawHill Interamericana. Mxico D.F. 1997.

36.- Masculino de 39 aos, limpia vidrios, con antecedente de fractura vertebral de C6 y fractura del extremo distal del radio derecho tras cada de un tercer piso se presenta a consulta, refiere dolor, limitacin de la movilidad y hormigueo en los dedos 2 y 3 de la mano derecha, de predominio nocturno, que mejora durante el da. Cul de los siguientes diagnsticos es el ms probable?: a) b) c) d) Radiculalgia C5-C6. Distrofia simptico refleja. Sndrome del tnel del carpo. Artrosis postraumtica del carpo.

El Sndrome del Tnel Carpiano (STC) es una patologa que afecta a la mano, provocada por una presin sobre el nervio mediano a nivel de la mueca. Esto provoca sntomas como adormecimiento y hormigueos en la mano (especialmente en los dedos pulgar, ndice, corazn y mitad del anular). Puede existir dolor, que puede estar limitado a la mano y mueca, pero que en algunas ocasiones se irradia hacia el antebrazo. El STC con frecuencia despierta al paciente por la noche, y los sntomas pueden aparecer con actividades como conducir un vehculo, escribir, u otros ejercicios que suponen una utilizacin significativa de la mano. En el sndrome del tnel del carpo avanzado, puede producirse una prdida de fuerza y una disminucin de la masa muscular en la base del pulgar. CAUSA El tnel del carpo es un canal formado por los huesos de la mueca y un ligamento (el ligamento transverso del carpo) situado en la cara palmar de la mueca. Por este tnel transcurren todos los tendones que flexionan la mueca y los dedos, y el nervio (nervio mediano) que recoge la sensibilidad del pulgar, ndice, corazn y parte del anular y moviliza los msculos de la base del pulgar. Algunas personas nacen con tneles estrechos y por tanto estn predispuestas a problemas de presin sobre el nervio. La utilizacin vigorosa de la mano, que conduce a una tendinitis de los tendones que flexionan el pulgar y los dems dedos, tambin puede conducir a un sndrome del tnel del carpo a travs del engrosamiento de las vainas tendinosas. Las vainas engrosadas "rellenan" el tnel presionando sobre el nervio. Las personas con artritis reumatoide, hipotiroidismo, diabetes, amiloidosis, insuficiencia renal y algunos otros problemas mdicos estn ms predispuestas a padecer este sndrome.

1.

Arthroshi I, Gummenson C, Johonsson R, Ornstein E, Ranstam J, Rossen I. Prevalence of carpal tunnel syndrome in a general population. JAMA. 1999;282:1538

2. Peridico El Pblico, 17 de octubre del 2007,pag 26 3. Durkan, JA. The carpal compression test: an instrumental device for diagnostic carpal tunnel syndrome. Lancet. 1990;335:393-5. 4. Marshall S, Tardif G. Injection local of steroids in the carpal syndrome. Cochrane Data Base of Systematic Reviews. 2005. Issue 5.

5. Verdugo RJ, Salinas RS, Castillo J, Cea JG. Tratamiento quirrgico versus tratamiento no quirrgico para el sndrome del tnel carpiano. Cochrane Data Base. 2005

37.- Se trata de paciente femenino de 43 aos de edad con diagnstico de polimiosistis al realizar la exploracin fsica, la disminucin de la fuerza muscular se caracteriza por:

a) b) c) d)

Ser de predominio distal Acompaarse de dolor intenso Ser de predominio proximal Afectar nicamente a la musculatura distal

POLIMIOSITIS
CUADRO CLNICO
Inicio insidioso: 3-6 meses Sntomas constitucionales Debilidad muscular proximal y simtrica Musculatura de la cintura escapular y plvica (suele empezar en la plvica); musculatura cervical Respeta msculos oculares y faciales Mialgias / Artralgias / Raynaud

La polimiositis (PD) y dematomiositis (DM), son enfermedades inflamatorias en las cuales el compromiso principal es la debilidad muscular, generalmente proximal y simtrica con atrofia subsecuente, generalmente indolora. Su etiologa y patogenia siguen siendo desconocidas, siendo considerada dentro de las enfermedades difusas del tejido conectivo.

Numerosos estudios sugieren que una exposicin previa a diversos virus como Influenzae A y B, Coxsackie virus tipo B y Picornavirus pueden desencadenar PM-DM; adems se cree que los fenmenos inmunolgicos juegan un papel preponderante, por la presencia de autoanticuerpos, depsito de inmunoglobulinas y complemento en las paredes vasculares. A nivel anatomo-patolgico las principales alteraciones se encuentran en los msculos esquelticos y en los vasos sanguneos. Siendo la vasculitis un signo comn en la dermatomiositis infantil y su severidad indicar una peor evolucin. Clnicamente los sntomas iniciales son la debilidad muscular proximal, el rash tpico, frecuente el edema en cara y a veces en miembros, fiebre moderada, posteriormente dificultad para la deglucin y debilidad en la voz; asimismo manifestaciones viscerales que no se presentan siempre, pero son de gran importancia en cuanto al pronstico, asimismo transtornos ventilatorios (pulmonares): por afectacin primaria (neumona intersticial) o secundaria a la disfuncin farngea (aspiracin) o a la debilidad de los msculos respiratorios (insuficiencia ventilatoria), o como complicacin del tratamiento (infecciones oportunistas).

Wortmann RL. Idiopathic inflammatory myopathies. A. Clinical features. In: Primer on the rheumatic diseases. New York: Springer-Arthritis Foundation; 2008. p. 363-7.

38.- Se trata de masculino de 19 aos, que presenta hemartrosis. Tiene TPT alargado, TP y TT normales, por lo que tiene una alteracin de la va: a) Del Complemento b) Intrnseca c) Extrnseca d) Colinrgica

La va intrnseca est constituida por la activacin secuencial de los factores XII, XI, IX,. VIII y V. el tiempo de tromboplastina parcial activada mide la actividad de la coagulacin intrnseca y sirve para monitorizar el tratamiento con la heparina no fraccionada.

39.- Masculino de 12 aos que inicia con dolor intraescrotal intenso de aparicin brusca al encontarse realizando actividad fsica, se tiene la sospecha de una torsin de testculo, Cul es, entre las siguientes, la prueba diagnstica de eleccin?:

a) b) c) d)

Ecografa. Ecografa-doppler. Radiografa escrotal. Palpacin bimanual.

Ecografa eco-doppler color: Es la prueba diagnstica de eleccin al permitir la visualizacin de la vascularizacin testicular pudiendo distinguir si el flujo circulatorio es normal, ausente o aumentado. En la torsin lo normal es que est reducido o ausente. Tiene una especificidad del 100% y una sensibilidad del 80%, aunque tiene sus limitaciones en las subtorsiones. Gammagrafa isotpica testicular con TC 99: Es tambin un mtodo sensible y especfico, pero del que no siempre se puede disponer en urgencias. Se ver un rea de captacin disminuida o ausente, rodeado de un rea de captacin aumentada por la hiperemia reactiva.

Bibliografa: 1) Sesions AE et al: Testicular torsin: direction, cadem, duration and desinformation. J. Urol 2003, 169: 663-665. 2) Cummings JM el al: Adult testicular torsin. J.Urol. 202, 167: 2109, 167: 2109-2110 3) Bedos F et al: Manual de Urologa. Ed Masson SA. Barcelona 1996. 297-335.

40.- A travs de un estudio de casos y controles se quiere conocer si existe asociacin entre tabaquismo y cncer de lengua. La variable tabaquismo se medir como Leve (uno a 10 cigarrillo en 24 hrs), moderado (11 a 20 cigarrillos en 24hrs) severo (ms de 21 cigarrillos en 24 hrs). Como podemos clasificar esta variable:

a) b) c) d)

Cuantitativa continua Cuantitativa discreta Cuantitativa discontinua Cualitativa ordinal

En las variables de tipo ordinal las observaciones se clasifican y ordenan por categoras segn el grado en que los objetos o eventos poseen una determinada caracterstica. Por ejemplo, se puede clasificar a las personas con respecto al grado de una enfermedad en leve, moderado o severo.

Moreno A. Principales medidas en epidemiologa. Rev Salud Pblica Mex, 2000;42( 4): 338

41.- Femenino de 27 aos. Acude al servicio de urgencias por presentar salida de lquido vaginal. Antecedentes: G2, P1, cursa embarazo de 36 semanas de gestacin exploracin fsica: cervix con 10% de borramiento, 1 cm de dilatacin y Tarnier positivo. La complicacin ms frecuente en esta paciente es?

a) b) c) d)

Parto pre trmino. Sepsis neonatal. Corioamnioitis. Endometritis.

Corioamnioitis: El diagnstico de la infeccin intraamnitica (IIA) es bsicamente clnico. La corioamnionitis se debe descartar en toda gestante que presente fiebre sin foco aparente, sobre todo si se sospecha o se ha confirmado una rotura de membranas. Los criterios ms empleados para el diagnstico son: fiebre materna y, al menos, 2 de los siguientes signos: taquicardia materna, taquicardia fetal, irritabilidad uterina, leucocitosis materna o lquido amnitico purulento o maloliente. Progresos de obstetricia y ginecologa: ISSN 0304-5013, Vol. 48, N. 6, 2005 , pags. 316317.

42.- Mujer de 26 aos, con embarazo de trmino, sin antecedentes de control prenatal. G 3. C-1. Se ingresa al servicio de obstetricia por presentar actividad uterina regular y dolorosa. Ef.: Deambulante, tranquila, adecuada coloracin de tegumentos, abdomen con fondo uterino a 32 cm. con producto nico vivo en situacin transversa dorso inferior FCF 144, al tacto vaginal crvix dilatado a 3 cm. y membranas ntegras. Se realiza cesrea con retencin de placenta e invasin a vejiga. La alteracin placentaria que presenta esta paciente es: a) b) c) d) Placenta increta Placenta acreta Placenta marginal Placenta percreta

Es la penetracin y adherencia anormal de la placenta en la pared uterina. Se divide en: Placenta acreta. Placenta increta. Placenta percreta. ACRETA: Las vellosidades se adhieren al miometrio. INCRETA: Penetran ms de la mitad del espesor del miometrio. PERCRETA: Atraviesa todo el espesor del miometrio, llegando a la serosa, incluso atravesndola y adhirindose a rganos vecinos.

Factores: Endometrisis previa.

Tumores submucosos. (Miomas) Cicatrz uterina previa. (Cesrea, miomectoma) Implantacin baja. (Placenta previa) Malformaciones placentarias. (Placenta extracorial) Legrado enrgico previo. Extraccin manual previa de una placenta.

Diagnstico transparto: Placenta retenida por ms de 20 minutos. Imposibilidad para encontrar un plano de separacin placentaria cuando se intenta su extraccin manual. Hemorragia incontrolable despus de la pseudoextraccin. El diagnstico histopatolgico corrobora el diagnstico clnico. Escenario menos deseable. Tratamiento: Histerectoma Obsttrica. Constituye una ciruga no planeada y secundaria al hallazgo del acretismo placentario con sangrado incohercible. Cesrea-Histerectoma. (Con diagnstico previo) Ciruga planificada ante un correcto diagnstico prenatal. Recomendacin ACOG: Maduracin pulmonar intrauterina. Inyectar al cordn umbilical 50 mg de metrotexate. Ligar el cordn en el nacimiento placentario y dejar la placenta in-situ. Embolizacin inmediata de arterias uterinas bilaterales, as como de ramas de la divisin anterior de la arteria iliaca interna con alcohol polivinlico. Continuar con 5 dosis I.M. de 50 mg de metrotexate y cuantificar niveles de hCG. Programar Histerectoma Total Radical Abdominal y/o Cistectoma parcial y/o reseccin pared anterior recto.

Lee et al. Conservative Management of Placenta Percreta. Obstet Gynecol, 112(2):421-424

43.- Lactante de un ao que inici con un cuadro diarrico prolongado, recibi tratamiento con antibiticos, se report con mejora clnica, posteriormente reinici con el cuadro enteral El agente causal ms probable en la recada es? a) b) c) d) Escherichia Coli Shigela Flexneri Clostridium Perfirgens. Clostridium Difficile

Clostrirum difficile es un bacilo anaerobio gram positivo, responsable de la colitis asociada a antibiticos. Coloniza el tracto digestivo despus de que su flora habitual haya sido alterada por el tratamiento antibitico. Tras la colonizacin, produce dos exotoxinas (toxina A y toxina B) que se unen al los receptores de las clulas epiteliales intestinales, conduciendo a la produccin de una diarrea secretora. Es una de las infecciones nosocomiales ms comunes, con una elevada morbimortalidad. La transmisin es fecal-oral y ha producido importantes epidemias dentro del ambiente hospitalario. No todas la cepas de Clostridium son productoras de enterotoxinas, en ocasiones se comportan como colonizantes del tracto digestivo sin producir enfermedad. Factores de riesgo: Uso de antibioterapia: prcticamente todos las antibiticos pueden predisponer a la colonizacin por Clostridium difficile. Asociacin frecuente: Ampicilina, Amoxicilina, Cefalosporinas y Clindamicina Asociacin ocasional: otras Penicilinas, Sulfamidas, Eritromicina, Trimetropim y Quinolonas. Raramente o nunca asociados: Aminoglucsidos, Tetraciclinas, Cloramfenicol, Metronidazol y Vancomicina. Pacientes con edad avanzada y debilitados. Intervencin quirrgica sobre tracto gastrointestinal. Contacto previo con paciente colonizado. Alimentacin enteral.
Manifestaciones clnicas: El espectro clnico abarca desde portadores asintomticos hasta enfermedad fulminante con megacolon txico. Los sntomas ms frecuentes son diarrea acuosa, dolor abdominal y fiebre. Las manifestaciones clnicas se inician entre 5-10 das despus de la administracin de antibiticos, aunque pueden aparecer el primer da o hasta 10 das despus de haber cesado el tratamiento antibitico. Colitis: Diarrea profusa (ms de 10 deposiciones al da) Presencia de leucocitos en heces. Fiebre, nauseas, anorexia, leucocitosis con desviacin a la izquierda. Dolor y distensin abdominal. Colitis parcheada difusa en la sigmoidoscopia. Colitis pseudomembranosa: Mayor nmero de deposiciones. Presencia de leucocitosis y sangre en heces. Fiebre, nauseas, anorexia, leucocitosis con desviacin a la izquierda. Ms marcada distensin abdominal. Placas adheridas, amarillentas de 2-10 mm en la sigmoidoscopia

BIBLIOGRAFIA: Bartlett JG, Moon N, Chang TW, et al. The role of Clostridium difficile in antibioticassociated pseudomembranous colitis. Gastroenterology ; 75:778.

Thomas C, Stevenson M, Riley TV. Antibiotics and hospital-acquired Clostridium difficileassociated diarrhea: a systematic review. J Antimicrob Chemother 2003; 51:1339. Tedesco FJ. Pseudomembranous colitis: pathogenesis and therapy. Med Clin North Am ; 66:655. Fekety R, Shah AB. Diagnosis and treatment of Clostridium difficile colitis. JAMA; 269:71.

44.- Lactante masculino de 8 meses de edad, con 8 Kg. de peso, antecedentes de ser sano, de manera sbita inicia con clico intenso acompaado de sudoracin y palidez, rechazo de alimentos, vmitos y evacuaciones mucosanguinolentas, a la E.F. Se palpa masa en colon transverso, la primera posibilidad diagnstica es: a) b) c) d) Apendicitis aguda. Invaginacin intestinal. Divertculo de Meckel. Malrotacin intestinal.

Invaginacin intestinal Obstruccin intestinal aguda provocada por la introduccin del tubo digestivo dentro de s mismo. Clnica: lactante sano y bien nutrido, que de forma sbita, cada 10-15 minutos, durante unos minutos, presenta crisis de llanto, acompaadas de sntomas vagales (palidez, sudoracin, decaimiento), irritabilidad y rechazo del alimento. Inicialmente permanece asintomtico entre los episodios de llanto. De forma progresiva aparecen vmitos, anorexia, decaimiento, sangre roja en heces, e incluso colapso vascular y shock. Exploracin fsica: Fosa iliaca derecha vaca. Signo de Dance (13%). Palpacion de masa en hipondrio derecho y colon transverso signo de morcilla (24 a 90%) Tacto rectal, hocico de tenca (0.5 al 3%) aumento de ruidos hidroareos y

Puede presentar sangre en el tacto rectal, excepcionalmente prolapso rectal de la invaginacin. Rx. Simple:

Patrn anormal de aire Opacidad en CID 25-60% Datos de oclusin Distensin de asas Niveles hidroareos Ausencia de aire en recto

Tratamiento: Ayuno. Terapia hdrica. Esquema Antimicrobiano. Ampicilina-Amikacina. Sonda a derivacin Catter Venoso. Sonda Urinaria.

ONEILL PEDIATRIC SURGERY 1998 URG. PED. HIM 5 ED 2002 OLDHAM SURGERY OF INFANTS AND CHILDREN. 1997

45.- Hombre de 72 aos, diagnosticado de carcinoma de prstata, en este momento acude al servicio de urgencias por presentar confusin mental, nuseas, vmitos y estreimiento. Se realizan pruebas de laboratorio y destaca una calcemia de 16mg/Dl. La primera medida teraputica que debe tomarse ante ste paciente es: a) b) c) d) Administracin de Solucin salina y furosemida por va intravenosa. Hormonoterapia (leuprolide y estrgenos). Difosfonatos por va oral. Glucocorticoides por va intravenosa.

Los sntomas que provoca la hipercalcemia estn en relacin con sus niveles en sangre. Valores comprendidos entre 10,5 y 12 gr/dl no suelen provocar sntomas; a partir de estos valores la sintomatologa es progresiva, afectando con ms intensidad y gravedad a todos

los sistemas del organismo. No est establecido un orden de aparicin de la sintomatologa, ni su correspondencia con los valores de calcemia. Sntomas de la hipercalcemia Sistema nervioso central Desrdenes mentales Dificultades cognitivas Ansiedad Depresin Confusin, estupor y coma Calcificacin corneal Suicidios (descritos aisladamente) Sistema neuro-muscular Fatiga o cansancio muscular Mialgias Descenso de la funcin de msculos respiratorios Laxitud articular Sistema renal Nefrolitiasis Diabetes inspida nefrognica (poliuria y polidipsia) Deshidratacin Nefrocalcinosis Sistema gastrointestinal Nuseas y vmitos Anorexia Estreimiento Dolor abdominal Pancreatitis lcera pptica Sistema esqueltico Dolor seo Artritis Osteoporosis Ostetis fibrosa qustica Resorcin subperistica Quistes seos Embarazo Hipoparatiroidismo neonatal Tetania neonatal Bajo peso al nacer Retraso crecimiento intrauterino Hiperemesis gravdica Alta morbilidad neonatal y materna Partos pretrmino Sistema cardiovascular Hipertensin arterial Calcificacin vascular Calcificacin miocrdica

Otros

Hipertrofia miocrdica Acortamiento intervalo QT Arritmias cardacas Queratitis Conjuntivitis Anemia normoctica normocrmica Gota o pseudogota

El tratamiento de la hipercalcemia est orientado fundamentalmente a eliminar la causa que la produce. En los casos sintomticos se requiere de un tratamiento inicial especfico. Debe ser tratada la que presenta sntomas o supera los 14 mg/dl. Los mecanismos para actuar sobre la hipercalcemia son: Aumento de la eliminacin renal de calcio. Disminucin de la absorcin intestinal de calcio. Disminucin de la resorcin sea de calcio. El tratamiento siempre debe iniciarse con la rehidratacin del paciente, que produce un aumento del volumen extracelular, alcanzando una ganancia de volumen de 1,5- 2,5 litros en las primeras 24 horas. A continuacin, se utilizar un diurtico de asa, tipo furosemida, que aumenta la excrecin renal de sodio y calcio. La dosis de diurtico se ajustar en funcin de las cifras de calcio a lo largo del tratamiento. Cuando la funcin renal est comprometida puede recurrirse a la hemodilisis con calcio bajo en el lquido de dilisis. En los casos en los que est implicado un aumento de la resorcin sea como causa de hipercalcemia, deberemos controlarla con: Bisfosfonatos: disponemos de clodronato y pamidronato. El ms empleado es el primero, porque la dosificacin es ms fcil. Su administracin en el caso de la hipercalcemia es intravenosa, inicia el efecto a las 72 horas y alcanza el mximo a la semana. Calcitonina: acta ms rpido que los bisfosfonatos, pero slo mantiene el efecto durante 72 horas.

Otros frmacos antirresortivos: Mitramicina y el nitrato de galio, con menor experiencia, pueden usarse en caso de fracaso de los anteriores. En aquellos casos en los que est aumentada la absorcin intestinal de calcio, como en la produccin endgena de vitamina D (enfermedades granulomatosas o linfomas) deben tratarse con glucocorticoides. LECTURAS RECOMENDADAS 1. Achogue HJ, Madias EN. Changes in plasma potassium concentration during acute acid base disturbances. Am J Med 71:456, 2. Humphreys MH. Urgencias por trastornos en los lquidos, electrolitos y equilibrio cido bsico. En: Diagnstico y Tratamiento de Urgencias. Editado por MT Ho y CE Saunders. Editorial El Manual Moderno. Mxico DF, 3. Mora JM, Delgado VA, D'Achiardi R. Trastornos del potasio. En: Manual de Urgencias en Medicina Interna. Captulo Central. .

46.- Se trata de mujer de 44 aos, acude a consulta por presentar cefalea frontal de predominio matutino. Antecedentes ginecoobsttricos: ciclos menstruales irregulares. E.f.: estatura 1.56 m, peso 82 kg, T/a 140/100, glucemia en ayunas de 120 mg/dl, triglicridos 245 mg/dl, Hdl 37, colesterol total 320 mg/dl. El diagnstico es: a) b) c) d) Obesidad mrbida. Sndrome metablico. Hipertensin arterial esencial. Sndrome premenopusico.

Referencia:

OMS
DM o glucosa de ayuno anormal o intolerancia a la glucosa o resistencia a la insulina y 2 ms:

Sx metablico
Predominio de adipocitos viscerales
Adipocitos metablicamente activos Glucocorticoides activos Menos leptina

Glucocorticoides inactivos

lipotoxicidad

Resistencia a la insulina hiperglucemia

47.- Masculino de 20 aos de edad, acude a su clnica un lunes por la maana, el motivo de consulta es la presencia de ms de 8 evacuaciones lquidas con sangre, moco y pujo, las ltimas 24 horas, precedidas de dolor tipo clico que cede posterior a la defecacin, adems de tenesmo rectal. Llama la atencin que este cuadro inicio hace dos semanas, al principio solo existi disminucin de la consistencia de la materia fecal, hasta que, alrededor del cuarto da, solo haba evacuaciones lquidas. Afebril y sin vmitos. La exploracin fsica demuestra mucosas orales humectadas. Dolor a la palpacin de todo el abdomen, sin rebote, con ruidos intestinales intensos y abundantes. Su presin arterial es de 100/70. El diagnstico ms probable es:

a) b) c) d)

Diarrea crnica. Colitis por parsitos. Enfermedad intestinal inflamatoria. Diarrea aguda

Definicin:

Se considera diarrea aguda a la presencia de heces lquidas o acuosas, generalmente en nmero mayor de tres en 24 horas y que duran menos de 14 das; la disminucin de la consistencia es ms importante que la frecuencia.1 El nmero de evacuaciones intestinales hechas en un da vara segn la dieta y la edad de la persona. Los lactantes alimentados al seno materno tienen evacuaciones intestinales blandas frecuentes; sto no es diarrea.

Epidemiologa

La Organizacin Mundial de la Salud estima que cada ao se presentan 1,300 millones de episodios de diarrea en nios menores de cinco aos en pases en desarrollo (frica, Asia, excluida China, y Amrica Latina), que ocasionan 4 millones de muertes, relacionadas en el 50-70% con deshidratacin, lo que las ubica dentro de las principales causas de defuncin en estos pases. La mayora de los nios que sobreviven quedan con algn grado de desnutricin y los desnutridos, no slo padecen con mayor frecuencia de diarrea, sino que los episodios son ms graves.3 El tercer gran problema asociado a las diarreas, en nios mayores, es el ausentismo escolar o laboral.

Etiopatognia

Los agentes etiolgicos ms frecuentes son, en orden decreciente, virus, bacterias y parsitos. Los virus son la causa principal de las diarreas deshidratantes en nios menores de dos aos, siendo los rotavirus del grupo A, serotipos G1 y G3, los responsables de la mayora de los episodios.8 La diarrea osmtica que ocasionan se debe a que lesionan en forma focal las clulas de las vellosidades del intestino delgado, disminuyendo la produccin de las enzimas encargadas de la absorcin de la lactosa, entre otros disacridos, lo que aumenta la osmolaridad en la luz intestinal y produce mayor secrecin de agua que se pierde a travs de las heces. Sin embargo, las clulas de las criptas encargadas de reparar las vellosidades lesionadas, migran para substituirlas en un periodo de 24 a 72 horas, con lo que desaparece la diarrea.9 Las enterobacterias, como Escherichia coli, Salmonella sp, Shigella, Campylobacter jejuni y Vibrio cholerae 01, producen diarrea a travs de diversos mecanismos: 1. Liberacin de enterotoxinas (V. cholerae 01, E. coli enterotoxignica) que estimulan la adenilciclasa y aumentan la secrecin intestinal de agua, sodio y cloro; 2. Enteroinvasin (E. coli enterohemorrgica) con disolucin de la mucosa y del borde en cepillo y 3. Proliferacin intracelular, previa invasin de la mucosa ( Shigella) con aparicin de sangre en las evacuaciones, paso de microorganismos a la circulacin sangunea (bacteremia) y algunas veces sepsis. Shigella dysenteriae, produce adems enterotoxinas que estimulan la secrecin de agua y sodio en el intestino delgado y neurotoxinas que causan manifestaciones neurolgicas, desde convulsiones hasta estado de coma. En los dos ltimos decenios ha sido posible documentar el papel de otros patgenos como Campylobacter y Yersinia. Asimismo, se ha avanzado en los mecanismos fisio-patognicos de las infecciones producidas por algunos tipos de Escherichia coli, Clostridium difficile, Salmonella y

Aeromonas hydrophila.9

Algunos parsitos producen diarrea sanguinolenta (Entamoeba histolytica) o diarrea prolongada (Giardia lamblia). El mecanismo de produccin de diarrea es a travs de enteroinvasin (E. histolytica) o enteroadhesin (G. lamblia), en donde el dao ms grave es de carcter nutricional. El reservorio de los agentes de las diarreas infecciosas es el hombre, exceptuando el caso de Salmonella que es de origen animal. La fuente de infeccin es la materia fecal del hombre infectado, sintomtico o asintomtico, siendo este ltimo ms peligroso ya que no

presenta datos clnicos que permitan identificarlo; en el caso de algunos virus, las secreciones nasofarngeas pueden ser el origen. La infeccin genera inmunidad especfica, la cual es de duracin prolongada en la etiologa viral y ms corta en la bacteriana. El mecanismo de transmisin clsico es el ciclo ano-boca, entendido ste como la salida del agente infeccioso con la materia fecal y su ingestin, casi siempre a travs de las manos contaminadas o de fomites. Otro mecanismo de transmisin frecuente es la ingestin de alimentos contaminados o de agua, otras bebidas o hielo, no sometidos a algn proceso de purificacin. Algunos factores de riesgo en el husped son: el estado nutricional (crculo vicioso diarrea-desnutricin-diarrea); enfermedades previas de tipo anergizante (sarampin); ablactacin temprana o ausencia de alimentacin al pecho materno; saneamiento deficiente (carencia de agua potable, fecalismo); falta de educacin y hbitos higinicos; ignorancia o patrones culturales adversos (suspensin de alimentos, no aceptacin de medidas sanitarias y de manejo oportuno del paciente), y econmicos (limitantes a la incorporacin de obras de saneamiento, adquisicin de nutrientes y bsqueda de atencin mdica).

Referencias 1. World Health Organization. A manual for the treatment of acute diarrhoea for use by physicians and other senior health workers. Geneve: Program for Control of Diarrhoeal Diseases, World Health Organization, WHO/CDD/SER/80.2 Rev,2:1990. 2. World Health Organization. Division of Diarrhoeal and Acute Respiratory Disease Control. 25 years of ORS: Joint VMO/ ICDDR,B Consultative Meeting on ORS Formulation. Dhaka, Bangladesh, 10-12 December 1994. CDR/CDD/95.2 3. Seplveda J, Willet W, Muoz A. Malnutrition and diarrhea. Alongitudinal study among urban mexican children. Am J Epidemiol 1988; 127: 365-376. 4. Mota HF, Tapia CR, Welti C, Franco A, Gmez UJ, Garrido MT. Manejo de la enfermedad diarreica en el hogar, en algunas regiones de Mxico. Bol Med Hosp Infant Mex 1993; 50: 367-75. 5. Encuesta de manejo efectivo de casos de diarrea en el hogar. Direccin General de Epidemiologa, SSA. Consejo Nacional para el Control de las Enfermedades Diarreicas. Mxico, 1993 (Documento interno). 6. Glass RI, Lew JF, Gangarosa RE, Lebaron CW, Ho MS. Estimates of morbidity and mortality rates for diarrheal disease in American children. J Pediatr 1991; 118: 527-33

48.- Masculino de 35 aos con datos clnicos altamente sugestivos para diagnstico de leo mecnico simple, los datos radiogrficos de esta patologa son: a) Retencin de contenido en colon, sin niveles hidroaereos b) Liquido y aire libres en cavidad peritoneal

c) Niveles hidroareos a diferentes alturas, con dilatacin de asas e) Asas fijas y lisas, con niveles largos

ILEO MECANICO A/ Extraluminal Adherencias postquirrgicas (Causa ms frecuente 35-40%) Hernias externas (inguinales, crurales, umbilicales, laparotmicas, etc.) Hernias internas Torsiones Vlvulos Invaginaciones Efecto masa extraluminal (tumoracin, masa inflamatoria abceso) B/ Parietal Neoplasias Alteraciones congnitas (atresias, estenosis, duplicaciones, etc.) Procesos inflamatorios (Crohn, postradiacin, etc.) C/ Intraluminal Ileo biliar Bezoar Parasitosis Cuerpo extrao Impactacin fecal Tumoraciones leo mecnico Los signos radiolgicos de una obstruccin intestinal simple son: Asas distendidas proximales por retencin de lquido y gas. Niveles hidroareos. Reduccin o ausencia de gas y materia fecal en colon. Ante el hallazgo de gas en colon se descarta la existencia de una obstruccin de intestino delgado.

BIBLIOGRAFA 1) Di Lorenzo C. Pseudo-obstrucction. Current approaches. Gastroenterology 1999;116:980987. 2) Stanghellini V, Cammilleri M, Malagelada JR.Chronic idiopatic intestinal pseudobstruction: clinical and intestinal manometric findings. Gut 1987; 28: 5-12. 3) Scolapio JS, Ukleja A, Bouras EP et al. Nutritional management of chronic intestinal pseudo-obstruction. J Clin Gastroenterol 1991; 28:306-312. 4) Malagelada JR, Distrutti E. Managementof gastrointestinal motility disorders. A practical guide to drug selection and appropiate ancillary measures. Drugs 1996;52:494506.

49.- Recin nacido que presenta una conjuntivitis neonatal, para indicar el tratamiento usted relaciona los siguientes grmenes por ser los de mayor frecuencia causantes de esta enfermedad: a) b) c) d) Clamidia, treponema, gonococo Estreptococo grupo A, listeria Clamidia, gonococo, estafilococo aureus Listeria, pseudomonas

Las conjuntivitis neonatales (CN) siguen siendo uno de los motivos ms frecuentes de consulta, pudiendo llegar a comprometer seriamente la capacidad visual del beb, conduciendo incluso a la ceguera. La incidencia de esta enfermedad oscila entre 1,6%-12% en el primer mes de vida. Las CN pueden ser de origen infeccioso o qumico. Las CN de origen infeccioso pueden ser adquiridas durante la gestacin, durante el parto por contacto con la secrecin uretrovaginal, o bien en el post-parto y el mbito de convivencia diaria. Las CN que se manifiestan entre las 24 y las 48 horas del nacimiento son de origen qumico, debidas a la profilaxis efectuada por instilacin de solucin de nitrato de plata en el saco conjuntival del neonato, para prevenir la proliferacin de Neisseria gonorrhoeae.

Se debe tener presente la adquisicin de la infeccin perinatal a partir del ambiente en que se halla el beb. La etiologa ms frecuente en este caso corresponde a Staphylococcus aureus, Haemophilus influenzae y Streptococcus pneumoniae). En las ltimas dcadas, Chlamydia trachomatis ha resultado ser el microorganismo prevalente entre los agentes causales de enfermedades de transmisin sexual, con una prevalencia del 2 al 35% en embarazadas. La transmisin vertical se da entre el 60 y 70% en los hijos de madres infectadas .El riesgo de adquirir conjuntivitis en estos bebs es del 18 al 60%, mientras que el de neumona es del 10 al 25%. En el caso de N. gonorrhoeae , la infeccin ocular puede complicar el cuadro con ceguera y artritis. Desde el ao 1995, en nuestro hospital se lleva a cabo en forma rutinaria la bsqueda de grmenes comunes (GC) y C. trachomatis en todo recin nacido con conjuntivitis.

Bibliografa: Krachmer, Manis, Holland. Cornea and External Disease: Clinical Diagnosis and Management, Vol II. Mosby, 1997. Yanoff, Ophtalmology. Mosby, 1999.

50.- Mujer de 74 aos acude a consulta externa por referir debilidad general y apata, aadindose en la ltima semana disnea progresiva hasta ser de pequeos esfuerzos.

Antecedentes: cardiopata hipertensiva con funcin sistlica conservada, en los ltimos 2 aos ha tenido 3 episodios de fibrilacin paroxstica cardiovertidos elctricamente. Durante este tiempo ha recibido diversos tratamientos que incluan algunos de los siguientes frmacos: propafenona, amiodarona, digoxina, diltiacem y captopril. El ECG muestra fibrilacin auricular con frecuencia ventricular a 130 lpm, RX de trax cardiomegalia signos de congestin pulmonar y el estudio de funcin tiroidea una T4 libre elevada con una TSH indetectable. El medicamento responsable de la sintomatologa de esta paciente es: a) Propafenona. b) Digoxina. c) Amiodarona. d) Diltiacem.

Alteraciones de lab por amiodarona: Hepticos (1,2%) Elevacin del ASAT y ALAT 2-3 veces valor normal Elevaciones persistentes de LDH y Fosfatasa alcalina Gstricos (4,2%) Naseas. Vmitos. Estreimiento Pulmonares (1,9%) * Sntomas respiratorios y nuevas alteraciones en Rx de trax > 30% de reduccin en difusin pulmonar. (PFR) Tiroides (3,7%) Hipertiroidismo o hipotiroidismo clnico Cambios en la funcin tiroidea (TSH, T4, T3) que requiere medicacin Neurolgicos (4,6%) Temblor. Ataxia. Disestesias Insomnio Piel (2,3%) Fotosensibilidad; coloracin azul/griscea (piel) Ojos (1,5%) Alteraciones visuales. Halos visuales Visin nocturna borrosa. Depsitos corneales en la lmpara de hendidura. Bradicardia/Conduccin (3,3%) Bradicardia sintomtica. Bloqueo A-V de 3er grado Bradicardia < 50 lpm asintomtica. Bloqueo A-V de 1er y 2 grado Supresin del frmaco (23%) Por efectos adversos no tolerados, decisin del paciente o no cumplimentacin de prescripcin farmacolgica (*) Se relaciona con la dosis (ms frecuente en dosis >400 mg/da) y duracin del tratamiento.

Volumen 13, Nmero 4 Octubre - Diciembre 2002 pp 149 152 Cardiologa Trabajo de investigacin * Cardilogo, Ex-Residente Hospital de Especialidades, Centro Mdico Nacional La Raza, IMSS. Hospital de la Fe, San Miguel Allende, Gto. ** Jefe del Servicio de Cardiologa, Hospital de Especialidades Centro Mdico Nacional La Raza, IMSS.

51.- Masculino de 40 aos, acude a consulta por presentar de manera espontnea una erupcin de lesiones pruriginosas, habonosas, diseminadas por toda la superficie corporal, de evolucin fugaz as como una amplia placa edematosa en hemicara izquierda. El diagnstico ms probable es: a) b) c) d) Angioedema hereditario. Urticaria aguda. Erisipela. Vasculitis.

La urticaria se define como un sndrome reaccional de piel y mucosas caracterizado por edema y ronchas pruriginosas ocasionadas por edema vasomotor transitorio y circunscrito de la dermis que dura algunas horas; puede ser recidivante y de origen inmunolgico, no inmunolgico o desconocido.1 La lesin elemental es una ppula edematosa drmica (roncha).2 El angioedema se manifiesta tpicamente como un edema asimtrico causado por la presencia de plasma dentro de tejido celular subcutneo y mucosas.1 Clasificacin La urticaria se puede clasificar de acuerdo con diferentes parmetros: 1) segn la evolucin: en aguda o crnica; 2) segn el cuadro clnico: en urticaria ordinaria (urticaria propiamente dicha), urticaria fsica (por estmulo detonador), urticaria por contacto (inducida por un contacto qumico o biolgico) y angioedema (sin ronchas), en el cual el espectro de las manifestaciones clnicas de los diferentes tipos es muy amplio; y 3) segn el mecanismo potencial de su desarrollo: inmunolgico, no inmunolgico, mediada por el complemento, o bien urticaria autoinmune.3 Clasificacin por evolucin La urticaria aguda se define tradicionalmente por la presencia de ronchas de forma espontnea, casi la mayora de los das, por menos de seis semanas.4 La urticaria crnica se define como la presencia de ronchas de forma espontnea por ms de seis semanas, diariamente o casi la mayora de los das de la semana. BIBLIOGRAFIA 1- Yadav S, Upadhyay A, Bajaj A. Chronic urticaria: An overview. IJD [en lnea] 2006 [fecha de acceso 4 de marzo de 2007];51-3:171-177. Disponible en:

2- Woscoff A, Kaminsky A, Marini M, Allevato M. Dermatologa en Medicina Interna. Buenos Aires, Edicin de los Autores, 2003: 24-27. 3- Criado PR, Fachini Jardim Criado R, Maruta C, Costa Martins JE, Rivitti E. Urticaria. An.Bras.Dermatol 2005,v.80 n.6: 183-185.

52.- Female patient 23, who will have an operation with a history of hepatitis. Which of the following anesthetics are contraindicated in your case?

a) b) c) d)

Lidocaine Fentanyl Halothane Ketamine

El halotano reduce el flujo sanguneo esplcnico y heptico como consecuencia de una reduccin de la presin de riego. Puede producir necrosis heptica fulminate, que se caracteriza por fiebre, anorexia, nusea y vmito que pueden durar varios das despus de la anestesia y acompaarse por un exantema y eosinofilia perifrica. Este sndrome recibe el nombre de hepatitis por halotano. Hardman J, Limbird L. Goodman and Gilman Las bases farmacolgicas de la teraputica. Mc Graw Hill 2002. 361.

53.- Se trata de paciente femenino que cursa con diagnstico de Salmonella Typhy. Cul de los siguientes cultivos permite aislar con mayor frecuencia ste agente causal?

a) b) c) d)

Hemocultivo Coprocultivo Mielocultivo Urocultivo

DIAGNOSTICO DE LABORATORIO Dadas las variadas manifestaciones clnicas de las salmonelosis, la confirmacin del diagnstico de estas infecciones, requiere de mtodos microbiolgicos que permitan el aislamiento o identificacin del agente causal o de pruebas serolgicas que facilitan reconocer anticuerpos especficos presentes en el suero de los pacientes. Hemocultivo: es el procedimiento de eleccin, cuando se realiza apropiadamente y en medios selectivos a base de bilis. Coincidiendo con la fisiopatologa de la infeccin, son positivos especialmente durante la primera semana de la infeccin; se calcula que al final de la tercera semana de positividad solamente alcanza un 50%.

Mielocultivo: el cultivo del aspirado de mdula sea se considera como el mejor mtodo para el aislamiento de salmonella en los pacientes con fiebre tifoidea y paratifoidea. Aunque el procedimiento produce una molestia transitoria, en general es bien tolerado y los cultivos son ms rpidamente positivos. Se recomienda sea practicado por personal con experiencia. Pueden ser positivos an cuando los hemocultivos sean negativos. Coprocultivo: puede ser positivo desde el comienzo de la infeccin, aunque su mxima positividad en la infeccin aguda, se observa durante la tercera semana. Es particularmente til para el control postratamiento de los pacientes y para detectar los portadores crnicos. Cultivo de bilis duodenal: obtenido por aspiracin o utilizando la tcnica que lleva un dispositivo en cpsulas de gelatina. No es superior al hemocultivo y con certeza no supera a la asociacin del hemocultivo con el coprocultivo. Urocultivo: su valor diagnstico es muy limitado pues la bacteriuria no es continua. Su mxima positividad est en la tercera semana. La Salmonella tambin puede ser aislada de otros productos como las manchas rosadas o reoseolas tficas, de la secrecin bronquial, del lquido articular, etc. DIAGNOSTICO SEROLOGICO Reaccin de seroaglutinacin (Widal): es de poco valor como prueba diagnstica. En la infeccin no tratada slo cerca del 50% de los pacientes pueden tener un aumento significativo de las aglutininas contra el antgeno "O", en algn momento de la enfermedad. Las aglutininas contra el antgeno "H" no tienen valor diagnstico aunque puedan observarse ttulos elevados de ellas. En muchos casos de fiebre tifoidea no hay elevacin de los ttulos de aglutininas durante el curso de la infeccin y en ocasiones se pueden observar elevaciones no especficas, debido a reacciones cruzadas. Diagnstico inmunoenzimtico: la deteccin de anticuerpos IgM e IgG contra el lipopolisacarido por tcnica ELISA an no est disponible para uso rutinario. Con fines de investigacin se han utilizado otras pruebas dentro de las cuales estn la reaccin de polimerasa en cadena (PCR), las pruebas de fagotipificacin, las de susceptibilidad antimicrobiana y la investigacin del perfil plasmdico de algunas cepas. En los estudios epidemiolgicos se usan las pruebas de fagotipificacin, de susceptibilidad contra los antimicrobianos y el perfil plasmdico, las cuales han demostrado ser tiles y complementarias para el estudio de cepas aisladas de alimentos, o de aguas contaminadas, y en brotes de salmonelosis en los cuales se requiere establecer una fuente comn de infeccin.

LECTURAS RECOMENDADAS

Borrego JJ, Castro D, Jimnez Notorio M, Luque A, Rodrguez Avial C, Picazo JJ. Comparison of epidemiological markers of salmonella strains isolated from differents sources in Spain. J Clin Microbiol 30: 3058, 1992. De Los Rios O, Restrepo J, Carvajal CD. Salmonelosis: Revicin de conceptos. Comportamiento epidemiolgico en Antioquia. Bol Epidenmol Antioquia 7:19 Edelman R, Levine MM. Sumary of an international workshop on typhoid fever. Rev Inf Dis 8:329, 1996. Prada G. Infecciones por especies de Salmonella. En: Medicina Interna. Chalem, Escandn, Campos, Esguerra editores. Fundacin Instituto de Reumatologa e Inmunologa. Editorial Presencia. Santaf de Bogot. Tacket C. Molecular epidemiology of salmonella. Epidemiologic Reviews 11:99.

54.- Mujer de 30 aos de edad, que acude a consulta por presentar dolor en el recto, sin encontrarse una causa orgnica despus de mltiples exploraciones mdicas. Adems refiere que desde hace 4 aos ha presentado vmitos, dolor abdominal generalizado, palpitaciones, mareos, disfagia, visin borrosa, dolor en los miembros inferiores, dismenorrea y dispareunia. Se observa triste y ansiosa. El diagnstico ms probable es:

a) b) c) d)

Trastorno de somatizacin Trastorno de ansiedad crnico Depresin crnica con somatizacin Trastorno de conversin

Lpez- Ibor J J, Valds M M. Manual diagnstico y estadstico de los trastornos mentales (DSM- IV). Masson 2005. 545 574. El Trastorno de somatizacin se caracteriza por la presencia de muchos sntomas somticos que no pueden explicarse por los hallazgos fsicos o de laboratorio. Comienza antes de los 30 aos, puede perdurar durante aos, es crnico y va asociado a malestar psicolgico, a un deterioro del funcionamiento social y laboral y a la bsqueda excesiva de

ayuda mdica. Para hacer el diagnstico deben presentarse dolor en cuatro zonas del cuerpo, dos sntomas gastrointestinales, un sntoma sexual y un sntoma pseudoneurolgico. La ansiedad y la depresin son las patologas psiquitricas ms prevalentes.

55.- Hombre de 67 aos, el cual ingresa al servicio de urgencias quejndose de dolor torcico de 3 horas de evolucin, con diaforesis, palidez. El EKG muestra elevacin del segmento ST en la cara inferior. Cual de las siguientes es una contraindicacin absoluta para trombolisis:

a) Antecedente de evento vascular cerebral hemorrgico b) Enfermedad convulsiva crnica c) Enfermedad de Parkinson temprana sin demencia d) Historia de traumatismo hace 1 ao en accidente vehicular

CONTRAINDICACIONES A LA TROMBOLISIS Absolutas Antecedente de AVE hemorrgico - Aneurisma disecante. - Ditesis hemorrgica. - Hemorragia digestiva en el mes precedente. - Ciruga o traumatismo reciente (ltimas 3 semanas). Relativas - Tratamiento anticoagulante. - Hipertensin arterial refractaria > 180 /110 mmHg. - Maniobras de resucitacin cardiopulmonar prolongadas. - Embarazo. - Puncin de vaso en sitio no compresible (vena subclavia). - AVE isqumico en los ltimos 6 meses.

Como se explic previamente, a mayor precocidad de terapia, se logra una mayor reduccin de la mortalidad. Por ejemplo, si logramos reperfundir a un paciente con menos de 3 horas de evolucin, la mortalidad cae en un 50% en comparacin con la terapia realizada en un paciente con 12 horas de evolucin, en el cual la mortalidad cae slo en un 10%. Otro factor que influye en la disminucin de la mortalidad en los pacientes tratados con trombolisis es la extensin del infarto, ya que a mayor isquemia hay mayor disminucin de mortalidad.

BIBLIOGRAFIA 1. Overview of the management of suspected myocardial infarction. Reeder G.S., Kennedy H.S., Rosenson R.S. UpToDate v9.1, 2001 2. Diagnosis of acute myocardial infarction with biomarkers of cardiac injury. Jaffe Allan S. UpToDate v9.1, 2001 3. Braunwald: Heart Disease: A Textbook of Cardiovascular Medicine, 6th ed., 1114-1207. W. B. Saunders Company, 2001 4. Rakel: Conn's Current Therapy 2001, 53rd ed., 335-348, W. B. Saunders Company, 2001 5. Rosen: Emergency Medicine: Concepts and Clinical Practice, 4th ed., 1655-1979. MosbyYear Book, Inc., 1998 6. Futterman L.G., Lemberg L. Update on management of acute myocardial infarction: facilitated percutaneous coronary intervention. American journal of critical, 2000;9:70-6. 7. Wright R.S., Kopecky S.L., Reeder G.S. Update on intravenous fibrinolytic therapy for acute myocardial infarction. Mayo Clinic proceedings, 2000;75:1185-91.

56.- El signo clnico que indica irritacin del nervio citico como componente de una lumbalgia se denomina:

a) b) c) d)

Signo Signo Signo Signo

de Galeazzi de McMurray de Lassegue de Filkestein

La lumbalgia es una patologa muy frecuente en nuestro medio, con una enorme repercusin sanitaria y sociolaboral. Con la edad, se inicia una cascada degenerativa en la columna lumbar, que comienza en el disco intervertebral, continuando por las facetas articulares y dems elementos vertebrales. Esta degeneracin del raquis forma parte del envejecimiento normal del individuo, aunque en ocasiones puede causar dolor y/o alteraciones neurolgicas. Para comprender la fisiopatologa del dolor lumbar, ser preciso conocer que la inervacin de la columna lumbar se hace fundamentalmente por tres ramos nerviosos: ramo dorsal de los nervios espinales lumbares (o rami dorsal), nervio sinuvertebral de Luschka y ramos ventrales de la cadena simptica. Existen dos tipos de patrones de dolor en la columna lumbar: el dolor irradiado y el dolor referido. La cascada de la degeneracin consta en tres estadios. El primer estadio sera la disfuncin. El anillo fibroso se fisura y pierde la capacidad de contener al ncleo pulposo. Esto ocasiona primero el sndrome de disrupcin discal y, si el ncleo supera el contorno del annulus, las hernias discales. El segundo estadio de Kirkaldy-Willis es el de inestabilidad. En este estadio la movilidad en el segmento mvil aumenta de forma patolgica.

Maniobra de Lassgue: Es la ms importante, es casi patognomnica. Con el paciente en decbito supino se levanta la pierna extendida. Se considera positiva cuando aparece un dolor intenso a nivel lumbar, en la pierna o en ambos niveles, entre los 30-75 de flexin en la cadera, debido al estiramiento del nervio citico. Pasados los 70

puede aparecer un falso positivo (dolor tambin en sujetos sanos) por la distensin de los isquiotibales. Para descartarlo realizaremos otras maniobras diagnsticas.

Referencias: 1. Andersson GBJ. Epidemiologic features of chronic low-back pain. Lancet. 1999;354:5815. 2. Estudio EPISER. Sociedad Espaola de Reumatologa. 2000. 3. Herrera Rodrguez A, Rodrguez Vela J. Estenosis de canal lumbar. Rev Ortop Traumatol. 2002;4:351-72. 4. MacGregor AJ, Andrew T, Sambrook PN, Spector TD. Structural, psychological, and genetic influences on low back and neck pain: a study of adult female twins. Arthritis Rheum. 2004;51:160-7. 5. Kim KS, Yoon ST, Park JS, Li J, Park MS, Hutton WC. Inhibition of proteoglycan and type II collagen synthesis of disc nucleus cells by nicotine. J Neurosurg Spine. 2003;99:291-7. 6. Fujiwara A, Tamai K, Yamato M, An HS, Yoshida H, Saotome K, et al. The relationship between facet joint osteoarthritis and disc degeneration of the lumbar spine: an MRI study. Eur Spine J. 1999;8:396-401. 7. Eyre DR, Muir H. Types I and II collagens in intervertebral disc. Interchanging radialdistributions in annulus fibrosus. Biochem J. 1976;157:267-70.56.-5

57.- Hombre de 52 aos que presenta sbitamente dolor intenso en primer ortejo de pie derecho durante la noche posterior, tras ingesta de alcohol. Con los datos clnicos anteriores usted pensara que el paciente presenta:

a) b) c) d)

Hiperuricemia crnica sintomtica. Sndrome de reiter. Monoartritis infecciosa. Artritis gotosa.

Las causas que generan hiperuricemia son mltiples, pero en general podemos dividirlo en tres grupos:1 Hiperuricemia idioptica (10-15%) que representa los errores innatos del metabolismo, padecimientos caracterizados por un incremento en la sntesis de purinas y sobreproduccin de cido rico; 2 Por incremento del recambio metablico y que genera; un exceso de cido rico srico, observado en los sndromes mieloproliferativos, neoplasias, etctera, y 3 grupo conformado por padecimientos o condiciones que producen baja excrecin renal de cido rico. La acidosis metablica, insuficiencia renal, enfermedades

metablicas endocrinolgicas y el uso de frmacos, son las causas ms comunes de esta situacin. Factores de Riesgo: Todas las causas de hiperuricemia son un factor de riesgo para la gota. 1) La obesidad o el aumento o prdida repentinos de peso; 2) mayores de 40 aos; 3) sexo masculino; 4) Miembros de la familia con gota;5) Diurticos, tales como el hidroclorotiazido; 6) Algunas medicinas, como la aspirina; 7) Una diera rica en cristales de nitrgeno; 8) Consumo de alcohol; 9) Algunos tipos de cncer o tratamientos contra el cncer (por ejemplo, medicinas citotxicas); 10) Medicinas (tales como los que combaten la apopleja y otros); 11) Deshidratacin;12) Hipercolesterolemia;13) Enfermedad renal; 12) Desordenes endocrinos, como el hipotiroidismo y el hiperparatiroidismo.

Como es una enfermedad inflamatoria tiene un comportamiento episdico, de distribucin universal, predomina en el hombre con el 90% de los casos y 10% en mujeres, de preferencia posmenopusicas. De inicio mas frecuente entra la 4 y 6 dcada. Con diferentes estadios conocidos como hiperuricemia sintomtica, artritis gotosa aguda, periodo intercrtico y gota crnica tofcea. Entonces, de acuerdo a los estadios de la gota podemos mencionar: A Hiperuricemia asintomtica B Artritis Gotosa: Es la mas comn y se caracteriza por dolor agudo, intenso, que afecta a una ms articulaciones de predominio monoarticularlas, de acuerdo a orden de frecuencia son; el dedo gordo del pie tobillo, tarso, rodilla y mueca. C Periodo Intercrtico: intervalo entre un ataque y otro. El paciente queda libre de sntomas hasta la presentacin del siguiente ataque de artritis, la duracin del periodo asintomtico es variable. Posteriormente los periodos asintomticos se van acortando y el paciente evoluciona hacia la cronicidad. Gota Tofcea Crnica: Caracterizada por periodos recidivantes de artritis con depsitos de uratos conocidos como tofos, tumoraciones sobre la articulacin que pueden estar excretando un material calcreo.

Rev Pacea Med Fam 2006; 3(3): 2-5 Dra. Nilsa Selaya C. Dr. Cesar Rabaza M. Dr. Rafael Castillo R.

58.- Masculino de 53 aos de edad presenta una historia de 2 das de hemoptisis. Reporta un inicio agudo de 8 episodios de tos con sangre fresca (aprox una cucharadita de sangre por cada episodio). No reporta otros sntomas, excepto por una tos productiva de 5-10ml de esputo cada maana. Tiene una historia de EPOC, para lo cual toma broncodilatadores. Ha fumado 30 cigarrillos diariamente por los ltimos 30 aos. El examen fsico es normal, y una RX limpia. De los siguientes cual es la causa ms probable de la hemoptisis?

a) Bronquiectasias b) Carcinoma broncognico

c) Tb pulmonar d) Bronquitis crnica

La EPOC puede presentar episodios de hemoptisis con placa radiogrfica normal, aunque no es muy frecuente; las otras causas como TB pulmonar y Cncer pulmonar as como las bronquiectasias presentan lesiones en la radiografa que hace sospechar en estos diagnsticos y en el presente caso no se mencionan lesiones en la Rx, la deficiencia de alfa 1 provoca la presencia de enfisema pulmonar la cual igualmente debera tener una placa con alteraciones. 1.- Fishman AP, , Fishman JA, Grippi MA, Kaisser LR, Seor RM. Pulmonary Diseases and disorder. 3a. Edicin McGraw-Hill, EUA, 2006. 2.- Fraser, R ; Neil, C; Par, P; Diseases of the Chest, Third Edition, Editorial Elsevier, 2005. 3.- Murray and Nadels; Textbook Respiratory Medicine, Vol 1-2, Elsevier editorial, 2005.

59.- La frecuencia es de 1/20,000, bajo peso al nacimiento, LPH (Labio paladar hendido) microftalmia, holoprosencefalia, aplasia cutis vertex, riones poliqusticos, malrotacin del colon, y en nias puede existir tero bicorne, los datos anteriores son compatibles con la siguiente alteracin cromosmica:

a) b) c) d)

Trisoma 13 Trisoma 21 Trisoma 18 Trisoma 11

La trisoma 13 representa la tercera aneuploida autosmica viable ms frecuente en la especie humana, slo superada en frecuencia por la trisoma 21 y trisoma 181. Aunque su etiologa es an desconocida, estudios citogenticos en individuos afectados han podido determinar que aproximadamente un 75% de los casos corresponden a no-disyuncin meitica, un 20% a translocaciones y un 5% a mosaicismo debido a nodisyuncin postcigtica. La expresin fenotpica de la trisoma 13 es caracterstica y consiste en anomalas faciales, esquelticas y del sistema nervioso central, siendo tambin frecuentes las malformaciones estructurales del aparato cardiovascular, genitourinario y gastrointestinal (Tabla I).1 La mayora de los defectos estructurales asociados a trisoma 13 pueden ser identificados prenatalmente por ultrasonografa.2-6 Esto permite ofrecer estudio citogentico prenatal en casos seleccionados, establecer el diagnstico definitivo y manejar el embarazo afectado en forma racional, ya que esta anomala se asocia a una mortalidad perinatal cercana al 100%.

60.- Se trata de masculino de 22 aos, sin antecedentes patolgicos de importancia, inicia un cuadro de febrcula, dolor articular, tos seca persistente y astenia de dos semanas de evolucin. En el ltimo mes, sus dos hermanos menores que l han presentado consecutivamente un cuadro similar, que se ha autolimitado de forma progresiva. Tras practicrsele una radiografa de trax, el mdico le ha diagnosticado de neumona atpica. El agente etiolgico ms probable en este caso es: a) b) c) d) Legionella pneumophila. Coxiella burnetti (fiebre Q). Haemophilus influenzae Mycoplasma pneumoniae

Diversos estudios sugieren que esta enfermedad comprende entre el 15 y el 50% de todos los casos de neumonas en adultos e incluso ms en los nios en edad escolar.

La neumona por micoplasma es un tipo de neumona atpica y es causada por la bacteria M. pneumoniae. Este tipo de neumona generalmente afecta a personas menores de 40 aos.

Las personas que se encuentran en mayor riesgo de adquirir neumona por micoplasma incluyen aquellos que viven o trabajan en reas de hacinamiento como escuelas y hogares de personas abandonadas, aunque muchas personas que la contraen no presentan ningn factor de riesgo que se pueda identificar. Cuadro clnico

forma de neumona que, por sus peculiares caractersticas de presentacin clnico radiolgica se denomina neumona atpica primaria. Los sntomas se presentan de manera gradual en varios das, y consisten en fiebre, tos no productiva, cefalea y mialgias. A menudo, se acompaa de faringitis, rinitis, otitis y traqueobronquitis. La exploracin fsica se caracteriza por la parquedad de sntomas, auscultndose ligeros subcrepitantes, aunque los pacientes pueden presentar crepitantes francos, roncus y sibilantes. En la radiografa de trax se observan infiltrados retculonodulillares parahiliares o peribronquiales que pueden ser uni o bilaterales. Puede observarse la presencia de un pequeo derrame pleural en uno de cada cuatro o cinco pacientes. En los anlisis complementarios suele encontrarse una discreta leucocitosis en un 30% de los pacientes. Los nios con alteraciones inmunolgicas como la anemia de clulas falciformes, con anesplenia funcional o con sndrome de Down, pueden desarrollar una infeccin respiratoria grave y de evolucin fulminante. La hipogammaglobulinemia es tambin un factor de riesgo para las infecciones del tracto respiratorio y de sus complicaciones a nivel articular. Se han descrito otro tipo de infecciones, aunque en menor frecuencia. Generalmente, acompaan a un cuadro respiratorio, pero pueden aparecer en ausencia absoluta de sntomas de esta localizacin. En la tabla 1 se relacionan las principales manifestaciones y complicaciones extrapulmonares.

Mycoplasma pneumoniae produce infecciones del aparato respiratorio, principalmente en

Tabla 1. Infecciones extrapulmonares por Mycoplasma pneumoniae.

Sistema nervioso central

Meningoencefalitis, neuritis ptica, parlisis nervios craneales, parlisis ascendente (Sndrome Guillain-Barr), ataxia y psicosis Piel Erupcin eritematosa papular o vesicular. Sndrome de Stevens-Johnson Articular Mialgias, artralgias y poliartropatias Artritis sptica (especialmente en caso de hipogammaglobulinemia) Cardiaca Pericarditis, miocarditis y derrame pericrdico Sistema hematopoytico Anemia hemoltica asociada con aglutininas frias Prpura trombtica trombocitopnica Renal Glomerulonefritis, nefritis tubulointersticial, nefropata IgA Gastrointestinal Vmitos, diarreas y hepatitis colestsica. Pancreatitis Otros Otitis externa, otitis media y miringitis Rabdomiolisis Conjuntivitis, uvetis anterior, retinitis y neuritis ptica Abscesos tubo-ovricos BIBLIOGRAFA:

WAITES KB, TALKINGTON DF. Mycoplasma pneumoniae and its role as a human patogen. Clin Microbiol Rev 2004; 17:697-728. AUSINA V, RODRIGO C. Infecciones causadas por micoplasmas. En: Farreras-Rozman (eds). Medicina Interna, 15 ed. Madrid: Elsevier Espaa SA, 2004; pp 2362-2365. TALKINGTON DF, SHOTT S, FALLON MT, SCHWARTZ SB, THACKER WL. Analysis of eight commercial enzyme immunoassay tests for detection of antibodies to Mycoplasma pneumoniae in human serum. Clin Diagn Lab Immunol 2004; 11:862-867. MICHELOW IC, OLSEN K, LOZANO J, DUFFY LB, MCCRACKEN GH, HARDY RD. Diagnostic utility and clinical significance of naso- and oropharyngeal samples used in a PCR assay to diagnose Mycoplasma pneumoniae infection in children with community-acquired pneumonia. J Clin Microbiol 2004; 42:3339-3341. TEMPLETON KE, SCHELTINGA SA, GRAFFELMAN AW, VAN SCHIE JM, CRIELAARD JW, SILLEKENS P, ET AL. Comparison and evaluation of real-time PCR, real-time nucleic acid sequencebased amplification, conventional PCR, and serology for diagnosis of Mycoplasma pneumoniae. J Clin Microbiol 2003; 41:4366-4371.

61.- Se trata de paciente femenino de 21 aos que cursa con amenorrea secundaria, la cual presenta menstruacin posterior a la administracin de progestgenos, lo ms probable es que sta paciente presente:

a) b) c) d)

Estrgenos bajos Progesterona elevada Gonadotropinas altas Estrgenos normales

La presencia de estrgenos se puede establecer de dos maneras: con la prueba de desafo con acetato de medroxiprogesterona (AMP) y el ndice de maduracin vaginal. Si hay estrgenos, luego de la administracin de 10 mg de AMP diarios por 10 das debera presentarse un sangrado por vagina. Y de esta forma corroborar los niveles de estrgenos serian normales. La ausencia de esta metrorragia sugiere una obstruccin o un hipogonadismo. La otra alternativa es realizar el ndice de madurez vaginal que es un procedimiento sencillo que puede obtenerse en el consultorio. Usando una esptula como las de Papanicolaou el gineclogo extrae una muestra de la vagina y la extiende en un portaobjeto. Este portaobjeto se procesa y se estudia en el microscopio. Un extendido maduro tendr clulas epiteliales superficiales grandes y numerosas con un ncleo rodeado por una gran cantidad de citoplasma. Por el contrario, una muestra no estrognica tendr un mayor nmero de clulas parabasales y basales con ncleo grande rodeado por escaso citoplasma. Speroff L, Glass RH, Kase NG, eds. Clinical Gynecologic Endocrinology and Infertility, 5th ed. Baltimore: Williams & Wilkins; 1994:334-335.

62.- Se trata de femenino de 29 aos de edad, la cual inicia con hiperemesis gravdica de difcil control, as como sangrado trasvaginal. Se realiza el diagnstico de mola hidatiforme. El tratamiento inicial indicado en esta patologa es? a) b) c) d) Legrado por aspiracin. Histerectoma total abdominal. Metotrexate y seguimientos radiogrficos. Legrado uterino instrumental

La enfermedad trofoblstica gestacional agrupa a diferentes entidades interrelacionadas: mola completa, generalmente diploide con origen cromosmico paterno, mola parcial generalmente triploide, tumor trofoblstico del lecho placentario y coriocarcinoma, con tendencias variables a la invasin local y a las metstasis, cuyo denominador comn es la hipersecrecin de hCG. El coriocarcinoma es diploide y proviene de ambos progenitores, excluyendo probablemente su origen directo en la mola completa. El tumor trofoblstico del lecho placentario est constituido por trofoblasto mononuclear intermedio no conteniendo vellosidades corinicas e inmunohistoquimicamente caracterizado por expresar muchas de sus clulas hPL y unas pocas hCG. Tratamiento

Hay que tratar las complicaciones como la hipermesis, anemia, hipertensin, y alteraciones electrolticas, coagulopatas, alteraciones cardio-respiratorias y preeclampsia, procediendo a evacuar la mola lo antes posible, con lo que se producirn menos malignizaciones. La evacuacin del contenido uterino se realiza mediante dilatacin, y legrado por aspiracin. Adems se pauta profilaxis antibitica y oxitcicos.3 La histerectoma, con la mola en su interior, est indicada en pacientes de edad superior a 40 aos o en mujeres con ms de tres hijos, ya que en ambos grupos se ha demostrado una mayor incidencia de malignizacin. . Tras la ciruga, se mide la concentracin de gonadotropina corinica humana para determinar si la extirpacin ha sido completa. Si es as, el valor de esta hormona vuelve a la normalidad, en unas 8 semanas, y se mantiene en esos valores. Si una mujer a la que se le ha extirpado una mola queda embarazada, es difcil interpretar un valor alto de gonadotropina corinica humana, porque podra estar causado tanto por el embarazo como por una parte de la mola que no se ha extirpado. En consecuencia, a las mujeres a las que se les ha extirpado una mola se les recomienda no quedar embarazadas durante un ao. Las molas hidatiformes benignas no necesitan quimioterapia, pero las malignas s. Los frmacos que se usan para este tratamiento son el metotrexato, la dactinomicina o una combinacin de ambos. Referencias: Silverman L, Romero Zambrano F, Saldao S. Enfermedad molar. Diagnstico, tratamiento y seguimiento, 4- Puertas A, Lpez Fernandez J et al. Enfermedad trofoblstica. Casustica del Hospital Virgen de las Nieves de Granada. Cln Invest Gin Obs 20: 98-103. 5- Enfermedad trofoblstica gestacional. Propuesta Normativa Perinatolgica y Ginecolgica de Alto Riesgo. Ministerio de Asuntos Sociales. Tucumn. 1996-1997; 19:171-183. 6- Jones. Enfermedad Trofoblstica Gestacional: qu hemos aprendido en la ltima dcada. Am J Gynecol Obstet 1990;162: 1286-1292. 7- Resmen del simposio Enfermedad Troblstica Gestacional del Segundo Congreso Nacional de AGORA, 1990. 8- Gonzalez Merlo et al. Protocolos de diagnstico y tratamiento en Obstetricia y Ginecologa, Barcelona: editorial Salvat, 7:35-45.

63.- Lactante de un ao de edad, es atendida en consulta por erupcin inflamatoria aguda del rea cubierta por el paal de aproximados 4 das de evolucin. Antecedentes: gastroenteritis hace 4 das. Exploracin fisica: se aprecian ppulas y pstulas sobre una base eritematosa, intensa, brillante, asimismo lesiones satlites que se presentan en la periferia. El cuadro clnico de ste paciente est generado por:

a) b) c) d)

Infeccion bacteriana. Contacto prolongado con orina o heces. Alergia al material del paal. Dermatitis atpica

DERMATITIS DE LA ZONA DEL PAAL. SINONIMIA. Dermatitis del paal, dermatitis amoniacal,rozadura. DEFINICION. Erupcin inflamatoria aguda del rea cubierta por el paal. EPIDEMIOLOGIA. Afecta con mayor frecuencia recin nacidos y lactantes, con un pico de incidencia a los 9 a 12 meses, aparentemente en relacin con el cambio en la dieta. Se presenta adems en nios mayores y adultos con incontinencia urinaria y/o fecal secundaria a alteraciones urolgicas o neurolgicas. La prevalencia es del 7 al 35% en poblacin peditrica, ocupando el 3 a 4 causa de consulta dermatolgica en nios. No se ha descrito diferencia racial ni en cuanto al sexo. ETIOPATOGENIA. En 1973 Koblenzer clasific la dermatitis de esta zona en tres grupos: Grupo 1. Enfermedades que se producen en forma independiente del uso de paales (epidermolisis bulosa, histiocitosis de clulas de Langerhans, etc.). Grupo 2. Enfermedades agravadas por el uso de paales (dermatitis atpica, dermatitis seborreica, psoriasis, etc.). Grupo 3. Enfermedades provocadas por el uso de paales, en nios no predispuetos y como consecuencia directa de su uso, es el ms frecuente y ocupar nuestra revisin. Lo mas caracterstico es la dermatitis de contacto por irritante primario en la cual la humedad persistente provoca maceracin de la piel; lo anterior aunado a la friccin generada por los movimientos del beb altera la funcin de barrera y permite la accin de irritantes como son: heces (proteasas, lipasas y sales biliares), orina (urea y amoniaco), jabones, lociones, cremas y productos empleados para el lavado de los paales; por lo tanto, los irritantes actan en forma secundaria en piel ya daada previamente. Por otro lado, la piel as daada, es susceptible de infeccin por Candida albicans con mayor frecuencia (50%) y mas raramente, agentes bacterianos (S aureus, estreptococos y enterobacterias). La dermatitis por contacto alrgica no es comn el rea del paal, sin embargo algunas sustancias como gomas, plsticos, neomicina y mercurio pueden actuar como sensibilizantes. CUADRO CLINICO. Existen diferentes patrones de afeccin clnica: Dermatitis eritematosa simple. Es el mas frecuente. Afecta reas convexas (en W), respetando los pliegues. Las formas mas leves se caracterizan por eritema y descamacin; el eritema puede desarrollar una apariencia vidriosa y aparecer ppulas (fig1). Variedad sifiloide ppulo-erosiva o poserosiva de Sevestre-Jaquet. Es una forma moderada de la dermatitis eritematosa simple. Se presenta sobre las superficies convexas en que se observan ppulas del color de la piel o eritematosas, generalmente menores de 5 mm de dimetro, que por accin de la friccin o maceracin se erosionan en la cpula. Pueden volverse violceas y liquenificadas (fig.2). Ulcerosa. Se caracteriza por prdida cutnea epidrmica o mas profunda, siendo la presentacin mas grave de las anteriores; surge como consecuencia de la potencia y el tiempo de exposicin a la agresin y sensibilidad individual de la piel. Dermatitis perianal. Limitada a esta rea, se manifiesta como eritema, erosiones o pstulas; es mas frecuente en el periodo neonatal. Puede deberse a cambios qumicos

(alteracin del pH en procesos diarreicos o enteropatas) o infecciosos (dermatitis estreptocccica perianal, parasitosis). COMPLICACIONES: Candidosis. Es la complicacin mas frecuente. Compromete los pliegues inguinales e interglteos y se caracteriza por ppulas y pstulas sobre una base eritematosa, intensa, brillante, con bordes netos, elevados, geogrficos y escama blanquecina; asimismo lesiones satlites constituidas por ppulas y pstulas que se presentan en la periferia. Se ha demostrado colonizacin por C albicans en la dermatitis de la zona del paal con mas de 72 horas de evolucin (45-75%), an en ausencia del cuadro clnico caracterstico. Imptigo secundario. Agregndose pstulas y costras melicricas. Granuloma glteo infantil. Ppulas y ndulos rojizos o purpricos de 0.5 a 4 cm, en regin perianal y gltea; se ha asociado a irritacin y friccin crnica, C. albicans y uso de esteroides fluorinados. Fig. 1. Dermatitis Fig. 2. Variedad sifiloide Fig. 3. Candidosis. eritematosa simple ppuloerosiva. DIAGNOSTICO. Clnico; puede solicitarse examen directo con KOH y cultivo para corroborar infeccin mictica. La biopsia se reserva a casos resistente al tratamiento, en que se sospecha otra enfermedad de base. DIAGNOSTICO DIFERENCIAL. 1.Dermatitis seborreica. Se caracteriza por placas eritematoescamosas con aspecto graso curso crnico y falta de respuesta al tratamiento convencional, que puede afectar el rea del paal, pero afectando tambin piel cabelluda, reas centrofacial y pliegues proximales de los miembros (axilas e ingles). 2.Dermatitis atpica. Cuenta con ntecedentes personales o familiares de atopia; se relaciona con afeccin simultnea o previa en cara y pliegues de flexin (poplteos y antecubitales), prurito, historia de cronicidad y recurrencia y respuesta inadecuada al tratamiento habitual. 3.Acrodermatitis enteroptica. Alteracin autonmica recesiva del transporte y absorcin de zinc; se caracteriza por placas eritematoescamosas y erosionadas, de distribucin periorificial, comprometiendo el rea perianal pero tambin perioral. El cuadro se presenta con mayor frecuencia despus del destete. Se asocia con manifestaciones extracutneas como irritabilidad, diarrea, fotofobia y conjuntivitis. 4.Psoriasis. Puede encontrarse historia familiar de psoriasis y se caracteriza por un reas eritematosas, que pueden comprometer toda el rea del paal, generalmente sin escama (por efecto queratoltico de la urea), sin mejorar con los tratamientos mencionados. Puede presentar adems afeccin en piel cabelluda y uas. 5.Histiocitosis de clulas de Langerhans. En la zona del paal se presenta como dermatitis hemorrgica o erosiva grave, generalmente perianal, que no responde al tratamiento, sin embargo, generalmente se asocian a eritema y escamas adherentes en piel cabelluda, ppulas eritematosas o parduscas en tronco y lesiones purpricas en palmas y plantas, as como manifestaciones extracutneas. TRATAMIENTO. Medidas preventivas, como son el cambio frecuente de paal, recomendndose paales desechables con geles absorbentes (alginatos), de mayor tamao, para evitar el roce y facilitar la absorcin; realizar el aseo del rea con agua, dejando el rea sin paal el mayor tiempo posible. Las pastas con talco y xido de zinc son tiles como aislantes y para reducir la friccin y se aplican posterior a cada cambio de paal. En las lesiones con mas de 72 horas de evolucin o datos de infeccin por candida debe aplicarse algn agente antimictico, 2 veces al da, por 3 semanas.

En caso de imptigo segundario se recomienda el uso de antibiticos tpicos o sistmicos. La utilizacin de corticoesteroides deber ser racional, restringindose a casos resistentes a las terapias convencionales, utilizando esteroides de baja potencia y nunca ms de 2 semanas.

BIBLIOGRAFIA: HOSPITAL FEDERICO GOMEZ. GUAS DIAGNSTICAS Y TERAPUTICAS DE LAS 10 PATOLOGAS MS FRECUENTES DR CARLOS ALFREDO MENA CEDILLOS, JEFE DEL SERVICIO DRA ADRIANA MARA VALENCIA HERRERA 1. Kazaks EL, Lane AT. Diaper dermatitis. Pediatr Clin North Am 2000 Aug; 47 (4): 90919. 2. Mena-Cedillos CA. Dermatitis de la zona del paal. Bol Med Hosp Infant Mex 1997; 54: 386-91. 3. Ruiz-Maldonado R, Parish LCH, Beare LM. Tratado de dermatologa peditrica. McGraw Hill-Interamericana editores. Mxico, 1993. 4. Arenas R. Dermatitis de la zona del paal en: Dermatologa atlas. Diagnstico y tratamiento. McGraw Hill-Interamericana Editores. Segunda edicin. Mxico D.F. 1996; 41-2. 5. Pueyo de CS. Patologa reaccional en: Dermatologa infantil en la clnica peditrica. Artes Grficas Buschi. Argentina. 1999 Agosto; 183-212. 64.-Se trata de masculino en edad preescolar. Acude al servicio de consulta externa. Antecedentes: historia de geofagia. Hace 2 semanas presenta tos hmeda y "silbido del pecho". E.f.: temp. :37.2 c., mala higiene personal, trax con sibilancias espiratorias bilaterales. Se reportan labs. BH con anemia y eosinofilia. En la radiografa de trax de ste paciente espera encontrar datos de:

a) b) c) d)

Neumona lobar. Infiltrados migratorios. Infiltrado intersticial unilateral. Infiltrado miliar.

En 1932 Loeffler describe un sndrome caracterizado por sintomatologa respiratoria compatible con "pulmn sibilante",

Figura 1: Placa PA de trax del paciente., infiltrado intersticial Difuso abigarrado bilateral compatible con neumonitis.

Los hallazgos radiolgicos en los sndromes PIE generalmente son inespecficos, los infiltrados en la radiografa (Rx) trax pueden ser intersticiales, alveolares o mixtos, habitualmente son bilaterales y difusos. Radiolgicamente la eosinofilia inducida por parsitos se manifiesta por infiltrados fugaces y migratorios. En la NEC se observa la clsica imagen de fotografa negativa de edema pulmonar, caracterizada por compromiso de las zonas apicales y perifricas, con indemnidad de las zonas centrales e inferiores. La Rx trax en la ABPA muestra infiltrados fugaces, mediante la tomografa computada (TC) de trax se pueden evidenciar bronquiectasias centrales. En la NEA puede encontrarse un derrame pleural de escasa cuanta, que contiene un alto porcentaje de eosinfilos. La imagenologa adems es til en determinar la extensin del compromiso pulmonar, elegir los sitios ptimos para obtener la biopsia pulmonar y la respuesta al tratamiento. REFERENCIAS 1.- Del Giudice P, Desalvador F, Bernard E, Caumes E, Vandenbos F, et al. 2002. Lffler's syndrome and cutaneous larva migrans: a rare association. British J Dermatol 2002; 147: 385- 7 2. Botero D, Restrepo M. Parasitosis Humanas. 3 Ed., Medelln Colombia: Corporacin para Investigaciones Biolgicas 1998. 3.- Noem I, Atias A. Eosinofilia y parasitosis. En Atias A. Parasitologia Mdica. Mediterraneo, Santiago-Chile. 2000. 4.- Fujimura J, Murakami Y, Tsuda A, Chiba T, Migita M, Fukunaga Y. 2001.A neonate with Loeffler syndrome.Journal of Perinatology 2001; 21: 207-8 5.- Hunninghake GW, RichersonHB. Neumonitis por hipersensibilidad y neumonias eosinfilas. En Harrison TR. Principios de Medicina Interna. 15Ed. McGraw-Hill -Interamericana de Espaa, MadridEspaa. 2001 6.- Chusid MJ. Eosinophilia in childhood. Immunol and Allergy Clinics North America 1999; 19: 327-46 7.- Rothenberg ME. Eosinophilia N Engl J Med 1998; 338: 1592 8.- Noemi IH. Eosinofilia y parasitosis. Rev. Chil. Pediatr. 1999; 70: 1-7

9.- Hotez PJ, Broker S, Bethony JM, Bottazzi ME, Loukas A. Hookworm Infection. N Engl. J Med 2004; 351(8): 799-808 10.- Yilderan A, Ikinciogullari A. In the light of recent advances: eosinophil, eosinophilia and idiopathic hypereosinophilic syndrome. Turk Haematol 2005; 22(3): 107-16

65.- Se trata de paciente femenino de 42 aos diagnosticado con sndrome de Cushing, se realizan exmenes de laboratorio que demuestran una ACTH muy baja casi suprimida, la primera posibilidad de diagnstico es:

a) b) c) d)

Enfermedad hipotlamo-hipofisiaria Hiperplasia suprarrenal congnita Adenoma suprarreanal Adenoma hipofisiario

Aproximadamente 20 a 25% de los pacientes con sndrome de Cushing tienen una neoplasia suprarrenal. En la mayora de estos casos existe una produccin autnoma de cortisol por la neoplasia, lo que conduce a descenso en los niveles de ACTH a rangos indetectables o menores a 2 pmol/L o 10 pg/ml, por retroalimentacin negativa de la secrecin de ACTH. Williams GH, Dluhy RG. Enfermedades de la corteza suprarrenal. En Jameson JL (ed): Harrison. Endocrinologa. 1a ed. Madrid. MacGraw-Hill Espaa, 2006: 126-127.

66.- Se trata de masculino de 24 aos que acude por esterilidad; es azoosprmico, tiene fenotipo eunucoide, ginecomastia, distribucin feminoide del vello pbico, testculos pequeos y cromatina sexual positiva en 8%. El diagnstico ms probable es ? a) b) c) d) Sndrome de Klinefelter Sndrome de Noonan Sndrome de Turner Varn XX

SINDROME DE KLINEFELTER (SK) Es la cromosomopata ms frecuente y la causa ms habitual de hipogonadismo hipergonadotrpico en el varn. Descrito en 1942 como un sndrome caracterizado por hipogonadismo, testes pequeos y duros, azoospermia y ginecomastia. Se comprob posteriormente que el cuadro corresponde a una patologa gentica, cuya alteracin cromosmica ms habitual es la presencia de un cromosoma X adicional, reflejando un

cariotipo 47 XXY, que representa el 80% de los casos de SK, pero se han descrito otras variantes como mosaicismos: 47XXY/46XY, 47XXY/46XX, 47XXY/46XY/45X, etc. y formas con ms de un cromosoma X Y (48XXYY 47XXY/46XX/poliX). Esto hace que estos sujetos presenten una cromatina de Barr con masa presente, siendo esto propio de las mujeres por la presencia de 2 cromosomas X. La aparicin de ms de 2 cromosomas X, ocasiona una patologa que se diferencia del cuadro clsico de SK y se denomiona polisoma X del varn: 48XXY, 49XXXXY. La frmula XXY se debe a una no disyuncin del cromosoma X en la primera segunda divisin meitica, siendo ms frecuente la aparicin de SK en relacin a la edad materna ms avanzada. Su frecuencia se estima en 1/1000 recin nacidos varones, pero posiblemente sea ms elevada: en torno a 1/500 concepciones masculinas, y en varones con retraso mental moderado la frecuencia aumenta a 8/1000 aproximadamente. CLINICA: en general el diagnstico es tardo, debido a la pobre expresividad clnica de este sndrome en la infancia. 1) Talla alta: es propia del sndrome de Klinefelter en la edad de la adolescencia, pero no lo es en la poca prepuberal de forma habitual. Las proporciones corporales son eunucoides, con aumento desproporcionado de las extremidades inferiores y con inversin del cociente: segmento superior/inferior. Cuando este cociente es inferior a 1 en la edad puberal, sugiere fuertemente la existencia de un SK si se asocia a un cuadro de hipogonadismo aunque sea leve. La velocidad de crecimiento suele aumentar a partir de los 4 5 aos, y se adopta ya en la infancia un cierto hbito eunucoide. La edad sea es normal poco retrasada, coincidiendo con el marcado retraso de otras variantes benignas de retraso puberal. 2) Alteraciones genitales: Los testes pequeos y duros en la edad prepuberal, pueden presentarse en ocasiones, siendo por el contrario un hallazgo tpico pasada la adolescencia. La presencia de criptorqudea, micropene, hipospadias etc y a veces un fenotipo peculiar, puede orientar al diagnstico. En la edad puberal, la atrofia testicular es un signo constante, los testes son pequeos y su tamao va involucionando con el tiempo, contrastando con un desarrollo normal del pene y del relativamente normal desarrollo puberal. La histologa del testculo demuestra atrofia testicular con hialinizacin de los tbulos seminferos. 3) Ginecomastia: Est presente en un elevado n de casos (ms del 60%), siendo el riesgo de neoplasia mamaria 20 veces superior al de la poblacin normal. 4) Desarrollo intelectual: la inteligencia en general suele ser normal, bordeline, especialmente es el rea verbal la ms precoz y afectada, lo que condiciona dificultades de aprendizaje y en general peor rendimiento escolar. 5) Alteraciones hormonales: En la edad prepuberal, tanto el nivel de gonadotrofinas como de testosterona se encuentran en rango normal, tanto basal como tras estmulo. En la pubertad se va estableciendo ya respuesta propia del hipogonadismo hipergonadotrpico, especialmente el nivel de FSH que se eleva de forma llamativa.

6) Otras alteraciones asociadas: anomalas del tracto urinario, cbito valgo etc. (se recogen en la tabla adjunta). Especial relevancia por la ayuda que puede suponer para el pediatra, es la relativamente aumentada incidencia de incontinentia pigmenti. Son frecuentes las asociaciones del SK con neoplasias especialmente las de clulas germinales mediastnicas, y con otros tumores en la edad adulta. Rossodivita A, Colabucci F. Short stature in a patient with Klinefelter syndrome and growth hormone deficiency.Am J of Med Genet 1994; 49:244-6.

67.- Masculino de 47 aos con datos positivos de alcoholismo crnico, al cual se le diagnstica pancreatitis aguda Cul es la primera medida teraputica a adoptar? a) Iniciar antibiticos de amplio espectro. b) Administracin de inhibidores de la bomba de protones. c) Suspensin de la va oral. d) Aspiracin nasogstrica.

El 80 % de los pacientes con pancreatitis aguda se tratan mediante medidas de sostn, suspensin de la va oral, hidratacin intravenosa, analgsicos y alimentacin parenteral cuando la va oral se restringe ms all de la semana.

La indicacin de la suspensin de la va oral se basa en el dolor y la intolerancia digestiva. No debe ser prolongada y se debe restablecer secuencialmente luego de 48 horas sin dolor. La alimentacin parenteral no tiene ningn sentido si el restablecimiento de la va oral se realiza dentro de los primeros 7 das. De no ser as se sabe que esta patologa grave provoca una agresin severa que determina un estado hpercatablico por lo tanto debe implementarse soporte nutricional para evitar la desnutricin y las complicaciones que ella trae consigo (alteracin de la modulacin de la respuesta inflamatoria, translocacin bacteriana, inmuno supresin, etc.) El 60 % de esta enfermedad presenta hipermetabolismo (1,5 veces el metabolismo basal), el 40 % son normo o hipometablicos. Segn la Conferencia de Consenso de Nutricin de la Agresin (Francia 1998), los triglicridos no son contraindicados a menos que el paciente presente una hipertrigliceridemia importante. La necesidad de aporte debe ser de 0,25 a 0,30 g/Kg. La suplementacin de micronutrientes fundamentalmente antioxidantes (vitaminas A, C, E y selenio) y zinc estn indicadas. La suplementacin en base a glutamina, inmunomoduladores, nuevas emulsiones lipdicas en base a aceite de oliva necesitan aun ser confirmadas para la pancreatitis severa. La va enteral se debe privilegiar ya que no solo es mas fisiolgica sino que tambin presenta menor costo y complicaciones habiendo actualmente estudios que muestran un menor ndice de morbimortalidad en estos pacientes ya que la integridad de la barrera intestinal limitara la sobreinfeccin bacteriana pero fundamentalmente la fngica ( Kalfarentzos 97, Windsor 98, Pupelis 2000)

Referencias Banks PA, Freeman ML; Practice Parameters Committee of the American College of Gastroenterology. Practice guidelines in acute pancreatitis. Am J Gastroenterol. 2006 Oct;101(10):2379-400. Frossard JL, Steer ML, Pastor CM. Acute pancreatitis. Lancet. 2008;371:143-152.

68.- Se trata de masculino de 57 aos de edad acude a consulta refiriendo sensacin de cuerpo extrao en epigastrio e hipocondrio derecho, antecedente de pancreatitis crnica, afebril, presenta una masa abdominal palpable lisa y dura con datos de laboratorio de amilasa srica persistentemente elevada. El diagnstico de mayor probabilidad es:

a) b) c) d)

Cistadenoma pancretico. Pseudoquiste pancretico. Carcinoma pancretico. Coleccin pancretica aguda.

El pseudoquiste pancretico es una coleccin de jugo pancretico localizada, por lo general, en el interior o alrededor del parnquima pancretico. El pseudoquiste pancretico est confinado por una capa no epitelializada de tejido necrtico, fibrtico y de granulacin, que se desarrolla tras una lesin pancretica. Para su formacin requiere un mnimo de cuatro semanas desde que ese dao se produce. El pseudoquiste pancretico es una complicacin tanto de la pancreatitis aguda como de la crnica. Si bien la mayora de los pseudoquistes pancreticos se localizan en la cabeza y el cuerpo del pncreas, hasta un 20% de los mismos son extrapancreticos (1). Se han descrito pseudoquistes pancreticos en mltiples localizaciones, como cavidad pleural, mediastino y pelvis (2). Se presenta el caso de un pseudoquiste pancretico de localizacin heptica que apareci en el curso de una agudizacin de una pancreatitis crnica, y que se resolvi sin necesidad de drenaje. Diagnstico: Manifestaciones Clnicas: 1. 2. Pacientes en la cuarta o quinta dcada de vida, antecedentes etiolgicos. Pacientes con pancreatitis aguda que no resuelve luego de 5 a 7 das de tratamiento o luego de mejora recae. 3. Sensacin de cuerpo extrao y pesadez en la mitad superior del abdomen. 4. Si pancreatitis crnica, dolor abdominal o sntomas por compresin de vscera. 5. Nauseas, vmitos y prdida de peso por obstruccin duodenal. 6. Ictero, si compresin del coldoco. 7. Masa en abdomen superior, lisa y dura, muchas veces insensible. 8. Ms raramente ascitis y derrame pleural. Complementarios Diagnsticos:

1. 2.

Ultrasonografa Abdominal: Muchas veces diagnstico, preferido para vigilancia. TAC: Ideal para diagnstico.

Bibliografa 1. Hamm VB, Franzen N. Atypically located pancreatic pseudocyst in liver, spleen, stomach wall and mediastinum: their CT diagnosis. Rofo 1993; 159 (6): 522-7 2. Vitas GJ, Sarr MG. Selected management of pancreatic pseudocyst: Operative versus expectant management. Surgery 1992; 111 (2): 123-30. 3. Mofredj A, Cadranel JF, Dautreaux M, Kazerouni F, Hadj-Nacer K, Deplaix P, et al. Pancreatic pseudocyst located in the liver: a case report and literature review. J Clin Gastroenterol 2000; 30 (1): 81-3 4. Balzan S, Kianmanesh R, Farges O, Sauvanet A, O'toole D, Levy P, et al. Right intrahepatic pseudocyst following acute pancreatitis: an unusual location after acute pancreatitis. J Hepatobiliary Pancreat Surg 2005; 12 (2): 135-7.

69.- Debemos sospechar altamente de un retinoblastoma en un nio que presenta los siguientes sntomas:

a) Dolor, fotofobia y lagrimeo. d) Estrabismo y leucoria. b) Lagrimeo, fotofobia y aumento del dimetro corneal. c) Fotofobia y quemosis conjuntival.

RETINOBLASTOMA TUMOR OCULAR MS FRECUENTE EN INFANCIA. 1/20.000 RN 80% en < 3 aos Uni o bilaterales. Herencia 1. AD alta penetrancia (90-95%) 2. Espordicos

Diagnstico leucocoria estrabismo mala AV ojo rojo y doloroso celulitis orbitaria examen de rutina

Annals d.Oftalmologia 2001;9(2):74-92 N. Martn, MD. Coll, J. Garca, J. Snchez de Toledo, E. Trivio, M. Guitart, JJ. Gil 1Unidad Oftalmologa Peditrica. Hospital Maternoinfantil Vall d.Hebron 2Departamento de biologa-celular, fisiologa e inmunologa de la Universidad Autnoma de Barcelona 3Servicio oftalmologa Hospital General Vall d.Hebron

70.- Se trata de masculiono de 24 aos de edad, que acude al servicio de cardiologa, sin antecedentes patolgicos y sin hbitos txicos inicia con cuadro de 8 das de evolucin acompaado de fiebre y dolor centrotorcico intenso que aumenta con la inspiracin y los movimientos respiratorios. En el ecocardiograma se objetiva un derrame pericrdico importante, sin signos de compromiso hemodinmico. El diagnstico ms probable es:

a) b)

Pericarditis tuberculosa. Taponamiento cardaco.

c) d)

Pericarditis aguda idioptica. Pericarditis de origen autoinmune.

La pericarditis aguda (PA) es un sndrome clnico debido a una inflamacin del pericardio que se caracteriza por dolor torcico, roce pericrdico y alteraciones electrocardiogrficas evolutivas. Su incidencia y prevalencia son difciles de determinar. Estudios realizados en autopsias muestran una prevalencia de alrededor de 1% en la poblacin general, lo que sugiere que la presentacin en ocasiones se hace de manera subclnica. Representa un 5% de los dolores torcicos no isqumicos que acuden a urgencias 1. La causa ms frecuente de PA es la idioptica y/o viral, que corresponden a casi el 80% de los casos, de hecho, los trminos idioptico y viral suelen utilizarse de forma indistinta 2-5. Otras posibles causas son: Infecciosa (7%): Bacteriana: neumococo, estreptococo, estafilococo, neisseria, legionella. Tuberculosa. Vrica: coxsackie, influenzae, VIH, hepatitis, adenovirus, echovirus. Fngica: histoplasmosis, coccidiomicosis, blastomicosis, candidiasis. Otras: sfilis, protozoos, parsitos (entamoeba histolytica, toxoplasma). Neoplasias (7%): primarias: mesotelioma, sarcoma, fibroma, lipoma secundarias (metstasis): pulmn, mama, linfoma, leucemia, carcinoides La triada diagnstica clsica es: dolor torcico, roce pericrdico y alteraciones electrocardiogrficas que suele ser precedida de fiebre, mal estado general y mialgias (aunque en pacientes ancianos no suele presentarse la fiebre). Los sntomas principales son: Dolor torcico: dolor retroesternal o localizado en hemitrax izquierdo en la zona precordial, de caractersticas pleurticas (aunque en ocasiones puede simular un cuadro isqumico), que se puede irradiar al cuello o al trapecio izquierdo. Puede acompaarse de sensacin de falta de aire. Ocasionalmente se localiza en regin epigstrica, simulando un abdomen agudo. Suele aumentar con la inspiracin profunda, la tos, la deglucin y la posicin supina y mejorar con la incorporacin a la posicin de sentado. Roce pericrdico: es el hallazgo patognomnico de la exploracin fsica en la PA. Corresponde al movimiento del corazn dentro del pericardio y se asemeja al ruido que produce la friccin de cuero, por roce de las hojas pericrdicas inflamadas. Se da en aproximadamente el 60-85% de los casos 7,8. Puede ser transitorio, monofsico, bifsico o trifsico, segn la relacin que tenga con los movimientos cardiacos durante la sstole auricular, sstole ventricular y el llenado ventricular rpido. Se ausculta mejor a nivel de mesocardio y en parte baja de borde esternal izquierdo, sobretodo al final de la espiracin con el paciente inclinado hacia delante. Es independiente de la existencia de derrame.

Cuando slo tiene un componente se puede confundir con un soplo sistlico mitral o tricuspdeo. Es caracterstico del roce pericrdico su evanescencia (por lo que las auscultaciones deben ser repetidas en varias ocasiones) y los cambios en sus caractersticas segn la posicin en que se realice la exploracin.

Bibliografa
LeWinter MM, Kabbani S. Pericardial diseases. En: Braunwalds heart disease. Douglas P. Zipes editor. 7th. Ed. Philadelphia: Elsevier; 2005. p.1757-1780 Friman G, Fohlman J. The epidemiology of viral heart disease. Scand J Infect Dis Suppl 1993; 88: 710. [Medline] Braunwald E. Enfermedades del pericardio. En: Harrison Principios de Medicina Interna. Dennos L. Koper editores. 16 ed. Mxico: Mc-Graw-Hill; 2005. p. 1554-1571 Maisch B, Seferovic PM, Ristic AD, Erbel R, Rienmller R, Adler Y, et al, Grupo de Trabajo para el Diagnstico y Tratamiento de las Enfermedades del Pericardio de la Sociedad Europea de Cardiologa. Gua de Prctica Clnica para el diagnstico y tratamiento de las enfermedades del pericardio. Versin resumida. Rev Esp Cardiol. 2004; 57:1090-114. [Medline] [Texto completo] Zayas R, Anguita M, Torres F, Gimenez D, Bergillos F, Ruiz M, Ciudad M, Gallardo A, Valles F. Incidence of specific etiology and role of methods for specific etiologic diagnosis of primary acute pericarditis. Am J Cardiol. 1995 Feb 15; 75(5):378-82.

71.- Mujer de 23 aos, que presenta lesiones eritematoescamosas, edema y alguna vescula en la cara, escote, dorso de las manos y antebrazos. Las lesiones tienen 12 horas de evolucin y han aparecido tras una escursin al campo. Entre los antecedentes personales destaca acn vulgar en tratamiento con retinoides tpicos y doxiciclicina oral. El diagnstico ms probable es:

a) Erupcin lumnica poliforma. d) Reaccin fototxica. b) Eritrodermia por frmacos. c) Urticaria solar.

DEFINICIN Enfermedades cutneas que se producen por el aumento de capacidad de reaccin de la piel a las radiaciones lumnicas tras la administracin de una sustancia fotosensibilizante. Se conocen como reacciones de fotosensibilidad y pueden desencadenarse tanto por contacto

como por la administracin sistmica del agente fotosensibilizante. Si existe implicacin inmunolgica se denomina dermatitis fotoalrgica y si no dermatitis fototxica.

Dermatitis fototxica No existe un mecanismo inmunolgico, puede afectar a muchas personas siempre que exista dosis elevada de irradiacin y cantidad suficiente de sustancia qumica. Las lesiones aparecen tras la primera exposicin, son monomorfas, y se caracterizan por eritema intenso, edema y vesiculacin en reas de piel fotoexpuestas, marcando claramente los bordes de las zonas descubiertas, y onicolisis ungueal . Formas particulares de fototoxia: fitofotodermatitis (dermatitis de los prados, apio) (Fig. 3), dermatitis de Berloque, fotosensibilidad en tatuajes (sulfuro de cadmio), frmacos (tetraciclinas, AINEs, amiodarona (color azulado), clorpromacina (color gris).

Figura 3. Fitofotodermatitis
1. Litt Jz. Drug eruption reference manual 2001. New York: Parthenon, 2001. 2. Sullivan JR, Shear NH. Drug eruptions and other adverse drug effects in aged skin. Clinics in geriatric medicine 2002;18(1). 3. Lim HW, Gigli. Complement-derived peptides in phototoxic reaction. En: Daynes RA, Spikes JD, editors. Experimental and clinical photoimmunology. Boca Raton: CRC Press, 1983:81-93. 4. Torinuki W, Tagami H. Role of complement in chlorpromazine-induced phototoxicity. J Invest Dermatol 1986;86:142-4. 5. Hearst JE, Issacs ST, Kanne D, Rapoport H, Straub K. The reaction of the psoralens with deoxyribonucleic acid. Q Rev Biophys 1984;45:891-5. 6. Athar M, Elmets CA, Bickers DR, Mukhtar H,. A novel mechanism for the generation of superoxide anions in hematoporphyrin derivative-mediated cutaneous photosensitization. Activation of the xantine oxidase pathway. J Clin Invest 1989;83:1137-43. 7. Matsuo I, Inukai N, Fujita H, Ohkido M. Possible involvement of oxidationmof lipids in inducing griseofluvin photosensitivity. Photodermatol Photoimmunol Photomed 1990;7:213-

7. 8. Harber LC, Bickers DR. Photosensitivity diseadses. Principles of diagnosis and treatment. Ontario: BC Decker Inc, 1989:160-202. 9. Kockevar IE. Phototoxicity of nonsteroidal inflammatory drugs. Coincidence or specific mechanism?. Arch Dermatol 1989;125:824-6.

72.- Which one of the following conditions results in prologation of the partial thromboplastin time (PTT), but not the prothrombin time (PT)? a) b) c) d) Varicela hemorrhage as a result of cirrosis Therapy with broad-spectrum antibiotics Therapy with coumarin for phlebitis Menorrhagia resulting from von Willebrands disease

Enf. Von Willebrand

Manifestaciones

Epistaxis Hemorragia transvaginal Equimosis Gingivorragias Hematomas Hemartrosis

60 % 50 % 40 % 35 % 5% 3%

EvW Pruebas de escrutinio T. Hemorragia C. Plaquetaria TTPa TP Gpo AB0 Prolongado Normales, excepto 2B Normal o prolongado Normal 25 % bajo en 0

1.-Lee GR, Foerster J, Lukens J, Paraskevas F, Greer JP and Rodgers GM. Wintrobes clinical haematology; 10th Edition, Lippincott Williams & Wilkins, United States of America, 1999. 2.-Williams WJ. Manual Williams de hematologa 5a Edicin McGraw-Hill Interamericana, Mxico, 1997. 3.-Beutler E, Lichtman MA, Coller BS, Kipps T. Hematology. 5th International Edition. United Stated of America, 1995.

4.-Ruiz Argelles GJ. Fundamentos de hematologa ; 2. Edicin, Editorial Mdica Panamericana, Mxico, 1998.

73.- .Se trata de masculino de 27 aos acude a su consultorio con reporte de exmenes de laboratorio con los siguientes resultados. Hemolisinas bifsicas + (o anticuerpos de Donath-Landsteiner) productoras de hemoglobinuria paroxstica a frigore. La Entidad responsable de los resultados de este paciente es: a) Leucemia linftica crnica. b) Sfilis. c) Mieloma mltiple. d) Lupus eritematoso diseminado. Razones por las que se realiza el examen Este examen se realiza algunas veces cuando el mdico sospecha de un diagnstico de criohemoglobinuria paroxstica. Valores normales La ausencia de anticuerpos es lo normal.

Significado de los resultados anormales Los resultados anormales indican la presencia de criohemoglobinuria paroxstica (PCH), un trastorno que ocurre cuando la exposicin a bajas temperaturas hace que el sistema inmunitario produzca anticuerpos que destruyen los glbulos rojos. Estos anticuerpos se denominan anticuerpos de Donath-Landsteiner. A medida que las clulas son destruidas, la parte de los glbulos rojos (hemoglobina) que transporta el oxgeno es eliminada en la orina. La criohemoglobinuria paroxstica es un sndrome adquirido. Algunas veces, los anticuerpos se presentan con una infeccin viral (por ejemplo, sarampin y paperas) o con sfilis; sin

embargo, en algunos casos, el trastorno no est relacionado con una enfermedad y la causa se desconoce. K. Holmes, P. Mardh, P. Sparling et al (eds). Sexually Transmitted Diseases, 3rd Edition. New York: McGraw-Hill, 1999, chapters 33-37.

74.- Masculino de 14 aos de edad con antecedentes de convivencia con aves, gatos e ingestin muy frecuente de berros. Inicia su padecimiento actual hace dos semanas con fiebre de 39C sin predominio de horario, nusea y dolor en hipocondrio derecho. A la exploracin fsica con palidez generalizada y hepatomegalia 3-3-5. Resto de la exploracin sin datos patolgicos. Laboratorio con BH, con Hb de 11.5, Hto de 40, leucocitos; 16,500, eosinfilos; 38%, linfocitos; 30%, formas inmaduras; 4, plaquetas; 270,000, Fosfatasa alcalina; 280 UI/L.

El diagnstico ms probable en ste paciente es:

a) Larva migrans visceral b) Toxoplasmosis generalizada c) Fasciolosis e) Estrongiloidosis

Definicin La Fasciolosis es una zoonosis parasitaria causada por la Fasciola heptica que ocasiona patologa y sintomatologa hepato-biliar.

Clnica Se considera las siguientes formas de presentacin clnica: a) Sintomtica: Aguda o invasiva, crnica o de localizacin y extraheptica. 1. Aguda o invasiva: Hay tres elementos esenciales a identificar: hepatomegalia dolorosa, fiebre y eosinofilia con cifras que superan frecuentemente el 30-40%. 2. Crnica o de localizacin: La sintomatologa y signologa corresponden a padecimiento crnico hepato biliar incluyendo clicos biliares y litiasis biliar. 3. Extraheptica: Incluye ndulos subcutneos en el hipocondrio derecho, seno derecho, escpula derecha con poco dolor local y signos inflamatorios. Eosinofilia alta. b) Asintomtica: En algunas personas los sntomas o signos suelen pasar desapercibidos.

Diagnstico de Laboratorio

En la forma aguda, la bsqueda de huevos en las deposiciones es intil, ya que las formas juveniles estn en el tejido heptico, por lo tanto las pruebas inmunobiolgicas son importantes. Son tiles la inmunoelectroforesis o inmunodifusin buscando el arco 2 de Caprn (4). El inmunoblot o westernblot tiene buena sensibilidad y especificidad (5,6). Se han identificado fracciones antignicas en las cistenilproteasas de F. hepatica (7). En las formas crnicas, la bsqueda de huevos en heces es lo indicado. Son tiles la sedimentacin rpida de Lumbreras (8). Recientemente se han preparado anticuerpos monoclonales contra el parsito y ello ha permitido elaborar la tcnica de ELISA para detectar los coproantgenos (E/S) del parsito en heces. La ecografa de vas biliares (v.b.) puede detectar al parsito movindose en las v.b. o vescula. En formas extrahepticas, la eosinofilia alta es orientadora; F. hepatica en las biopsias confirma el diagnstico.

75.- Masculino de 46 aos de edad que inicia con dolor intenso en fosa renal izquierda ante la sospecha de litiasis renoureteral se realizan estudios de laboratorio y gabinetem, los Rx demuestran clculos radiolcidos, los cuales estn relacionados a: a) b) c) d) Calcio Estruvita Cistina cido rico

-Bruce E. Jarrell, R. Anthony Carabasi, Nacional Medical Series for Independent Study. Wiliams & Wilkins, 3rd Edition: 451-475. Los clculos que sepueden presentar en la va urinaria son formados por calcio, cido rico y cistina, los nicos radiolucidos son los de cido rico que representan un reto diagnstico.

76.- Masculino de 56 aos con diagnstico probable de sinusitis aguda, la proyeccin radiolgica que mejor valora los senos maxilares y las estructuras intranasales, en sta patologa es:

a) b) c) d)

Lateral. Submentoniana. Anteroposterior. Waters.

La proyeccin Waters es la proyeccin que mejor permite valorar la neumatizacin, opacificacin o engrosamiento de mucosa de los senos maxilares as como las estructuras intranasales.

Proyeccin de Waters u occipito-mentoniana para senos maxilares (Radiografa normal). Gonzlez-Saldaa N, Infectologa Clnica Peditrica, 7 edicin, pginas 63-98.

77.- Masculino de 18 aos que desde la infancia padece anemia, esplenomegalia e ictericia. El frotis de sangre perifrica muestra eritrocitos pequeos con palidez central y una fragilidad osmtica muy incrementada. El diagnstico ms probable es:

a) Anemia perniciosa b) Talasemia mayor c) Esferocitosis hereditaria d) Leucemia linfoblstica

La esferocitosis hereditaria (EH) es una enfermedad caracterizada por anemia hemoltica de severidad variable, con presencia de esferocitos en sangre perifrica y una respuesta clnica favorable a la esplenectoma. Con el desarrollo de nuevas tcnicas se encontraron las primeras alteraciones bioqumicas de las protenas de la membrana eritrocitaria, y posteriormente, se han podido precisar las alteraciones moleculares mediante las tcnicas del ADN recombinante. La EH es una enfermedad muy heterognea que se produce por un defecto intrnseco del glbulo rojo, y existen otras alteraciones secundarias a esta afeccin. La prueba ms utilizada para el diagnstico de la EH es la fragilidad osmtica del glbulo rojo. Se ha demostrado que esta enfermedad es producida por defectos de las protenas que intervienen en las interacciones verticales entre el esqueleto de la membrana y la bicapa lipdica. El tratamiento de eleccin en la EH es la esplenectoma, ya que es el ms efectivo en el control de la anemia, aunque la sobrevida de los glbulos rojos permanece acortada y los esferocitos no desaparecen. Este proceder se indica en pacientes con anemia hemoltica severa o en individuos moderadamente asintomticos pero que presentan litiasis vesicular.

Formas Clnicas La EH es una enfermedad muy heterognea desde el punto de vista clnico. Se puede observar desde el portador asintomtico hasta pacientes que presentan una anemia hemoltica crnica con grandes requerimientos transfusionales.24,25 Dependiendo de la severidad del cuadro clnico, de las cifras de hemoglobina, los niveles de bilirrubina y el conteo de reticulocitos, esta enfermedad se clasifica en 4 formas: portador asintomtico, EH ligera, EH tpica y EH severa.26,27

Portador asintomtico. En algunas familias se ha sealado un patrn de herencia

autosmico recesivo. En estos casos, los padres de un paciente afectado no presentan ninguna alteracin. En ocasiones la afectacin es muy leve, como ligero incremento de las cifras de reticulocitos, escasos esferocitos en periferia o fragilidad osmtica incubada alterada y puede no ser detectada por los exmenes de rutina. Debe tenerse en cuenta tambin que pueden ocurrir nuevas mutaciones dentro de una familia aparentando un patrn de herencia autosmico recesivo, por lo que siempre es importante un estudio minucioso de todos los miembros de la familia.3,4.

EH ligera. Comprende entre el 20 y 30 % de todos los pacientes con EH autosmica

dominante, los que pueden presentar una hemlisis ligera compensada.3,27 Los individuos son frecuentemente asintomticos y algunos casos son difciles de diagnosticar, ya que la anemia y la esplenomegalia son muy ligeras y en ocasiones pueden estar ausentes.28 Muchos de estos pacientes se diagnostican durante estudios familiares o cuando en la etapa adulta aparece el ctero y la esplenomegalia. Episodios hemolticos pueden presentarse en el curso de algunos procesos infecciosos como mononucleosis, parvovirus o citomegalovirus, as como durante el embarazo, por esfuerzos fsicos intensos o por sangramientos.27-30.

EH tpica. Entre el 50 y 60 % de los pacientes con EH autosmica dominante tienen esta forma clnica. Presentan una hemlisis compensada incompleta y una anemia de ligera a moderada. El ctero es comn en nios, aunque se puede ver tambin en los adultos y est asociado con infecciones virales ligeras, debido a la estimulacin reticuloendotelial y a un aumento de la hemlisis. Los requerimientos transfusionales son espordicos. La esplenomegalia est presente en el 50 % de los nios y en el 75 % de los adultos.2,3,31,32. EH severa. Estos pacientes (5-10 %) evolucionan con una hemlisis severa y presentan

frecuentes requerimientos transfusionales. La mayora de estos casos tienen una forma autosmica recesiva de la enfermedad. Pueden presentar crisis aplsticas, retardo del crecimiento y de la maduracin sexual. La esplenectoma es el tratamiento de eleccin en esta forma clnica.33,34. Generalmente la enfermedad debuta al nacimiento con ictericia y hemlisis y se requiere, en muchas ocasiones, de exanguinotransfusin.30,32.

REFERENCIAS BIBLIOGRFICAS

1. 2.

3. 4. 5.

6.

7. 8.

9.

Delaunay J. Genetic disorders in the red cell membrane. Crit Rev Oncol Hematol;19:79-110. Iolascon A, Miraglia del Giudice E, Perrotta S, Alloisio N, Morle L, Delaunay J. Hereditary spherocytosis: from clinical to molecular defects. Haematologica 1; 83:240-57. Scriver SR, Beaudet AL, Sly WS, Valle D. The metabolic and molecular bases of inherited disease. Philadelphia: McGraw-Hill Company, CD-ROM, . Yu J, Steck TL. Isolation and characterization of band 3, the predominant polypeptide of the human erythrocyte membrane. J Biol Chem ;250:9170-6. Kay MMB, Folwers N, Goodman J. Alterations in membrane protein band 3 associated with accelerated erythrocyte ageing. Proc Nall Acad Sci USA 1989;86:5834-8. Marsh WL. Molecular defects associated with McLeod blood group phenotype. En. Salmon E, ed. Blood groups and other red cells surface markers in health and disease:17-82. Tse WT, Lux SE. Red blood cell membrane disorders. Br J Haematol 1999;104:2-13. Speicher DW, DeSilva TM, Speicher KD, Ursitti JA, Hembach P, Weglarz L. Location of human red cell spectrin tetramer binding site and detection of a related closed hairpin loop dimer using proteolytic footprinting. J Biol Chem 1993;268:4227-31. Jordan C, Puschel B, Koob GR. Identification of a binding motif for ankirin on the a subunit of Na+ K+ATPase. J Biol Chem 1995;270:29971-4

78.- Masculino de 62 aos con antecedentes de insuficiencia cardiaca de 2 aos de evolucin, actualmente se presenta a consulta por presentar edema en MPS. Una de las siguientes puede ser la causa de un edema no inflamatorio:

a) Un aumento de la permeabilidad vascular. b) Una eliminacin excesiva de sal y agua por el rin. c) La disminucin de la presin hidrosttica intravascular. d) Un aumento de la presin hidrosttica intravascular.

Edema Acumulo de un exceso de lquido en el espacio tisular intercelular (intersticial) o en las cavidades del organismo. El edema localizado: se puede producir en el caso de una obstruccin del flujo seroso (producindose la inflamacin o hinchazn de una pierna por ejemplo). Tambin puede ser causado por un proceso infeccioso (como los abscesos causados por Staphylococcus aureus). Edema generalizado: es de carcter sistmico como en el caso de insuficiencia cardiaca o de sndrome nefrtico (que se caracteriza por una intensa proteinuria secundaria a la permeabilidad glomerular anmala). Cuando el edema es intenso y generalizado de tal forma que provoca hinchazn difusa de todos los tejidos y rganos de la economa, especialmente a nivel del tejido subcutneo, se denomina Anasarca. Exudado: lquido inflamatorio extravascular rico en protenas, detritus celulares y leucocitos. Se trata de un acumulo de lquido que se encuentran rico en restos celulares, leucocitos, restos protenicos y por lo general presenta una r>1.020. Se debe a un incremento en la permeabilidad endotelial con la salida de protenas plasmticas (principalmente albmina). Se presenta por infecciones pigenes, TB, etc. (de origen inflamatorio). Trasudado: en el lquido de edema de origen no inflamatorio y est relacionado con los casos de insuficiencia cardiaca y nefropatas por lo general no hay (es pobre) protenas, ni leucocitos, con una r1.012. nicamente es el acumulo de lquidos. El trastorno hemodinmico (trasudado) se presenta cuando se altera la Ley de Starling que es el equilibrio normal de los lquidos (intravascular y extravascular). Se mantiene por la accin de dos grupos de fuerzas opuestas, los que hacen que el lquido tienda a salir de la circulacin son la presin osmtica del lquido intersticial y la presin hidrosttica intravascular; los que hacen que el lquido pase a la circulacin son la presin osmtica de las protenas plasmticas (principalmente albmina) (presin coloidosmtica) y la presin hidrosttica tisular. Por lo tanto el equilibrio entre estas fuerzas es tal que en los capilares musculares perifricos existe un movimiento neto de lquidos hacia fuera, pero ste lquido es drenado a los linfticos, por lo que no se produce edema. Los factores que aumentan la presin hidrosttica intravascular o disminuye la presin coloidosmtica intravascular dan lugar al aumento de la salida del lquido desde los capilares, con el consiguiente trastorno hemodinmico el edema.

Nota: en la lesin (por falta de la Angiotensina, que regula las sales) por prdida de protenas (principalmente albumina) (o sea la proteinuria) se produce el Edema generalizado (Anasarca).

Fisiopatologa del edema Autores: Jos Mara Sillero Fernndez de Caete

Localizacin: Seminario mdico, ISSN 0488-2571, Vol. 49, N. 1, 1997 , pags. 7486

Organizacin Panamericana de la Salud Programa de Publicaciones (DBI/E) 525 Twenty-third Street, NW

79.- Which of the following physical sings and syntoms is indicative of left ventricular failure? a) b) c) d) Neck vein distensin Ascites Anorexia Orthopnea

INSUFICIENCIA CARDIACA IZQUIERDA Los sntomas ms caractersticos son la disnea paroxstica nocturna, la ortopnea y la tos. Al examen fsico se constata la existencia de taquicardia, R3 y/o R4, pulso alternante, estertores inspiratorios de predominio en las bases. Algunas veces se auscultan sibilancias. Para confirmar su existencia se deben solicitar los siguientes estudios: a. Radiografa de trax. Para constatar la existencia de cardiomegalia, vasos sanguneos prominentes, lneas B de Kerley, (patrn en "alas de mariposa" por edema pulmonar gravitacional) y derrame pleural.

b. Gases arteriales

c.

Cuadro hemtico

d. Creatinina

e.

Electrocardiograma

f.

Ecocardiografa

LECTURAS RECOMENDADAS Bigger JT. Why patients with congestive heart failure die.

Circulation 75 (suppl, IV):28, 1997 Braunwald E. Heart Disease. En: Textbook of Cardiovascular Medicine. WB Saunders Co. Philadelphia, 1990 Matiz H. Insuficiencia cardiaca congestiva En: Diagnstico y Tratamiento Integral en Medicina. Editado H Matiz. Coleccin Educacin Mdica Vol. 5. Fundacin Escuela Colombiana de Medicina Santaf de Bogot, 1991

80.- El dolor abdominal intermitente, la anemia, la proteinuria, la necrosis tubular, la esterilidad, los trastornos de la conducta, la polineuropata perifrica se encuentran en la intoxicacin por: a) b) c) d) Plomo Organoclorados. Sulfuro de hidrogeno. Etilenglicol

La intoxicacin por plomo es la ms comn de las exposiciones a metales, el cual tiene muchos usos, las fuentes ms frecuentes vienen de las minas y del reciclado de materiales conteniendo plomo. Este metal es absorbido por pulmones y del tracto gastrointestinal. El mecanismo de accin es por unin a los grupos sulfhidrilo y txico para las enzimas dependientes de zinc. Diagnstico: La toxicidad aguda se presenta luego de una exposicin respiratoria a altas concentraciones, con encefalopata, insuficiencia renal y sntomas gastrointestinales. La toxicidad crnica es la ms frecuente y se manifiesta con compromiso multisistmico: hematopoytico, del sistema nervioso, gastrointestinal, rin y sistema reproductor. (Astenia, dolor abdominal, irritabilidad, nusea, vmitos, prdida de peso, cefalea, anemia, neuropata perifrica, ribete de Burton, IRC, proteinuria, Nefritis intersticial, etc). En los exmenes auxiliares podemos encontrar anemia, punteado basfilo, aumento del cido rico, etc.

REFERENCIA BIBLIOGRFICAS 1. Keogh JP y Boyer LV. Lead en Sullivan y Krieger editores: Clinical Environmental Health and Toxic Exposures. Lippincott Williams & Wilkins, 2da edicin, 2001

2. Decreto Supremo N 019-98-MTC. Dispone eliminar del mercado la oferta de gasolina 95 RON con plomo y reducir el lmite mximo de contenido de plomo en la gasolina 84 RON. (14/07/98) 3. Shannon Michael. Lead en Haddad, Shanon y Winchester editores: Clinical Management of Poisoning and Drug Overdose. WB Saunders, 3ra edicin, 1998. 4. Krantz A, Dorevitch S. Metal exposure and common chronic diseases: A guide for the clinician. Dis Mon 2004; 50:215- 262. 5. Nogu S. Burton s Line. N Engl J Med 2006; 354:e21. 6. Rempel D. The lead-exposed worker. JAMA 1989; 262:532-4.

81.- Mujer de 26 aos G-3, P-1, A-1 con 39 SDG por FUR. Reporta contracciones uterinas que han sido regulares las ltimas tres horas. Al examen encuentras que las contracciones son cada tres minutos y duran 50 segundos y son firmes a la palpacin. Tuvo ruptura de membranas hace una hora y lo demuestras con papel de nitrazina. El examen digital cervical demuestra una dilatacin de 5 cm, con borramiento del 100% y presentacin en vrtex en estacin 0. Cual de los siguientes criterios es el ms preciso para decir que se encuentra en la fase activa del trabajo de parto?

a) b) c) d)

Dilatacin cervical mayor de tres centmetros Borramiento cervical ms de 90% Duracin de las contracciones de ms de 30 seg Ruptura de membranas

FASES DEL TRABAJO DE PARTO El trabajo de parto se divide en tres fases: Fase 1 latente Es llamado as al periodo que sirve para la preparacin uterina del parto, ocurre al final del embarazo y va hasta el inicio de las contracciones del trabajo de parto. Los aspectos a destacar en este lapso es el reblandecimiento cervical, el aumento importante en el nmero de receptores para oxitocina a nivel de las clulas endometriales, un aumento sustancial en los puentes de unin y el nmero de conexinas a nivel miometrial y por consiguiente una mayor sensibilidad a los agentes uterotnicos. Fase 2 activa Es el lapso que representa el trabajo de parto activo, y se acepta que se inicie cuando existen 3 cm de dilatacin y las contracciones uterinas son aptas para producir un avance en el trabajo de parto; se divide en tres periodos: Primer periodo. Se inicia cuando las contracciones uterinas alcanzan la frecuencia, intensidad y duracin suficientes para causar borramiento y dilatacin del cuello uterino, y finaliza cuando ste se encuentra en completa dilatacin. El lapso de tiempo que dura es variable, pero se acepta como normal hasta diez horas en primigrvidas y ocho horas en multigrvidas; pero independientemente de esto, se debe considerar como adecuado si el borramiento y la dilatacin cervical son progresivos e ininterrumpidos.

Segundo periodo. Se inicia con una dilatacin cervical completa y termina con la expulsin del feto; tiene una duracin variable, pero se acepta como normal una hora en pacientes primparas y 30 minutos en multparas; y tiene como caracterstica que debe de ser progresivo e ininterrumpido. Tercer periodo. Este comienza inmediatamente finalizada la expulsin fetal y termina con

la expulsin total de la placenta y las membranas corioamniticas; a este periodo se le conoce tambin como de alumbramiento y es el ms corto de los periodos del parto; como norma general se acepta que no debe de extenderse ms all de 10 minutos. Existen algunos autores que incluyen un cuarto periodo dentro del trabajo de parto, el cual abarca aproximadamente la hora posterior al alumbramiento, y comprende el lapso de tiempo cuando ocurre la contraccin y retraccin de las fibras miometriales, as como la trombosis de los vasos adyacentes, lo cual es un efectivo control de la hemorragia del sitio de implantacin de la placenta. Fase 3 Este periodo es el que representa el regreso de la mujer a su estado previo al embarazo, y se caracteriza por la involucin uterina, la eyeccin lctea y por ltimo la restauracin de la fertilidad; existen estudios que involucran en esta fase a la endotelina-1 y a la oxitocina como substancias responsables de estos cambios postparto.

PROGRAMA DE ACTUALIZACION CONTINUA PARA GINECOLOGA Y OBSTETRICIA PAC GO-1 Libro 3 Obstetricia 2005

82.- Se trata de femenino de 32 aos de edad que cursa en ste momento con diagnstico de preclampsia leve, el frmaco de eleccin que se administra en esta patologa es: a) b) c) d) Nifedipina. Alfametildopa. Inhibidores de la enzima convertidora de angiotensina. Clonidinas.

Prevenir complicaciones a corto plazo de las mujeres con PA elevada que comprometa el bienestar fetal Cuando la PAS es mayor o igual a 150 mmHg y la PAD mayor o igual a 100 mmHg. El propsito es alcanzar cifras de TA alrededor de 140/90. La medicacin antihipertensiva se reserva para los casos en que la PAD 100 mmHg. Se recomienda continuar el tratamiento antihipertensivo previo al embarazo, exceptuando el uso de IECA. La alfametildopa y la hidralazina va oral son los frmacos de eleccin dado su uso extensivo con seguridad y eficacia y sin efectos colaterales para el feto (excepto hidralazina en lupus).

ALFA METILDOPA 500-2000 MG/DA HIDRALAZINA 50-200 MG/DA LABETALOL 100-400 MG/DIA ATENOLOL 50-200 MG/DA NIFEDIPINA 10-30 MG/DA 1. Aagard K, Belfort M. Eclampsia: Morbility, mortality, and management. Clin Obstet Gynecolol. 2005; 48: 12-23. 2. Oyarzn E. Sndrome hipertensivo del embarazo en Oyarzn E. Ed. Embarazo de alto riesgo. Ediciones Universidad Catlica de Chile. Santiago. 1997: 157175. 3. Roberts J, Redman C. Pre-eclamsia: More than pregnancy induced hypertens

83.- Masculino de 7 meses de edad, presenta fiebre alta desde hace 3 das, acompaada de hiperemia farngea. La fiebre cede al cuarto da de la enfermedad, momento en que aparece un exantema morbiliforme, que desaparece en un plazo de 3 das. El diagnstico ms probable es:

a) b) c) d)

Rubola Infeccin por herpes virus humano 6 Reaccin medicamentosa a antitrmicos Infeccin por enterovirus

Nos encontramos ante un caso de exantema sbito tpico (tambin llamado rosola infantil o 6 enfermedad). Se trata de una enfermedad exantemtica, de mayor incidencia entre los 6 y 12 meses (como el caso de la pregunta) y que se presenta con una frecuencia del 90% antes de los aos de vida. No muestra preferencia por ningn sexo y es algo ms frecuente al final de la primavera y principios del verano. En el 80-92% de los casos de exantema sbito el agente etiolgico es el Herpes Virus Humano. Lo ms caracterstico de este perodo febril es la ausencia de hallazgos fsicos suficientes para explicar la fiebre, as como el buen estado general del lactante a pesar de la misma. Al tercer o cuarto da cede la fiebre y aparece una erupcin maculosa o maculopapulosa que comienza en tronco y se extiende a los brazos y al cuello. Tambin afecta algo a piernas y cara. La erupcin desaparece en unos 3 das, normalmente no dejando descamacin ni despigmentacin residual. El diagnstico de este proceso es fundamentalmente clnico, por la edad y la secuencia de las manifestaciones clnicas.

Es caracterstico una BHC realizada en 24 -36h puede mostrar leucocitosis con neutrofilia, pero ms all de 48h aparece el patrn tpico d leucopenia con neutropenia absoluta y linfocitosis relativa.

Nelson, Tratado de Pediatra 15 Ed., pgs. 1120-21

84.- Varn de 4 semanas de vida acude a consulta inicialmente con una historia de 2 semanas de vmitos crecientes y escasa ganancia ponderal. Los vmitos, a menudo proyectados, han persistido pese a que se ha cambiado varias veces de frmula. En las ltimas 24 horas tiene vmitos en posos de caf. Las heces han sido firmes y el nio defeca un da si y otro no. La exploracin fsica revela un lactante delgado e irritable con abdomen plano. En el hipocondrio derecho se nota una masa firme en forma de aceituna. De los siguientes estudios de laboratorio, el que debe llevarse a cabo inicialmente es: a) b) c) d) Radiografa de abdomen Valoracin de electrlitos en suero. Ecografa abdominal Recuento sanguneo completo.

La historia y la exploracin fsica son tpicas de un diagnostico de estenosis hipertrfica del piloro. En este caso, la consideracin mas importante es el estado hidroelectroltico del paciente, ya que en este trastorno pueden observarse anomalas graves. Es ms comn una alcalosis metablica hipoclormica, y la terapia inicial anterior a la correccin quirrgica, debe incluir normalizacin de los electrolitos del suero. En muchos lactantes con estenosis Pilarica se observa hematemesis, normalmente como resultado de una gastritis superficial. Sin embargo, rara vez es de consecuencia hemodinmica. La confirmacin del diagnstico puede realizarse fcilmente en la mayora de los casos mediante ecografa de abdomen, que revela un canal pilrico alargado y engrosado. La radiografa de abdomen puede mostrar un estomago distendido y lleno de aire. La radiografa con contraste revela un canal pilrico alargado con retraso del vaciamiento gstrico. Un recuento sanguneo completo no tiene ningn valor en el diagnstico de la estenosis Pilrica.

LECTURA RECOMENDADA Acta Peditrica, Mex. 2010;31(2):50-54 INP Estenosis hipertrfica del ploro. Estudio clnico-epidemiolgico Dr. Carlos Baeza-Herrera,* Dra. Aln Villalobos-Castillejos,** Dr. Arturo Arcos-Aponte,*** Dr. Javier LpezCastellanos,**** Dr. Luis Manuel Garca-Cabello*****

85.- Mujer de 39 aos la cual presenta amenorrea secundaria de 2 aos y medio de evolucin. Los niveles reportados de prolactina son de 150ng/ml (normal hasta 20 ng/ml). La resonacia magntica detecta macrotumor de 2,8 cm. de dimetro con expansin lateral izquierda. Sin presentar alteraciones visuales. El tratamiento de eleccin es: a) Ciruga por tratarse de un macrotumor. b) Tratamiento mdico con agonistas dopaminrgicos. c) Somatostatina previa a ciruga. d) Radioterapia hipofisaria previa a ciruga.

Tratamiento Los dopaminrgicos han revolucionado el tratamiento del prolactinoma y virtualmente han dejado fuera a la ciruga; as independientemente del tamao del adenoma la primera opcin teraputica es la farmacolgica.1,2 Con los dopaminrgicos se consigue en poco tiempo restaurar el funcionamiento ovrico y corregir la esterilidad, incluso antes de que se normalice la concentracin de prolactina; asimismo se consigue reducir el tamao del adenoma. Los dopaminrgicos actan sobre los receptores localizados en las clulas mamotrpicas de la hipfisis anterior y suprimen la sntesis y secrecin de prolactina con la consecuente normalizacin del eje hipotlamogonadotropico hipotlamogonadotropico- ovrico. La accin dopaminrgica puede ocasionar efectos colaterales indeseables como nusea, hipotensin arterial, constipacin nasal, mareo y estreimiento, los cuales no necesariamente corresponden con la dosis utilizada, pero s se relacionan con el tipo de dopaminrgico. Est ampliamente documentada la superioridad de la farmacoterapia para el tratamiento de los prolactinomas; adems la ciruga es raramente curativa, incluso en el caso de microadenoma.4,5 Cuadro I. Agentes dopaminrgicos que se usan hiperprolactinemia y el prolactinoma. Genrico Comercial Dosis (mg) Bromocriptina Parlodel 2.5-5 diaria Lisurida Dopergin 0.2 diaria Quinagolida Norprolac 25-50 diaria Cabergolina Dostinex 0.5 c/4 das como tratamiento de la

Los adenomas hipofisarios representan el 10% de todos los tumores intracraneales diagnosticados y 25% de los tumores cerebrales que son intervenidos quirrgicamente. Los objetivos del tratamiento de un paciente con un adenoma de la pituitaria son: eliminar el efecto de la masa tumoral (compresin sobre estructuras vecinas) disminuir la produccin excesiva de hormonas, restaurar la funcin normal de la pituitaria y evitar la recurrencia. El tratamiento de eleccin para todos los prolactinomas es con un agonista de la dopamina. La bromocriptina y la cabergolina son efectivas para reducir el tamao del tumor y para restaurar la funcin gonadal. El tratamiento quirrgico debe recomendarse slo cuando falla el tratamiento mdico. Los tumores de la pituitaria productores de hormona de crecimiento son tratados preferentemente mediante adenomectoma transesfenoidal, pero la normalizacin de los niveles de HC y de IGF-1 ocurre en menos de la mitad de los pacientes con macroadenomas;

por lo tanto, un importante nmero de pacientes acromeglicos requiere un tratamiento adicional. Los anlogos de la somatostatina son en la actualidad los medicamentos que ms usados para el control de la acromegalia. En grupos especiales de pacientes, el tratamiento con agonistas de la dopamina y somatostatina parece que suprimen mejor los niveles de HC que cuando se administran esos frmacos en forma separada

Figura 1. Paciente de 20 aos quien consult por amenorrea primaria. El estudio de RMN muestra un macroprolactinoma que invade el seno cavernoso izquierdo y envuelve la cartida del mismo lado.

RMN de control 10 meses despus de tratamiento con un agonista dopaminrgico. Corte coronal en T1. No se observa tumor. Tallo hipofisario central y quiasma ptico libre. REFERENCIAS BIBLIOGRFICAS Gac Md Mx Vol. 140 No. 5, 2004 Referencias 1. Schlechte JA. Prolactinoma. N Engl J Med 2003;349:2035-2041. 2. Zrate A, Canales ES, Jacobs LS, Soria J, Daughaday WH. Restoration of ovarian function in patients with the amenorrhea-galactorrhea syndrome after long-term therapy with L-Dopa. Fertil Steril 1973;24:340. 3. Tyson JE, Carter JN, Andreassen B, Huth J, Smith B. Nursing mediated

86.- Masculino de 36 aos con poliuria se le practica una prueba de deshidratacin. Despus de la restriccin de lquidos, su osmolalidad urinaria mxima es de 550 mosm/kg y la plasmtica es de 295 mosm/kg. Luego de 1 h de aplicar la inyeccin subcutnea de 5 U de vasopresina acuosa, la osmolalidad urinaria es de 860 mosm/kg. Cul de los siguientes diagnsticos es probable? a) b) c) d) Sano Diabetes mellitus Diabetes inspida parcial Diabetes inspida nefrgena

Allen R. M. MMS Medicina Interna. 5. Edicin. National Medical Series. Mc. Graw Hill. 2006. (captulo 9 I B 1 c; cuadro 9-2). El paciente tiene diabetes inspida parcial. La respuesta definitiva del enfermo a la inyeccin de hormona antidiurtica (ADH) indica que no produce concentraciones mximas eficaces de ADH despus de la restriccin de lquidos, y por tanto tiene diabetes inspida parcial o completa. La capacidad para lograr una concentracin urinaria normal o casi normal indica que el dficit de ADH slo es parcial. La respuesta de ADH descarta diabetes inspida nefrgena. La diabetes mellitus, otra causa de poliuria, se diagnostica por las concentraciones de glucosa en orina y sangre ms que por estudios del procesamiento renal de agua.

87.- Masculino de 66 aos con antecedentes de constipacin y dolor abdominal en hipogastrio y fosa ilesa izquierda de manera recurrente. El enema baritado muestra mltiples divertculos de pequeo calibre en el sigmoides. Cul es el tratamiento ms apropiado?

a) b) c) d)

Colonoscopa con biopsia Inhibidores de la bomba de protones Enema con esteroides Dieta alta en residuos

Tratamiento El tratamiento de la diverticulosis intenta reducir el espasmo segmentario. Una dieta rica en residuos es de utilidad y puede suplementarse con preparados de semillas. Las dietas bajas en residuos estn contraindicadas. Tericamente deberan ser de ayuda los antiespasmdicos (p. ej., la belladona); su valor en la prctica es difcil de enjuiciar. Su utilizacin crnica, especialmente en los ancianos, suele causar efectos secundarios adversos. No est justificada la ciruga en caso de enfermedad diverticular sin complicaciones. La reseccin con anastomosis del rea del intestino afectada en la colitis espstica (una combinacin de divertculos, espasmo y diarrea) puede llevar a resultados equvocos.

Deben cateterizarse las arterias mesentricas superior e inferior. La inyeccin selectiva de vasopresina controla la hemorragia en el 70% de los Pacientes. En algunos casos la hemorragia recurre a los pocos das y es necesaria la ciruga. Es posible practicar la reseccin segmentaria si se conoce el punto sangrante; en alrededor del 75% de los Pacientes este punto se encuentra en una zona proximal respecto a la flexura esplnica, aun cuando los divertculos predominen en el lado izquierdo. Si no se puede identificar el punto sangrante est indicada una colectoma subtotal. Los divertculos gigantes deben ser quirrgicos. Estas lesiones pueden observarse en las radiografas abdominales simples o demostrarse mediante enema de bario. Dado que la posibilidad de infeccin o perforacin es alta, se prefiere la reseccin del rea afectada del colon. Blibliografa Shakelfords. Surgery of the alimentary tract. 5a. Ed. 2002. Tomo 3. Feldmans. Gastroeneterology. 2002. Perez. Anatoma y fisiologa del hgado. Univ. Catlica de Chile. 2005. Bratiz. Serum laboratory test in cirrhosis. Journal of Hepatology. Slovakia. 2005. Paradis. Glycomics. Journal of hepatology. Ireland. Agosto 2005.

88.- Mujer de 65 aos que desde hace dos semanas presenta ictericia. Hace dos das se aade confusin mental. A la EF FR 20, FC 110, TA 90/60, Temperatura 39, no responde a comandos verbales, pero se aleja del dolor que se provoca al palpar el hipocondrio derecho y epigastrio. Los datos clnicos en el caso anterior son sugestivos de:

a) b) c) d)

Colangitis Coledocolitiasis Cncer de pncreas Cirrosis

Diagnstico El diagnstico de colangitis se basa en la asociacin de signos y sntomas de infeccin con los propios de una obstruccin biliar. La presentacin clsica es la aparicin de dolor en hipocondrio derecho o epigastrio junto a fiebre, generalmente alta, con escalofros, e ictericia (trada de Charcot). Cuando se aade confusin mental y shock (sepsis) se denomina Pentada de Reynolds, que se observa con menor frecuencia, pero habitualmente se asocia con una colangitis supurada grave. Sin embargo, la correlacin entre la clnica tpica, las formas atpicas y la presencia de pus en la va biliar es pobre y en muchos casos de colangitis faltan algunos de estos rasgos. Algunos enfermos, sobre todo de edad avanzada, pueden tener confusin mental o shock sin fiebre, o existir una leucocitiosis con desviacin izquierda como nica manifestacin de la infeccin, por lo que se debe sospechar la existencia de una colangitis subyacente, sobre todo en enfermos de edad con algunas de estas manifestaciones. El laboratorio muestra hallazgos de obstruccin biliar con aumento variable de la bilirrubina y enzimas de colestasis. Es habitual encontrar leucocitosis con

desviacin izquierda, siendo por lo general las cifras ms altas, en torno a 20.000 por mm3 , reflejo de las formas ms graves. La ecografa es la tcnica de eleccin para detectar la existencia de obstruccin biliar por su elevada eficacia y versatilidad, pudindose detectar tambin complicaciones de la colangitis, como el absceso heptico. Dentro de la colangitis se pueden establecer unas formas leves, generalmente de buen pronstico, y unas formas graves, donde se concentra la mayor parte de la mortalidad. Las primeras se manifiestan como cuadros febriles sin signos de afectacin sistmica que se autolimitan espontneamente o bajo tratamiento mdico en 24-48 horas. Las formas inicialmente graves son las que asocian confusin mental, hipotensin, shock o fracaso renal. La edad es un factor constante de aumento de la morbimortalidad. Referencias bibliogrficas: 1. Bilhartz LE, Horton JD. Gallstone disease and its complications. En: Gastrointestinal and liver diseases. Sleisenger and Fordtran. Filadelfia: WB Saunders Co., 1998; 948-972. 2. Chung-Mau L, Chi-Leung L, Lai ECS, Sheuns-Tat F, Wong J. Early versus delayed laparoscopic cholecystectomy for treatment of acute cholecystitis. Ann Surg 1996; 223: 37-42. 3. Hamy A, Visset J, Likholatnikov D, Lerta F, Gibaud H, Savigny B et al. Percutaneus cholecystostomy for acute cholecystitis in critically ill patients. Surgery 1997; 121: 398401. 4. Harris A, Chong Hen Chang A, Torres-Viera C, Hammett R, Carr-Locke D. Meta-analysis of antibiotic prophylaxis in endoscopic retrograde cholangiopancreatography (ERCP). Endoscopy 1999; 31: 718-724. 5. Hermann RE. Surgery for acute and chronic cholecystitis. Surg Clin North Am 1990; 70: 1.263-1.275. 6. Koo Kp, Thirlby RC. Laparoscopic cholecystectomy in acute cholecystitis. What is the optimal timing for operation? Arch Surg 1996; 131: 540-545. 7. Lai ECS, Mok FPT, Tan ESY, Lo CM, Fan ST, You KT et al. Endoscopic biliary drainage for severe acute cholangitis. N Engl J Med 1992; 326: 1.582-1.586. 8. Marton KI, Doubilet P. How to image de gallbladder in suspected cholecystitis. Ann Int Med 1988; 109: 722-727. 9. Van den Hazel SJ, Speelman P, Tytgat GNJ, Dankert J, Van Leeuwen DJ. Role of antibiotics in the treatment and prevention of acute and recurrent cholangitis. Clin Infect Dis 1994; 19: 279-286. 10. Westphal J-F, Brogard J-M. Biliary tract infections A guide to drug treatment. Drugs 1999; 57: 81-91.

89.- Mujer de 28 aos manifiesta que inicialmente present un enrojecimiento, sensibilidad y dolor en el borde externo del prpado. Actualmente cursa con orzuelo, usted decide el siguiente tratamiento por ser el de eleccin:

a) b) c) d)

Compresas tibias y antibiticos tpicos Compresas fras Drenaje Reseccin amplia

Orzuelo Un orzuelo es una infeccin, en general provocada por un estafilococo, de una o ms de las glndulas que se encuentran en el borde del prpado o por debajo de ste. Se forma un absceso que tiende a romperse y, en consecuencia, genera una pequea cantidad de pus. El orzuelo a veces se forma al mismo tiempo que la blefaritis o bien como resultado de sta. Una persona puede tener uno o dos orzuelos en toda su vida, pero otras los desarrollan repetidamente. El orzuelo en general se manifiesta primero con un enrojecimiento, sensibilidad y dolor en el borde externo del prpado. Luego, una pequea rea se torna redondeada y sensible y se hincha. El ojo puede lagrimear, volverse muy sensible a la luz intensa y provocar la sensacin de que hay algo en su interior. Generalmente, slo una parte muy pequea del prpado se hincha, pero a veces se inflama en su totalidad. En general aparece un diminuto punto amarillento en el centro de la zona hinchada. A pesar de que se recurre a los antibiticos, no parecen ser demasiado tiles en estos casos. El mejor tratamiento consiste en aplicar compresas calientes durante 10 minutos varias veces al da. El calor ayuda a que el orzuelo madure, se rompa y drene. Cuando se forma un orzuelo en una de las glndulas ms profundas del prpado, una afeccin llamada orzuelo interno, el dolor y los dems sntomas suelen ser ms intensos. El dolor, el enrojecimiento y la hinchazn suelen aparecer slo en un rea muy pequea, en general en el borde del prpado. Como esta clase de orzuelo rara vez se rompe por s solo, el mdico puede abrirlo para drenar el pus. Los orzuelos internos suelen ser recurrentes.

90.- Masculino de 66 aos que cursa con antecedente de infarto agudo de miocardio, el siguiente grupo de medicamentos son un tratamiento generalmente indicado, por su alta disminucin de mortalidad:

a) b) c) d)

Betabloqueadores. Nitritos. Anticoagulacin oral. Antiarrtmicos.

Los BB son recomendados en todos los pacientes que han presentado un IAM,

siempre que no tengan contraindicaciones para su uso, y de modo permanente (indefinidamente): clase de recomendacin I, nivel de evidencia A. Se ha puesto en evidencia que los BB son infrautilizados en esta indicacin. Ms de 35.000 pacientes han sido incluidos en estudios postinfarto con BB. Se ha demostrado una reduccin de la mortalidad total, muerte sbita y reinfarto

del orden del 20-25%.

Situacin clnica/indicacin Todos los pacientes sin contraindicaciones, indefinidamente Para mejorar supervivencia Para prevenir reinfarto Prevencin primaria de la muerte sbita Prevencin/tratamiento de arritmias ventriculares tardas

Clase de recomendacin

Nivel de evidencia

I I I IIb

A A A B

Los BB siguen estando indicados en el tratamiento de la HTA, aunque existen

crticas a su empleo en el primer escaln. Pese a ello, son sin duda de primera eleccin si el/la paciente presenta angina, cardiopata isqumica en general, insuficiencia cardaca, taquiarritmias, glaucoma o embarazo.

Los BB son frmacos de primera eleccin en la insuficiencia cardaca con disfuncin

sistlica. Slo el carvedilol, el bisoprolol o el nebivolol son BB que puedan ser empleados a las dosis disponibles en nuestro medio para dicho tratamiento. Todos los pacientes sin contraindicaciones deben recibir BB si presentan

cardiopata isqumica y, muy especialmente, si existe angina o han presentado un IAM.

ESC Expert consensus document on b-adrenergic receptor blockers. The Task Force on Beta-Blockers of the European Society of Cardiology. Eur Heart J 2004; 25: 13411362

91.- Masculino de 52 aos que es derivado al servicio de dermatologa por presentar prurito en manos y antebrazos, eritema y esfacelacin con exacerbacin desde hace ms de 3 meses. As mismo mculas en cuello y parte alta del trax, agregndose edema facial. Antecedentes de trabajar durante 15 aos en fbrica de cementos. Exploracin fsica: Placa eritematosa con hiperqueratosis en miembro plvico izquierdo con huellas de rascado y acompaado de edema. Cual es el diagnstico ms probable? a) b) c) d) Dermatitis por contacto Psoriasis Urticaria Dermatitis Atpica

Las dermatitis por contacto son consecuencia del contacto de la piel con una sustancia qumica y pueden producirse por varios mecanismos patognicos. Con frecuencia se hacen sinnimos dermatitis por contacto y eccema por contacto, puesto que la mayora de estas reacciones tienen una presentacin clnica eccematosa. Sin embargo, sobre todo la dermatitis alrgica por contacto, tambin es posible que adopten otros patrones, como urticariforme, liquenoide, eritema exudativo multiforme, etc. Segn el mecanismo patognico, pueden distinguirse dermatitis por contacto irritativas, alrgicas, fototxicas y fotoalrgicas. Debe tenerse en cuenta que una misma sustancia puede ser responsable de dermatitis por contacto por diversos mecanismos. Bibliografa: 1. Bielsa marsol I. Eccemas (I). En: Ferrndiz C. Dermatologa Clnica. Madrid: Harcourt 2001: 117-125. 2. Fernndez Vozmediano JM, Nieto Montesinos I. Dermatitis por contacto en la infancia. En: Fonseca Capdevila E, ed. Dermatologa peditrica. Madrid: Grupo Aula Mdica 1997: 456567.

3. Garca Bravo B, Rodrguez Rey E. Dermatitis de contacto en la infancia. Piel 2000; 15: 316323. 4. Gimnez Camarasa JM, Conde Salazar L, de la Cuadra Oyanguren J, Fernndez Vozmediano JM, Fonseca Capdevila E, Gimnez Arnau Am, Ortiz de Frutos FJ. Documento de Consenso en Eczema. Madrid: Grupo Aula Mdica 1998. 5. Grupo Espaol para la Investigacin de Dermatitis de Contacto. Batera estndar. (En lnea). (15-01-2001). Disponible en: http:www.arrakis.es/%7Egeidc/index.htm Dermatitis por contacto La piel es la primera barrera de defensa del organismo y est expuesta a mltiples agresiones del medio ambiente, tales como fro, calor, humedad, jabones, cosmticos y otros mltiples agentes fsico-qumicos entre los que destacan los medicamentos de uso tpico.

92.- Infant, 33 weeks of gestation presenting in the first day of life, tachypnea, xiphoid retraction, intercostal retractions, nasal flaring, deep moan and cyanosis. On physical examination, crackles are audible bibasilar. The chest radiograph shows reticulonodular infiltrates with air bronchogram. What is the most likely diagnosis? a) b) c) d) Hyaline membrane disease. Pneumonia Transient tachypnea of the newborn. Meconium aspiration syndrome.

La enfermedad de membrana hialina se debe a un dficit de surfactante, afecta a recin nacidos prematuros, su frecuencia aumenta en hijos de madres diabticas y en embarazos mltiples. El cuadro clnico se caracteriza por datos de dificultad respiratoria de inicio precoz como: taquipnea, quejido intenso, aleteo nasal, retracciones xifoideas e inter y subcostales y cianosis parcialmente refractaria al oxgeno. Los sntomas progresan hasta alcanzar un mximo al tercer da. En la auscultacin aparecen crepitantes en ambas bases. En la radiografa de trax se presenta un infiltrado reticulonodular con broncograma areo en unos pulmones poco ventilados, se pueden presentar atelectasias. En la gasometra se observa hipoxemia, hipercapnia y acidosis respiratoria. El tratamiento consiste en soporte respiratorio, administracin endotraqueal de surfactante y antibiticos. Por otro lado, la taquipnea transitoria del recin nacido se presenta en recin nacidos de trmino, que nacen por cesrea o parto vaginal rpido, debido a un retraso en la absoprcin del lquido; manifestndose con un distrs respiratorio de inicio precoz, la auscultacin es normal.

Manual pediatra, 7 edicin p. 1302. Gua clnica SDR neonatalMarzo 2006

93.- Masculino de 14 aos de edad con antecedentes de convivencia con aves, gatos e ingestin muy frecuente de berros. Inicia su padecimiento actual hace dos semanas con fiebre de 39C sin predominio de horario, nusea y dolor en hipocondrio derecho. A la exploracin fsica con palidez generalizada y hepatomegalia 3-3-5. Resto de la exploracin sin datos patolgicos. Laboratorio con BH, con Hb de 11.5, Hto de 40, leucocitos; 16,500, eosinfilos; 38%, linfocitos; 30%, formas inmaduras; 4, plaquetas; 270,000, Fosfatasa alcalina; 280 UI/L. Cul es la forma infectante en esta entidad clnica?

a) b) c) d)

Larva filariforma Metacercaria Ooquiste Huevo larvado

El parsito y su ciclo evolutivo

La Fasciola hepatica adulta, es un gusano aplanado en forma de hoja . Mide de 2-5 cm. Es hermafrodita. Los huevos en el exterior y en un ambiente acutico, desarrollan en su interior el miracidio, que al eclosionar, busca al caracol Lymneido (2)., lo penetra y se desarrollan los estadios larvarios de esporoquiste, redia madre, redia hija y cercarias, las que abandonan al caracol y se adosan a las hojas y tallos de las plantas acuticas, transformndose en metacercaria (forma infectante). FIGURA 1

Esta es ingerida por el animal o el hombre, en el "berro", verduras de tallo corto o en el agua, deja en libertad la forma juvenil en el intestino, la que penetra la pared intestinal, cae a la cavidad peritoneal y se dirige al hgado, perfora la cpsula de Glisson y migra por el hgado hasta las vas biliares, donde finalmente se desarrolla el adulto. El tiempo entre la ingesta de la metacercaria y la localizacin del adulto es de 2 a 3 meses.

Ciclo biolgico de Fasciola heptica (tomado de Atas-Parasitologa Mdica).

Bibliografa:
1. Legua, G. 1991. Distomatosis heptica en el Per. Epidemiologa y Control. Segunda edicin. U. Nacional Mayor de San Marcos, Lima. 2. Vivar, R. 1987. Aportes al conocimiento de algunos Tremtodes en el Per. Tesis. UPRP, Lima. 3. Lpez, M; Zerda, K; Nquira, C. y Guerra, H. Major Fasciola hepatica antigens are mainly localized within the digestive tube of the adult. Parasitologa al Da. 1997; 21:104-108. 4. Caprn, A; Biguet, G; Tran Van Ky e Rose G. Posibilities nouvelles dans le diagnostic immunologie de la distomatose humaine a Fasciola hepatica. Press. Med. 1964;72:31033107. 5. Crdova, M; Herrera, P; Nopo, L; Bellatin, J; Nquira, C; Guerra, H. and Espinoza, J. Fasciola hepatica cysteine proteinases: immunodominant antigens in Human Fascioliasis. Am. J. Trop. Med. Hyg., 1997; 57(6):660-666. 6. Crdova, M; Retegui, L. and Espinoza J.R. Immunodiagnosis of human fascioliasis with Fasciola hepatica cystine proteinases. Trans. Roy. Soc. Trop. Med. Hyg. 1999; 93:54-57.

94.- Femenino de 43 aos portadora de DM tipo II, e HTAS, es ingresada al servicio de Medicina Interna por cetoacidosis. Posterior a su recuperacin metablica inicia con fiebre, cefalea, dolor facial, disminucin del nivel de conciencia y enrojecimiento nasal con lesin negruzca en fosa nasal derecha. El diagnstico ms probable en sta paciente es:

a) b) c) d)

Endocarditis por S. aureus. Carcinoma epidermoide. Infeccin por M. tuberculosis. Infeccin por Mucor.

Mucormicosis es el nombre comn dado a varias diferentes enfermedades causadas por hongos de la orden de los Mucorales. Muchas diferentes especies han sido implicadas como agentes de sndromes clnicos similares.

Los Mucoraceos son hongos que se encuentran en todas partes y son comunes habitantes de materia en descomposicin. Por ejemplo, Rhizopus sp. Frecuentemente puede ser recuperado de pan mohoso. Por su rpido crecimiento y prolfica capacidad de formar esporas, inhalacin de conidias debe ser una experiencia cotidiana. La presencia de esporas Mucorales en cinta adhesiva no estril se demostr fue la fuente de mucormicosis cutnea primaria. Aun cuando estos hongos crecen en muchos nichos ecolgicos, la infrecuencia de enfermedad debida a estos organismos da fe de su baja potencial virulencia en el husped humano. En contraste a la amplia distribucin de este hongo, la enfermedad en humanos esta limitada, en muchos casos a poblacin con severo inmunocompromiso, diabetes mellitus o trauma.

Manifestaciones clnicas En 1973 Meyer y Armstrong12 categorizaron las diferentes presentaciones clnicas de mucormicosis, considerando el rgano involucrado y haciendo una divisin en seis entidades: rinocerebral, cutnea, gastrointestinal, pulmonar, diseminada y formas miscelneas. Existe una fuerte asociacin entre la entidad subyacente y la forma de presentacin. La mucormicosis rinocerebral se presenta ms frecuentemente en pacientes diabticos con acidosis, y debido al advenimiento de quimioterapias cada vez ms potentes, se aprecia con ms frecuencia en pacientes leucmicos con neutropenia prolongada, en aquellos que reciben mltiples esquemas de antibiticos y corticoesteroides, as como en pacientes con trasplantes de rganos. En pocas ocasiones se ha documentado esta forma invasora en personas sin enfermedades subyacentes. La forma pulmonar puede presentarse en pacientes con leucemia y neutropenia; la mucormicosis gastrointestinal se observa ms comnmente en pacientes con desnutricin calrico-proteica y en prematuros, y la diseminada en pacientes con dficit inmunolgico grave como trasplantados, leucmicos y nefrpatas tratados con deferoxamina.1,2 La infeccin rinocerebral es la presentacin ms frecuente y caracterstica de mucormicosis, siendo la rinoorbitaria y la mucormicosis paranasal estadios tempranos de esta.

La infeccin generalmente inicia en senos paranasales o paladar duro, y se extende a senos adyacentes con diseminacin a travs de senos etmoidales y zona retroorbitaria; puede tener acceso al cerebro a travs del pex orbitario, lmina cribosa y por va vascular. Una costra necrtica sangrante en paladar o en mucosa nasal y un drenaje ocular de pus negruzco orientan al diagnstico. Puede haber una progresin rpida y presentarse el deceso en pocos das o ser indolente si la enfermedad subyacente se logra controlar. Inicialmente puede haber dolor facial, cefalea, fiebre y algn grado de celulitis orbitaria, conforme progresa la invasin de la rbita la prdida de la funcin del II, III, IV y VI nervios craneales puede ocurrir, as como tambin puede haber prdida de la funcin de msculos extraoculares, proptosis, quemosis progresiva, congestin nasal, epistaxis y letargia. La disfuncin de nervios craneales, especialmente el V y VII, ocurre de manera tarda manifestando ptosis y midriasis, lo cual es un factor pronstico grave, en caso de trombosis retiniana, hay prdida de la visin y puede haber afeccin intraocular. El compromiso cerebral es la complicacin ms seria y puede manifestarse como: infarto, absceso, trombosis del seno cavernoso, hematoma subdural y necrosis del lbulo frontal.2 Otras complicaciones son trombosis de arteria cartida interna y vena yugular, e incluso se ha reportado infarto de miocardio por oclusin de coronaria y aborto sptico. Puede haber presentaciones crnicas y secuelas tardas a pesar del tratamiento aparentemente satisfactorio por lo cual siempre debe realizarse un seguimiento del paciente a largo plazo

BIBLIOGRAFIA:

Mayo Clinic. Pulmonary Diseases Mandele?Douglas. Infections Diseases Pennington. Respiratory infections, Diagnosis and Management Ronald B. George., Richard W. Light, Michael A. Matthay. Chest Medicine 3era. Edicin. Scott E Davis. Neumona Mictica. Clnicas mdicas de Norteamrica 1997; 5: 1092 ,1094.

95.- Al existir la sospecha de lquido libre en la cavidad peritoneal y despus de analizar los hallazgos de manera conjunta de las placas de abdomen y en el ultrasonido abdominoplvico, Dnde se debern de buscar los cambios de densidad de manera inicial? a) b) c) d) Infradiafragmtico Transcavidad de los epiplones Correderas parieto-clicas Fondos de saco de Douglas y de Morrison

Pedrosa C, Casanova R. Diagnstico por imagen. Mc Graw Hill 2001 p. 209, 210.

La diseminacin del lquido intraperitoneal est producida por la gravedad y la presin hidrosttica secundaria al movimiento diafragmtico.la pelvis es la parte ms dependiente de la cavidad peritoneal, su capacidad es de 300 cc, a partir de lo cual el lquido asciende por las correderas parietoclicas. El lquido plvico desplaza las asas intestinales que rellenan los espacios plvicos, el leon del lado derecho y el colon sigmoides en el izquierdo, lo que resulta de la paricin de una densidad homognea en al pelvis menor, en contra del patrn poco homogneo de las asas intestinales que tienen gas y lquido. En la pelvis femenina el lquido se acumula en los recesos laterales de la vejiga y al tero, y en el hombre puede verse en la fosa vesicorrectal y los recesos plvicos laterales. En el ultrasonido el diagnstico de pequeas cantidades de lquido debe hacerse fundamentalmente en el saco de Douglas, en la bolsa de Morrison y en el receso yuxtaesplnico.

96.- Al realizar la exploracin clnica y colocar un diapasn que est vibrando frente al conducto auditivo del odo que queremos explorar (conduccin area) y apoyando despus sobre la mastoides (conduccin sea), podemos de modo sencillo y en la consulta, distinguir entre sordera nerviosa (alteracin en la cclea o nervio auditivo) y sordera de conduccin (trastorno en el sistema de transmisin tmpano-osicular). Cul de estas afirmaciones es correcta para un paciente que presenta una sordera de conduccin? a) b) c) d) La percepcin del sonido es mejor por va sea que por va area. La percepcin del sonido es igual por va area que por va sea. La percepcin del sonido es mejor por va area que por va sea. La percepcin del sonido es indistinguible tanto por va area como sea.

MEDICIN CLNICA DE LA AUDICIN La valoracin audiolgica mnima debe incluir la determinacin de los umbrales de conduccin area y sea, el umbral de recepcin y la discriminacin del lenguaje, una timpanometra y pruebas de reflejos acsticos, que incluyan la prueba de deterioro de los reflejos. La informacin obtenida por medio de estas tcnicas permite determinar si hace falta una mayor diferenciacin entre la sordera neural y la sensorial. La audicin por conduccin area se valora presentando un estmulo acstico mediante auricular o altavoces. Una sordera o elevacin del umbral de audicin detectada por esta prueba se puede deber a defectos en cualquier parte del aparato auditivo: pabelln auricular, conducto auditivo, odo medio o interno, VIII par craneal o vas auditivas centrales.

La audicin por conduccin sea se valora colocando una fuente sonora (el vibrador de un audimetro o un diapasn) en contacto con la cabeza. El sonido produce una vibracin a travs del crneo, que alcanza las paredes seas de la cclea y estimula directamente el odo interno. La audicin por conduccin sea no atraviesa los odos externos y medio y permite valorar la integridad del odo interno, del octavo nervio craneal y de las vas auditivas centrales. Si aumenta el umbral de conduccin del aire y el umbral de conduccin sea es normal, la sordera es de conduccin, mientras que si ambos umbrales aumentan por igual es de tipo neurosensorial. Existen algunas formas de sordera mixtas con componente neurosensorial y de conduccin, en las que aumentan ambos umbrales, aunque el de la conduccin area es ms significativo. Manual Merck 10. Edicin en Espaol

Editors of The Merck Manual

Robert S. Porter, MD, Editor-in-chief Justin L. Kaplan, MD, Senior Assistant Editor Editorial Board of The Merck Manual

97.- Paciente masculino de 3 aos de edad que acude a urgencias con una historia de secrecin nasal purulenta y de mal olor unilateral desde hace 5 das. El diagnstico ms frecuente es: a) b) c) d) Atresia de coanas unilateral. Rinitis crnica por Rinovirus. Fibrosarcoma del correte nasal. Cuerpo extrao intranasal.

Una de las patologas ms frecuentes que enfrenta el otorrinolaringlogo en su prctica mdica, es la presencia de cuerpos extraos animados o inanimados en vas areodigestivas superiores. La mayora de las veces su tratamiento (extraccin) no reviste mayor importancia sobre todo cuando no han sido manipulados por mdicos de primer contacto, pero en otras, se convierte en un procedimiento muy difcil de realizar, ya sea por las complicaciones que se produjeron o por la presencia del mismo. En la poblacin infantil es ms frecuente por la curiosidad propia de esa edad y la tendencia a colocarse una variedad de elementos en las fosas nasales. A veces existen ciertas dificultades en el diagnstico, pero la trada sintomtica de obstruccin nasal, rinorrea unilateral y halitosis, nos sugiere la posibilidad de un cuerpo extrao hasta que no se demuestre lo contrario. Independientemente de su naturaleza animada o inanimada, stos pueden alojarse y/o impactarse en distintos niveles.

El tratamiento electivo es la extraccin por vas naturales de los mismos; en los nios que no colaboran es necesario recurrir a la anestesia general. 1. Manual de Urgencias en Otorrinolaringologa. Manuel Toms Barbern, Julio GarcaPolo Alguacil, Guillermo Til Prez

98.- Se reporta ultrasonografa renal donde usted observa una masa, con presencia de una pared delgada bien delimitada, en el que se interrumpe el contorno renal, sin presencia de ecos en su interior y refuerzo posterior. El diagnstico ms probable es:

a) Absceso renal. b) Hipertrofia de columna de Bertin. c) Carcinoma renal. d) Quiste renal simple. Los criterios para el diagnstico ultrasonogrfico de un quiste simple renal son los siguientes: 1.- Forma ovoide o esfrica 2.-Ausencia de ecos internos (quiste anecoico) 3.-Presencia de una pared delgada y lisa bien definida que lo separa del parnquima. 4.-.El refuerzo acstico ms all de la pared posterior del quiste es proporcional a su contenido lquido. 5.-Se observa una banda estrecha de forma acstica por fuera del borde externo. Si se cumplen con todos estos criterios no es necesario realizar ms estudios para diagnosticar la masa. Sin embargo, ocasionalmente la ecografa puede no resultar la tcnica ms idnea (por ejemplo cuando hay calcificacin de la pared del quiste, ste es hemorrgico o existen mltiples quistes). -Brenner and rector The Kidney 2 volmenes 2004 Saunders 2525 pag.

99.- Se trata de primigesta de 26 aos con control prenatal regular, cursa con ruptura de membranas a las 30 semanas de gestacin, se interrumpe el embarazo por cesrea. Es probable que el recin nacido prematuro disminuya la posibilidad de presentar enfermedad de membranas hialinas por la siguiente medicacin materna: a) b) c) d) Antibioticos IV Oxitocina IV Sulfato de magnesio IV Esteroides IM

Uso prenatal, un pilar en neonatologa

Por su efecto en la maduracin fetal, los corticoesteroides se han utilizado prenatalmente desde hace ms de tres dcadas. Liggins y Howie en 1972, reportaron por primera vez los efectos benficos de los esteroides, administrados prenatalmente, en la maduracin pulmonar y en la disminucin de la incidencia de SDR. Desde ese primer reporte a la fecha, se han descrito otros efectos positivos. Su uso "adecuado" y oportuno ha demostrado tener beneficio en el recin nacido pretrmino.Ahora se sabe que la administracin de esteroides a la madre embarazada se ha asociado a disminucin en la incidencia de SDR, hemorragia intraventricular, displasia broncopulmonar y mortalidad neonatal. Se ha observado que el mximo beneficio en el recin nacido se obtiene de 24 horas a siete das despus de la administracin materna de esteroides. Sin embargo, an antes de 24 horas de administrados, se ha reportado beneficio. Aunque los cambios bioqumicos generados con los esteroides desaparecen despus de siete das, los cambios estructurales persisten. Algunos estudios clnicos han demostrado efecto benfico de esteroides an despus de siete das de administrados. Un metanlisis de 12 estudios, realizado por Crowley y col. en 1990, demostr que el uso prenatal de esteroides disminua la incidencia de SDR en aproximadamente 50%. Es claro que el efecto ms importante se encuentra en los fetos menores de 34 semanas de edad gestacional; sin embargo, tambin se ha demostrado disminucin en la incidencia de SDR en neonatos de mayor edad gestacional.18 Probablemente el efecto benfico ms importante asociado al uso prenatal de esteroides sea la disminucin en la mortalidad neonatal. Crowley y col. demostraron en el metanlisis, con ms de 3 000 neonatos, que sta disminua considerablemente (razn de momios 0.59, intervalo de confianza =0.470.75). La disminucin en mortalidad se ha detectado an en pacientes menores de 800 g. Bol. Med. Hosp. Infant. Mex. v.62 n.5 Mxico sep. /oct. 2005 Esteroides en Neonatologa: entusiasmo, uso, abuso y desuso. Dnde est el justo medio? Steroids in neonatology: enthusiasm, use, abuse and disuse. Is there an equilibrium? 1. Liggins GC, Howie RN.A controlled trial of antepartum glucocorticoid treatment for prevention of the respiratory distress syndrome in premature infants. Pediatrics. 1972; 50: 51525. [ Crowley P, Chalmers I, Keirse MJNC. The effects of corticosteroid administration before preterm delivery:An overview of the evidence from controlled trials. Br J Obstet Gynaecol. 1990; 97: 1125. [ Van Marter LJ, Leviton A, Kuban KCK. Maternal glucocorticoid therapy and reduced risk of bronchopulmonary dysplasia. Pediatrics. 1990; 86: 331 6. Morales WJ, Diebel D, Lazar AJ, Zadrozny D.The effect of antenatal dexamethasone administration on the prevention of respiratory distress syndrome in preterm gestations with premature rupture of membranes. Am J Obstet Gynecol 1986; 154:5915. Gamsu HR, Mullinger BM, Donnai P, Dash CH. Antenatal administration of betamethasone to prevent respiratory distress syndrome in preterm infants: Report of a UK multicentre trial. Br J Obstet Gynaecol. 1989; 96:40110.

2.

3. 4.

5.

6. Young BK, Klein SA, Katz M. Intravenous dexamethasone for prevention of neonatal respiratory distress: A prospective controlled study. Am J Obstet Gynecol. 1980; 138:2039. 7. Doyle LW, Permezel MJ, Kitchen WH. Is there a lower limit for birth weight/gestational age and antenatal steroid therapy? Aust NZ J Obstet Gynaecol. 1992; 32: 1935.

100.-Femenino de 5 aos, es atendida en consulta para control de infecciones de vas urinarias comprobada. Antecedentes: Infeccin de vas urinarias en 6 ocasiones desde el nacimiento, actualmente con mejora acentuda despus de que recibi tratamiento con antibiticos. Exploracin fsica: actualmente asintomtica. El siguiente paso para confirmar el diagnstico en este paciente es: a) b) c) d) Examen general de orina. Ultrasonido abdominal. Ultrasonido renal y de vias urinarias. Urocultivo.

Revisin sistemtica bien realizada con poco riesgo de sesgo. NICE 2007 En el paciente peditrico con infeccin de vas urinarias comprobada, se debe investigar la presencia de alteraciones anatmicas y funcionales. Evidencia compuesta por estudios clasificados como 1+ y con gran consistencia entre ellos. Baumer JH, 2007

Los sntomas y signos ms comunes en menores de cinco aos por los que acuden a urgencias con el primer episodio de IVU son: fiebre 80%, irritabilidad 52%, anorexia 49%, malestar 44%, vmito 42%, diarrea 21%. Los sntomas menos comunes (en menos de 20%): disuria, orina ftida, dolor abdominal, frecuencia y hematuria.

La presencia de fiebre >38C, bacteriuria y dolor lumbar sugiere pielonefritis, mientras que la presencia de sntomas urinarios como disuria asociada a bacteriuria, pero no a sntomas sistmicos, sugiere cistitis o IVU baja.

En nios mayores de tres aos puede utilizarse de primera instancia la bsqueda de nitritos, estearasa leucocitaria, estudio microscpico y urocultivo. En nios menores de tres aos con sospecha de IVU debe tomarse de inmediato la muestra para urocultivo. La bsqueda de nitritos y estearasa leucocitaria solo se realizar cuando el estudio microscpico sea imposible de realizar.

Prevencin, diagnstico y tratamiento de la infeccin de vas urinarias no complicada en menores de 18 aos en el primero y segundo nivel de atencin de atencin Mxico:

Secretara de Salud; 2008. M

La infeccin urinaria (ITU) se define como la invasin, multiplicacin y colonizacin del tracto urinario por grmenes que con mayor frecuencia provienen de la regin perineal. La confirmacin diagnstica est dada por un urocultivo con un recuento de colonias superior a 1 ufc/ml si la muestra es tomada por puncin vesical, superior a 10.000 ufc/ml si es tomada por bolsa recolectora o sondeo, y mayor de 100.000 ufc/ ml si es tomada durante la mitad de la miccin (segundo chorro)1. Bacteremia se define como la presencia de bacterias viables en sangre confirmada por hemocultivos. Debe corresponder a un slo germen, el cual debe ser el mismo encontrado en el urocultivo. Dada la alta prevalencia de alteraciones anatmicas y funcionales del rin y vas urinarias que presentan los nios con ITU, se debe realizar estudio imagenolgico inicial con ecografa renal y vesical, y uretrocistografa, independiente de su edad y sexo, excepto en la mujer mayor de 5 aos con un primer episodio de ITU baja2. Se define malformacin de la va urinaria como una alteracin en las diferentes etapas de la nefrognesis, la cual comienza en la octava semana de la gestacin, donde surge la yema ureteral derivada del conducto mesonfrico de Wolf, penetrando el blastema metanfrico para desarrollar los riones fetales2,3 .

Referencias: 1. Garin, Eduardo; Olavarra, Fernando; Garca, Vctor Nieto; Valenciano Blanca; Campos, Alfono and Young, Linda. Clinical Significance of Primary Vesicoureteral Reflux and Urinary Antibiotic Prophylaxis After Acute Pyelonephritis: A Multicenter, Randomized, Controlled Study, Pediatrics 2006;117;626-632. Glassberg, K. Special Article: Annual Meeting of the Section on Pediatric Urology. Pediatrics , 1988;81: 588-594. Gordillo, P. Malformaciones Urolgicas y Renales. En: Nefrologa Pediatrica. Mxico. 1999,.2 edicin Mosby. Pginas.133 - 157 . American Academy of Pediatrics, Committee on Quality Improvement, Subcommittee on Urinary Tract Infection. Practice Parameters: The Diagnosis, Treatment, and Evaluation of the Initial Urinary Tract Infection in Febrile Infants and Young Children. Pediatrics , 1999;103:843-852 Hodson, C: The radiologic diagnosis of pyelonephritis. Proc R soc Med 1959;52:669. Winberg, J; Andersen, H.; Bergstrom, T. et al. Epidemiology of symptomatic urinary tract infection in childhood . Acta pathol scand suppl 1974;252:1. Salas, Paulina; Alvarez, Enrique y Saieh, Carlos. Pautas de diagnstico y tratamiento en infeccin urinaria en nios. Documento de la Rama de Nefrologa de la Sociedad Chilena de Pediatra. Rev. chil. pediatr. , jun. 2003, vol.74, n 3: 311-314.

2. 3. 4.

5. 6. 7.

8. Cavagnaro, Felipe. Infeccin urinaria en la infancia. Rev. chil. infectol. , jun. 2005, vol.22, no.2, p.161-168.

Você também pode gostar